Sunteți pe pagina 1din 226

Contents

Scheme-Of-Work 1

Notes On Teaching 21

Fully Worked Solutions


Chapter 1 Factors and Multiples 33
Chapter 2 Real Numbers 51
Chapter 3 Approximation and Estimation 81
Chapter 4 Introduction to Algebra 99
Review Exercise 1 115
Chapter 5 Algebraic Manipulation 119
Chapter 6 Simple Equations in One Variable 149
Chapter 7 Angles and Parallel Lines 167
Chapter 8 Triangles and Polygons 187
Review Exercise 2 222
[2nd Edition] Suggested Scheme of Work • Express

Week Topic/Objectives Strategies & Learning Experiences Activities Resources Websites

Term 1 Chapter 1 (Learning Experiences in italic ) Chapters 1 – 8


Week 1 Factors And Multiples in Book 1A
1.1 Primes, Prime Factorisation
and Index Notation
1.2 Highest Common Factor
• recognise prime numbers Explaining factors of numbers p.2–8 Textbook
Classifying whole numbers based on p.3 Class Activity 1 p.8–10 Textbook http://staff.argyll.
their number of factors and explain why epsb.ca/jreed/math9/
0 and 1 are not primes E-book
strand1/1102.htm
• express a composite number as Demonstrating prime factorisation and Teacher’s Guide
a product of its prime factors index notation http://www.42explore. http://amby.com/educate/
• understand index notation com/number.htm math/2-1_fact.html
• represent the prime
factorisation of a number in
index notation
• find the HCF of a group Demonstrating the ways of finding HCF
of numbers by using prime Illustrating the application of HCF in
factorisation real life situations

Term 1 1.3 Least Common Multiple


Week 2 1.4 Square Roots & Cube Roots
• find the LCM of a group Demonstrating the ways of finding LCM p.11–15 Textbook http://staff.argyll.
of numbers by using prime Illustrating the application of LCM in p.15–18 Textbook epsb.ca/jreed/math9/
factorisation real life situations strand1/1102.htm
E-book
• understand the meanings of the Relating perfect squares with square
square root and cube root of a Teacher’s Guide http://www.bbc.co.uk/
roots and perfect cubes with cube roots to
number schools/gcsebitesize/
illustrate the method of finding the square
maths/number/surdsrev1.
• find square roots and cube root and cube root of a number
shtml
roots by prime factorisation Illustrating the finding of square roots and
cube roots by prime factorisation http://staff.argyll.
Reinforcing the concepts by applications epsb.ca/jreed/math9/
strand1/1103.htm

1
2
Week Topic/Objectives Strategies & Learning Experiences Activities Resources Websites

Term 1 Chapter 2
Week 3 Real Numbers
2.1 to 2.3 Idea of Negative http://staff.argyll.
Numbers and The Number epsb.ca/jreed/math9/
Line, The Four Operations of strand1/1102.htm
Integers
http://staff.argyll.
• recognise the use of negative Discussing examples of the use of p.23 Class Activity 1 p.23–27 Textbook
epsb.ca/jreed/math7/
numbers in the real world negative numbers in the real world p.27–34 Textbook strand1/1107.htm
• represent numbers on a Presenting the number line and the order p. 34–40 Textbook http://lgfl.skoool.
number line of numbers
AlgeDiscTM application co.uk/viewdetails_ks3.
• identify integers, and Represent integers on the number line
in AlgeToolsTM aspx?id=569
perform addition, subtraction, Use algebra discs or the AlgeDiscTM p.28 Class Activity 2
E-book http://www.bbc.co.uk/
multiplication and division on application in Alge ToolsTM to make p.30 Class Activity 3
Teacher’s Guide schools/gcsebitesize/
them sense of addition, subtraction and
p.35 Class Activity 4 maths/algebra/
multiplication involving negative
inequalitiesrev1.shtml
integers, and develop proficiency in
operations with integers http://staff.argyll.
epsb.ca/jreed/math7/
Demonstrating the addition, subtraction
strand1/1203.htm
and multiplication and combined
operations of integers

Term 1 2.4 Rational Numbers http://staff.argyll.


Week 4 2.5 Real Numbers and epsb.ca/jreed/math9/
Summary of the Use of strand1/1101.htm
Calculators http://www.freewebs.
• identify rational numbers and Explaining the relationship between p.40–46 Textbook com/weddell/Equiv%20
perform the four operations on fraction and rational numbers p. 46–52 Textbook Fractions%20Contents.
them Illustrating the four operations of html
E-book
• identify real numbers and rational numbers http://www.freewebs.
perform the four operations on Teacher’s Guide
Exploring the relationship between com/weddell/
them rational numbers and decimals, and the comparing%20fractions.
• use the functions of a existence of real numbers p.48 Class Activity 5 swf
calculator to carry out Introducing recurring decimals p.49 Class Activity 6 http://www.bbc.co.uk/
operations schools/gcsebitesize/
Representing rational numbers and real
numbers on the number line maths/number/
fractionsrev1.shtml
Demonstrating the use of calculators
Week Topic/Objectives Strategies & Learning Experiences Activities Resources Websites

Term 1 Chapter 3
Week 5 Approximation And
Estimation
3.1 – 3.2 Rounding Off Numbers
to Decimal Places and Significant
Figures
• round off numbers to a Recapitulating the idea of rounding off p.55–60 Textbook http://www.bbc.co.uk/
required number of decimal integers p. 60–63 Textbook schools/gcsebitesize/
places Extending it to rounding off a number to maths/number/
E-book roundestimaterev1.shtml
a specified number of decimal places
Teacher’s Guide
Introducing the concept of significant p.60 Class Activity 1
• round off numbers to a figures
required number of significant Demonstrating the techniques of
figures rounding off a number to a specified
number of significant figures, in
particular, showing whether a zero is a
place holder or has significant value

Term 1 3.3 Estimation


Week 6 • estimate quantities to an Estimating numbers and measures to p.64 Class Activity 2 p.64–72 Textbook http://www.calculator.
(half) appropriate degree of accuracy an appropriate degree of accuracy, p.66 Class Activity 3 E-book org/CalcHelp/index.htm
comparing the estimates and sharing
• estimate the results of estimation strategies Teacher’s Guide http://www.mathsisfun.
computation Using daily life examples to show the com/numbers/estimation-
• understand the effects of necessity of estimation tips-tricks.html
rounding errors in intermediate Comparing follow-through errors p.68 Class Activity 4
steps of calculations arising from intermediate values that are http://openlearn.open.
rounded to different degrees of accuracy ac.uk/mod/oucontent/
view.php?id=398404
Making estimates and checking the p.69 Class Activity 5
reasonableness of answers obtained from
calculators
Demonstrating estimation strategies

3
4
Week Topic/Objectives Strategies & Learning Experiences Activities Resources Websites

Term 1 Chapter 4
Week 6 Introduction To Algebra
(half) 4.1 The Use of Letters in
Algebra Generalising numerical expressions to p.76–80 Textbook http://www.mathsisfun.
• use letters to represent algebraic expressions E-book com/algebra/
numbers or variables Illustrating the algebraic notation using introduction.html
Teacher’s Guide
• interpret algebraic notations daily life examples

Term 1 4.2 Evaluation of Algebraic


Week 7 Expressions and Formulae,
4.3 Translation of Real-world
Situations into Algebraic
Expressions
• evaluate algebraic expressions Pointing out the difference between an p.81–85 Textbook http://staff.argyll.
and formulae algebraic expression and a formula p.86–89 Textbook epsb.ca/jreed/math8/
Illustrating with examples the evaluation strand2/2101.htm
E-book
of algebraic expressions and formulae by
substitution Teacher’s Guide
http://www.
Comparing and examining the difference p.83 Class Activity 1 mrbartonmaths.com/
between pairs of expressions such as 2n algebra1.htm#A1Page6
and 2 + n,
n2 and 2n , 2n2 and (2n)2 using
spreadsheets
Write algebraic expressions to express
• express real-world situations in mathematical relationships
algebraic terms Translating real-world situations into
algebraic expressions through a variety
of examples and practices
Week Topic/Objectives Strategies & Learning Experiences Activities Resources Websites

Term 1 Chapter 5
Week 8 Algebraic Manipulation
5.1 Like Terms and Unlike
Terms
5.2 Addition and Subtraction
of Linear Expressions
• differentiate between like Introducing the idea of like terms and p.95–99 Textbook http://lgfl.skoool.
terms and unlike terms unlike terms p.99–103 Textbook co.uk/viewdetails_ks3.
Using the AlgeDiscTM application in Alge p.96 Activity 1 aspx?id=554
E-book
ToolsTM to
– make sense of and to interpret linear Teacher’s Guide http://algetools.edumall.
expressions, http://www.staff sg/cos/o.x?c=/algetools/
– collect like terms vu.edu.au/mcaonline/ algetools&uid=
units/algebra/alglike. 150&ptid=415
• add and subtract linear Using the AlgeDiscTM application in p.99 Activity 2
html
algebraic expressions Alge ToolsTM to develop the concepts http://staff.argyll.
of addition and subtraction of linear epsb.ca/jreed/math9/
expressions, and the distributive law strand2/2103.htm
Finding addition and subtraction of
linear expressions, including removing
brackets

Term 1 5.3 Simplification of Linear


Week 9 Algebraic Expressions
5.4 Factorisation By Using
Common Factors
• simplify linear algebraic Using the AlgeDiscTM application in p.103 Class Activity 3 p.103–110 Textbook http://algetools.edumall.
expressions Alge ToolsTM to develop the concept of p. 111–112 Textbook sg/cos/o.x?c=/algetools/
multiplication of algebraic expressions, p. 106 Task 4 algetools&uid=150&ptid
and the distributive law E-book =415
Working in groups to select and justify Teacher’s Guide
pairs of equivalent expressions http://lgfl.skoool.co.uk/
content/keystage4/
Demonstrating the steps in simplifying maths/pc/lessons/uk_
linear expressions, including removing ks4_factors_1/interface.
brackets html?id=f1

5
6
Week Topic/Objectives Strategies & Learning Experiences Activities Resources Websites

• factorise a linear algebraic Introducing the concept of factors of


expression by using common algebraic expression by drawing an
factors analogy between numerical factors and
algebraic factors
Demonstrating the technique of factoring
by common factors

Term 1 Revision / Test


Week10

Term 2 Chapter 6
Week 1 Simple Equations In
One Variable
6.1 – 6.3 Simple Linear
Equations, Equations Involving
Brackets and Simple Fractional
Equations
• understand the concepts of Introducing the concept of equations p.117–121 Textbook http://algetools.edumall.
equations and the solution of TM sg/cos/o.x?c=/algetools/
Using the AlgeDisc application in Alge p.118 Class Activity 1 p.121–123 Textbook
an equation algetools&uid=150&ptid
ToolsTM to explore the concepts of linear p.123–125 Textbook =415
equations, and solve linear equations
E-book http://staff.argyll.
Reinforcing the solving equations skills epsb.ca/jreed/math8/
• solve linear equations in one with/without brackets Teacher’s Guide
strand2/2202.htm
variable Explaining the difference between a
• solve simple fractional linear equation and a fractional equation http://staff.argyll.
equations epsb.ca/jreed/math9/
Demonstrating how simple fractional
strand2/2103.htm
equations can be reduced to linear
equations and solving them
http://www.studygs.net/
equations.htm
Week Topic/Objectives Strategies & Learning Experiences Activities Resources Websites

Term 2 6.4 Forming Linear Equations


Week 2 to Solve Problems
(half) • formulate linear equations in Formulate linear equations to solve p.125 Class Activity 2 p.125–129 Textbook http://revisionworld.
one variable to solve problems problems E-book co.uk/gcse-revision/
(may draw model diagrams to help the maths/number-and-
Teacher’s Guide algebra/algebra/linear-
students)
equations

Term 2 Chapter 7
Week 2 Angles And Parallel
(half) Lines
7.1 Points, Lines and Planes
7.2 Angles
• describe a point, a line, a line Exploring the basic features of the p.133 Class Activity 1 p.133–136 Textbook http://nso.archive.
segment, a ray and a plane Geometer’s Sketchpad and using the (GSP) p.136–143 Textbook teachfind.com/
• construct lines, line segments software to construct points, lines, line downloader/
segments and angles E-book bfac59bcf989899711
and angles using the
Geometer’s Sketchpad Recognise the properties of a point, a Teacher’s Guide cf3948facef21d.exe
line, a line segment, a ray and a plane http://www.math.ubc.
• identify different types of angles ca/~cass/Euclid/ http://nso.archive.
• recognise the properties of Classifying angles according to their teachfind.com/
adjacent angles on a straight sizes downloader/
line, angles at a point and Using an intuitive approach to discover ee92a8e1a457409086
vertically opposite angles the properties of angles baacb95f70986b.zip

http://www.active-maths.
co.uk/whiteboard/meas_
angle1.html

http://lgfl.skoool.
co.uk/viewdetails_ks3.
aspx?id=415

7
8
Week Topic/Objectives Strategies & Learning Experiences Activities Resources Websites

Term 2 7.3 Parallel Lines &


Week 3 Transversals
7.4 Perpendicular Bisectors
and Angle Bisectors
• recognise the properties of Recalling the concept of parallel lines p.143–149 Textbook http://staff.argyll.
angles formed by parallel lines and the method of constructing parallel p.149–156 Textbook epsb.ca/jreed/math7/
and transversals lines strand3/3203.htm
E-book
• find unknown marked angles Introducing the terms corresponding
in a diagram using the above angles, alternate angles and interior Teacher’s Guide http://lgfl.skoool.
properties angles that a transversal makes with two co.uk/viewdetails_ks3.
lines (the two lines may not be parallel) aspx?id=550

Investigating the properties of angles p.145 Class Activity 2 http://www.


formed by two parallel lines and a (GSP) mathsrevision.net/gcse/
transversal using the GSP pages.php?page=17
Demonstrating the applications of the
• construct perpendicular above properties in finding angles http://staff.argyll.
bisectors and angle bisectors involving parallel lines epsb.ca/jreed/math7/
using compasses and ruler strand3/3203.htm
Showing students the proper use of a
• recognise the properties of pair of compasses
http://lgfl.skoool.
perpendicular bisectors and Showing the constructions of co.uk/viewdetails_ks3.
angle bisectors perpendicular bisector and angle bisector aspx?id=550
using a straight ruler and compasses
Using GSP to construct and study the p.151 Class Activity 3
properties of a perpendicular bisector of (GSP)
a line segment and of a bisector of an p.153 Class Activity 4
angle (GSP)
Week Topic/Objectives Strategies & Learning Experiences Activities Resources Websites

Term 2 Chapter 8
Week 4 Triangles And Polygons
8.1 Triangles
8.2 Quadrilaterals
• classify triangles based on Classifying triangles according to the p.161–170 Textbook http://lgfl.skoool.co.uk/
their sides and angles number of equal sides and the types of p.171–180 Textbook content/keystage3/maths/
• understand the general angles of the triangle and telling the pc/learningsteps/TRILC/
names of different types of triangles E-book launch.html
properties of sides and angles
of a triangle Recalling the construction of a triangle Teacher’s Guide
with 3 given sides using a ruler and a
pair of compasses
Investigating the general properties p.163 Class Activity 1
relating the sides and angles of a
triangle
Exploring the angle properties of p.166 Class Activity 2
triangles
Illustrating the applications of the above
angle properties of triangles to find
unknown angles in triangles
Stating the definitions and names of p.172 Class Activity 3
special quadrilaterals
Exploring a parallelogram to discover p.172 Class Activity 4
• identify different types of its properties and extending the (GSP)
quadrilaterals knowledge to study the properties of p.175 Class Activity 5
• recognise the properties of special quadrilaterals (GSP)
special quadrilaterals Illustrating the applications of the above
properties to find unknown angles in
quadrilaterals

9
10
Week Topic/Objectives Strategies & Learning Experiences Activities Resources Websites

Term 2 8.3 Polygons


Week 5 8.4 Construction of Triangles
and Quadrilaterals
• identify different types of Classifying types of polygons and telling p.180–190 Textbook http://lgfl.skoool.co.uk/
polygons the names of some common polygons p.190–195 Textbook content/keystage3/
• solve problems involving Obtaining the formula for angle sum of p.181 Class Activity 6 maths/pc/learningsteps/
E-book SYMLC/launch.html
the angle sum of interior and interior and exterior angles of a polygon p.184 Class Activity 7
exterior angles of any convex Teacher’s Guide
Illustrating the applications of the above (GSP) http://mathopenref.com/
polygons angle properties of polygons to find tocs/constructionstoc.
unknown angles in polygons en.wikipedia.org/wiki/ html
Recognising the line and rotational p.187 Class Activity 8 polygon
symmetry in some special quadrilaterals http://www.keymath. http://www.learnnext.
p.188 Class Activity 9
• recognise rotational and line and regular polygons com/DG/dynamic/ com/lesson/CBSE-VIII-
symmetry properties in some exterior_angle_sum. Maths-Construction-of-
special quadrilaterals and Allowing students to have hands-on html Quadrilaterals.htm
regular polygons practice in constructing triangles and
• construct triangles and quadrilaterals
quadrilaterals using a pair of Encouraging students to discuss on
compasses, ruler, set squares whether given conditions are enough for
and/or protractor when a construction
sufficient information is given

Term 2 Revision/Exam
Week
6 – 10
Week Topic/Objectives Strategies & Learning Experiences Activities Resources Websites

Term 3 Chapter 9 Chapters 9–16


Week 1 Ratio, Rate And Speed in Book 1B
9.1 Ratio
9.2 Rate
• compare two or more Recalling the definition of a ratio and http://www.bbc.co.uk/
quantities by ratio extending to ratios involving rational schools/gcsebitesize/
numbers p.2–10 Textbook maths/number/ratiosrev1.
• describe the relationship
between ratio and fraction Demonstrating the method of p.11–16 Textbook shtml

• divide a quantity in a given simplifying ratios using manipulations E-book


and calculators http://staff.argyll.
ratio Teacher’s Guide epsb.ca/jreed/math7/
• solve problems involving ratio Extending the concept to ratios of three strand1/1209.htm
quantities http://www
• solve problems involving rate mathleague.com/help/
in our daily life Using the concept of equivalent ratios to ratio/ratio.htm
find the ratio a : b : c, given the ratios
a : b and b : c
Discussing and explaining how ratios p.5 Class Activity 1
are used in everyday life
Using appropriate mathematical
language to describe a ratio relationship
by making connection between ratios
and fractions, e.g. the ratio of A to B is
1
2 : 3 is the same as “A is of B”
2
Consolidating the concept by applying
it to solve problems involving ratios in
daily life
Understand the concepts of rate and
average rate
Discussing examples of rate such as p.12 Class Activity 2
currency exchange rates, interest rates,
tax rates, rate of rotation and speed

Demonstrating the application in problems


involving rate in our daily life

11
12
Week Topic/Objectives Strategies & Learning Experiences Activities Resources Websites

Term 3 9.3 Speed


Week 2 • recognise the relationships Stating the relationship between speed, p.17–22 Textbook http://lgfl.skoool.
(half) between distance, time and time and distance E-book co.uk/viewdetails_ks3.
speed Finding out and comparing the speeds p.18 Class Activity 3 aspx?id=485
Teacher’s Guide
• understand the concepts of of bicycles, cars, trains, aeroplanes and
constant speed and average spaceships, and their respective units to
speed have a sense of their magnitude
• express speed in different Illustrating the difference between
units and convert it from one uniform speed and average speed, and
unit to another explaining why average speed is not the
• solve problems involving average of speeds using examples
speed Demonstrating the conversion between
the speed units km/h and m/s
Demonstrating the applications of speeds

Term 3 Chapter 10 Percentage http://www.bbc.co.uk/


Week 10.1 – 10.4 Simple Percentage schools/gcsebitesize/
2– Problems, Reverse Percentages, maths/number/
Percentage Increase and percentagesrev1.shtml
4(half) Decrease, Discount and GST
• express one quantity as a Explaining the meaning and usage of p.27–33 Textbook http://www.bbc.co.uk/
percentage of another percentage schools/gcsebitesize/
p.33–35 Textbook
• compare two quantities by maths/number/
Discussing the meaning of percentage p.27 Class Activity 1 p.35– 41 Textbook
percentage percentagesrev2.shtml
in some daily life examples and making
connections between percentages and p.41–47 Textbook
• recognise percentages greater http://www.bgfl.org/bgfl/
than 100% fractions/decimals E-book
custom/resources_ftp/
• solve problems involving Demonstrating how to express one Teacher’s Guide client_ftp/ks2/maths/
reverse percentages quantity as a percentage of another percentages/index.htm
and to compare two quantities using
• calculate percentage increase percentage http://staff.argyll.
and decrease
epsb.ca/jreed/math8/
• solve problems involving strand1/1104.htm
discounts and GST
Week Topic/Objectives Strategies & Learning Experiences Activities Resources Websites

Discussing some misconceptions such as p.31 Class Activity 2


“If A is 5% more than B, then B is 5%
less than A”
Reinforcing the concept by solving some
problems involving reverse percentage
and algebraic techniques
Highlighting percentage increase and
decrease in some daily life situations
Demonstrating the solving of problems
involving percentage increase and
decrease (including concept of p. 40 Qns. 11 & 12,
percentage points) – show examples p. 49-50 Qns.10-12
of percentages from newspapers and
magazines
Illustrating the calculation of discounts
in transactions
Explaining briefly the idea of taxation
and GST
Examine bills and receipts to find p.43 Class Activity 3
examples of discount, service charge and
GST, and to check the calculated values
Illustrating the solving of problems
involving discounts and GST, based on
real-world contexts

13
14
Week Topic/Objectives Strategies & Learning Experiences Activities Resources Websites

Term 3 Chapter 11 Number


Week Patterns
http://www.bbc.co.uk/
4 (half) 11.1 Number Patterns and schools/gcsebitesize/
–5 Sequences maths/algebra/
11.2 General Term of a sequencesrev1.shtml
Sequence
• recognise number patterns and Introducing and recognising number p.52 Class Activity 1 p.52–57 Textbook http://www.bbc.co.uk/
sequences patterns schools/gcsebitesize/
p.58–63 Textbook
• find the terms of a sequence maths/algebra/
Introducing the idea of the general term E-book sequencesrev2.shtml
• find the formula for the of a sequence as an outcome of finding
the rule of a sequence Teacher’s Guide
general term of a sequence http://www.mathsisfun.
• solve problems involving Explore number patterns including those p.61 Class Activity 2 com/algebra/sequences-
number patterns and sequences from real-world situations and write series.html
algebraic expressions to represent the
patterns
Modelling some problems

Term 3 Chapter 12
Week 6 Coordinates And Linear http://www.shodor.org/
Graphs interactivate/activities/
LinearFunctMachine/
12.1 – 12.2 Cartesian
Coordinate System and Idea of
http://nso.archive.
a Function
teachfind.com/download
• construct the Cartesian Delivering the idea of a Cartesian p.68–73 Textbook
er/332a4b3b094f2317f83
coordinate system in two coordinate system as a necessity to p.74–81 Textbook fbb451a3a4966.swf
dimensions and state the locate positions on a plane
coordinates of points on it E-book
Specifying and plotting points on a p.71 Class Activity 1 http://lgfl.skoool.
• recognise the idea of a function two-dimensional Cartesian plane in a Teacher’s Guide
co.uk/viewdetails_ks3.
• plot a graph of a set of ordered battleship game. aspx?id=414
pairs as a representation of Engaging students with ample
a relationship between two practices and activities to plot points http://staff.argyll.
variables on the Cartesian plane and to state the epsb.ca/jreed/math7/
coordinates of given points strand2/2102.htm
Week Topic/Objectives Strategies & Learning Experiences Activities Resources Websites

Explaining the concept of a function by p.74 Class Activity 2 http://nso.archive.


using a function machine to generate teachfind.com/
input and output values to the concept of downloader/
function as “only one output for every c022ffe9f431e64d22aae
input” and represent a function using 91cb41d9b9f.swf
verbal, tabular, graphical and algebraic
forms http://www.webmath.
Practising the different ways a function com/gline.html
can be represented
Associating ordered pairs with points
on a coordinate plane to represent a
relationship between two variables

Term 3 12.3 – 12.4 Linear Functions


Week 7 And their Graphs, and
Gradients of Linear Graphs
• recognise linear functions and Defining a linear function y of x p.81–86 Textbook http://staff.argyll.
draw graphs of linear functions Demonstrating the steps of drawing a p.87–96 Textbook epsb.ca/jreed/math7/
• find the gradient of a linear graph strand2/2102.htm
E-book
linear graph as the ratio of Illustrating linear relationships in real-
the vertical change to the world contexts by drawing linear graphs Teacher’s Guide http://nlvm.usu.edu/
horizontal change Developing the idea of gradient of a en/nav/frames_
straight line as the ratio of the vertical asid_109_g_3_t_1.
change to the horizontal change html?open=activities
Demonstrating the way of finding the
http://www.shodor.org/
gradient of a straight line in various
interactivate/activities/
situations
OrderedSimplePlot/
Interpreting the meanings of positive
gradient, negative gradient, zero gradient http://www.shodor.org/
and undefined gradient interactivate/activities/
Studying how the graph of y = ax + b p.91 Class Activity 3 SlopeSlider/
changes when either a or b varies (GSP)
Using a linear function to represent
p.93 Class Activity 4
the relationship between two variables
(distance and time when the speed
is constant), show the relationship
graphically and identify that the rate
of change is the gradient of the linear
graph

15
16
Week Topic/Objectives Strategies & Learning Experiences Activities Resources Websites

Term 3 Chapter 13
Week 8 Simple Inequalities
13.1 – 13.2 Solving Simple
Inequalities and Applications of
Simple Inequalities
• understand the idea of Representing a relationship involving a p.102–106 Textbook http://www.coolmath.
inequality given variable using an inequality p.106–109 Textbook com/algebra/07-solving-
• solve simple linear inequalities Exploring the multiplication property of p.102 Class Activity 1 inequalities/index.html
E-book
in the form ax . b, ax , b, inequalities
as > b and ax  b where a Teacher’s Guide http://www.bbc.co.uk/
Demonstrating the way of solving a schools/gcsebitesize/
and b are integers linear inequality and of representing the maths/algebra/
• compare and contrast the solution on a number line inequalitiesrev1.shtml
solutions of inequalities with Illustrating the difference in the
different inequality signs such representation of solutions of inequalities
as x . 3 and x > 3 with different inequality signs

• formulate simple problems Formulating inequalities, based on


that can be represented by real-world contexts, using the problem-
inequalities solving strategy

• solve simple word problems Illustrating the applications of simple


involving inequalities inequalities in word problems

Term 3 Chapter 14
Week 9 Perimeters And Areas Of
http://www.bbc.co.uk/
Plane Figures
schools/gcsebitesize/
14.1 Area of Parallelogram maths/geometry/
14.2 Area of a Trapezium areaandperimeterrev1.
• find the area of a Recapitulating the formulae of areas p.113–120 Textbook shtml
parallelogram and perimeters of squares, rectangles, p.121–126 Textbook
• find the area of a trapezium triangles and circles http://illuminations.
E-book nctm.org/ActivityDetail.
Solving some reverse problems
involving the above formulae to Teacher’s Guide aspx?ID=106
consolidate students’ understanding
http://illuminations.
nctm.org/ActivityDetail.
aspx?ID=21
Week Topic/Objectives Strategies & Learning Experiences Activities Resources Websites

Making connection between the area of p.115 Class Activity 1


a parallelogram and that of a rectangle
to derive the formula of the area of a
parallelogram
Identifying the height corresponding p.116 Class Activity 2
to any given side of a triangle or a
parallelogram that is taken as the base
Solving problems involving area of a
parallelogram
Making connection between the area of p.121 Class Activity 3
a trapezium and that of a parallelogram
to derive the formula of the area of a
trapezium
Identifying the height corresponding to p.122 Class Activity 4
the parallel sides of a trapezium
Illustrating the application of the
formula for the area of a trapezium

Term 3 14.3 Perimeter and Area of


Week Composite Plane Figures
10 • solve problems involving Extending the mensuration of basic p.126–130 Textbook http://mathworld.
perimeters and areas of figures to find the perimeters and E-book wolfram.com/Trapezium.
composite plane figures areas of composite figures html
Teacher’s Guide
http://nlvm.usu.
edu/en/nav/frames_
asid_282_g_3_t_3.
html?open=activities

Term 3 Revision/Test
Week
10

17
18
Week Topic/Objectives Strategies & Learning Experiences Activities Resources Websites

Term 4 Chapter 15 http://lgfl.skoool.


Week 1 Volume And Surface co.uk/viewdetails_ks3.
Areas Of Solids aspx?id=551
15.1 Views and Nets of 3D http://www.bbc.co.uk/
Shapes schools/ks3bitesize/
15.2 Volume and Total Surface maths/measures/volume/
Area of Prisms revise3.shtml
• visualise and draw sketches Making studies of 3D shapes p.135 Class Activity 1 p.134–140 Textbook http://en.wikipedia.
of 3D shapes from different Visualising and drawing sketches of 3D p.140–150 Textbook org/wiki/
views shapes from different views Prism_%28geometry%29
E-book
• visualise and draw nets of Visualising and drawing the nets of p.137 Class Activity 2 Teacher’s Guide http://www.shodor.org/
cubes, cuboids, prisms and cubes, cuboids, prisms and cylinders interactivate/activities/
cylinders for calculation of
Deriving the formulae of volume and SurfaceAreaAndVolume/
surface areas http://www.mathsnet.
surface area of prisms using intuitive net/geometry/solid/ http://staff.argyll.
• find the volume and total approach
surface area of prisms nets.html epsb.ca/jreed/math8/
Demonstrating the applications of the strand3/3206.htm
above formulae http://staff.argyll.
epsb.ca/jreed/math8/
strand3/3207.htm

Term 4 15.3 – 15.4 Volumes and


Week 2 Surface Areas of Cylinders and
Composite Solids
• find the volume and surface Deriving the formulae for the volume p.150–156 Textbook http://www.mathsisfun.
area of cylinders and surface area of a cylinder through p.156–162 Textbook com/geometry/cylinder.
• convert between cm2 and m2, intuitive reasoning html
E-book
and between cm3 and m3 Demonstrating the applications of the
above formulae Teacher’s Guide http://staff.argyll.
• solve problems involving epsb.ca/jreed/math8/
volume and surface area of Illustrating with examples the conversion strand3/3206.htm
composite solids between cm2 and m2, and between cm3
and m3 http://staff.argyll.
Developing students’ skills in finding the epsb.ca/jreed/math8/
volume and surface area of composite strand3/3207.htm
solids of prisms and cylinders
Week Topic/Objectives Strategies & Learning Experiences Activities Resources Websites

Term 4 Chapter 16
Week 3 Data Handling
16.1 – 16.3 Collection of
Data , Organisation of Data,
Pictograms and Bar Graphs
• recognise different methods of Discussing the various methods of p.167–170 Textbook http://www.bbc.co.uk/
collecting data collecting data p.171–176 Textbook schools/gcsebitesize/
• organise raw data into Selecting an appropriate data collection p.169 Class Activity 1 maths/statistics/
p.176–185 Textbook collectingdatarev1.shtml
frequency tables method to suit the needs
E-book
• interpret a frequency table Grouping students to collect data and Teacher’s Guide http://www.
• construct, analyse and interpret describe their procedures primaryresources.co.uk/
pictograms and bar graphs Illustrating the rationale of organising maths/mathsF1.htm#bar
data and the method of constructing a
frequency table
Drawing simple inferences from a
frequency table
Revising the construction of pictograms
and bar graphs
Illustrating with examples the analysing p.182 Class Activity 2
and interpreting of these graphs

19
20
Week Topic/Objectives Strategies & Learning Experiences Activities Resources Websites

Term 4 16.4 Line Graphs and Pie


Week 4 Charts
16.5 Use & Misuse of
Statistical Graphs
• construct, analyse and interpret Guiding students to construct the line p.185–194 Textbook http://nso.archive.
line graphs and pie charts graphs and pie charts, and to observe the teachfind.com/download
information presented in these graphs p.195–207 Textbook er/4a338c9b65580e0513
• draw simple inferences from
statistical diagrams Developing students’ ability in analysing E-book cc86a429f55a00.swf

• describe the purposes and and interpreting graphs through a variety


of statistical situations Teacher’s Guide http://nlvm.usu.
appropriateness of use of the edu/en/nav/frames_
different forms of statistical Asking students to draw comparisons p.190 Class Activity 3 asid_200_g_3_t_5.
representations between the advantages and html?open=instructions
• explain why a given disadvantages of the different statistical
statistical diagram can lead to graphs through activities http://nlvm.usu.
misinterpretation of data Organising students to work p.195 Class Activity 4 edu/en/nav/frames_
collaboratively on a survey to asid_323_g_3_t_5.html
* collect and classify data,
* present data using appropriate http://www.bbc.co.uk/
representations including the use of schools/gcsebitesize/
software, maths/statistics/
* analyse data representingdata1rev3.
* use the data to make decisions shtml
Comparing various statistical p.195 Class Activity 5
representations and justifying why a http://www.bbc.co.uk/
particular representation is more suitable schools/gcsebitesize/
than others for a given situation maths/statistics/
representingdata3hirev3.
Guiding students to look out for certain shtml
features of the various statistical
representations which can give http://www.
misleading information primaryresources.co.uk/
Illustrating correct comparison of p.204 Class Activity 6 maths/mathsF1.htm#bar
statistical diagrams and the making of
inferences from them http://math.youngzones.
org/stat_graph.html
Term 4 Revision / Exam
Week
5–7
Notes On Teaching

Chapter 1  Factors and Multiples

Suggested Approach
Students have learnt factors and multiples in their primary schools. It may be better to use some examples to recall these
concepts before introducing prime numbers.

The index notation in this chapter is confined to positive integral indices only. It is not necessary to introduce the index laws.

Throughout this chapter, it is advisable to use simple numbers to illustrate the concepts and reinforce them with sufficient
hands-on experience.

We simply introduce the definition of prime numbers in this chapter. For abler students, they are encouraged to find out more
facts and properties of prime numbers from the Internet.

1.1 Primes, Prime Factorisation and Index Notation


After recalling factors and multiples, we classify whole numbers based on their number of factors. Students should
discuss why 0 and 1 are not prime numbers nor composite numbers.

As 0 = 0 × 1 = 0 × 2 = 0 × 3 = …, the numbers 1, 2, 3, … are all factors of 0. Thus, 0 has more than two factors and
hence it is not a prime number. Since 0 is less than 1 and it cannot be written as a product of two whole numbers in
which neither of them is 0, it is not a composite number.

The number 1 has only one factor, this is itself. Therefore, 1 is not a prime number. Since 1 is not greater than 1 and it
cannot be written as a product of two whole numbers other than 1, it is not a composite number.

Teachers may discuss the following questions with their students after defining prime numbers.
• Is there a finite number of prime numbers?
• How can we determine whether a number is a prime number?

If p1, p2, p3, …, pn are the first n prime numbers, then p1 p2 p3 …pn + 1 must be a prime number. This shows that there
is an infinite number of primes. If n is a composite number, then it should have a prime factor p that is less than the
square root of n. Hence to show that a number n is a prime number, it is sufficient to show that n is not divisible by any
prime number less than the square root of n.

Students should be aware that a factor tree may span in different ways, but each way will give the same prime factors.
Since 1 is not a prime number, we do not write the factor 1 in prime factorisation. Teachers may ask students to write
down some possible factors of a number based on its prime factorisation.

1.2 Highest Common Factor (HCF)


Students should understand the difference and the relationship between a common factor and the highest common factor
of two numbers. They may find the HCF of two or three numbers either by prime factorisation or short division method.
Application problems may be used to arouse students’ interest and help them relate the idea of highest common factors
to real-life situations.

21
1.3 Lowest Common Multiple (LCM)
Students should understand that every two numbers have an infinite number of common multiples. Some application
problems involving LCM should be introduced.

For abler students, they could explore the relationship that for any two numbers a and b, a × b = their HCF × their LCM.

1.4 Square Roots and Cube Roots


The idea of square, square root, cube and cube roots can be introduced through some simple and concrete examples. It
should be emphasized that n denotes the positive square root of the number n. The negative square root of the number
n is denoted by n which will be discussed in Chapter 2.

Notes On Teaching
22
Chapter 2  Real Numbers

Suggested Approach
Students have learnt whole numbers, fractions and their four operations at the primary level. In this chapter, the concept of
numbers is extended from whole numbers to real numbers. We first of all introduce the idea of negative numbers through some
everyday examples. The number line is used as a graphical representation of numbers. The operations of integers are explored
using the algebra discs in AlgeTools.

Rational numbers are introduced as a natural consequence of division of integers. Students should recognise that a rational
number can be expressed as a terminating decimal or a repeating decimal.

Teachers are not required to introduce the term irrational number. It is enough for students to be aware that real numbers
correspond to the points on the number line and some of the real numbers are not rational numbers.

2.1 Idea of Negative Numbers and the Number Line


An intuitive idea of negative numbers arises from situations such as time zones and the temperature scale below zero.
Students should be able to classify integers into three groups, namely positive integers, zero and negative integers.

Students should learn how to draw a number line. They should build up the concept of inequality and ordering using
the number line. The idea of -8 < -3 can be illustrated by the fact that -8 °C is colder than -3 °C.

2.2 Addition and Subtraction of Integers


Class activities using the AlgeDisc applications in AlgeTools are employed to explore the addition and subtraction of
integers. Students should use the concept of zero pairs and the negative of a number can be obtained by flipping the
discs.

After exploration, students should be able to conclude the rules of addition and subtraction. Further examples should be
used to reinforce the concept.

2.3 Multiplication, Division and Combined Operations of Integers


Multiplication can be explored using the grouping and ungrouping of algebra discs. Division of integers is considered
as the reverse process of multiplications of integers. Students should be reminded that division by zero is undefined.

It is crucial that students know the order of operations of integers in an expression involving brackets and the four
operations. They should be reminded not to jumping steps before mastering the techniques.

2.4 Rational Numbers


Rational numbers are introduced as an extension of integers. Students should learn to master the skills of operations
of rational numbers as an extension of operations of integers and fractions. In this section, it is advisable that students
carry out the operations by hand. However, tedious manipulations should be avoided.

2.5 Real Numbers and Summary of the Use of Calculators


Students should understand that decimals can be classified as terminating decimals, recurring decimals and ‘non-terminating
and non-recurring’ decimals. All decimals are real numbers and can be represented on the number line. Those terminating
decimals and repeating decimals are rational numbers.

Different calculators may use different key sequences to manipulate an expression. Students should be reminded to
familiarise themselves with their own calculators.

23
Chapter 3  Approximation and Estimation

Suggested Approach
Students should recognise that all measurements are approximations owing to the limitation of measuring instruments. Further,
we sometimes use approximated numbers because they are easier to remember. A number line can be used to illustrate the idea
of ‘4/5 rounding’. To reinforce the idea of significant figures, students should be encouraged to use them in various application
problems.

Estimation techniques can be explored to ask students to work in groups to estimate quantities (numbers and measures) to an
appropriate degree of accuracy in a variety of contexts, compare the estimates and share the estimation strategies.

Students should compare follow-through errors arising from intermediate values that are rounded in different degrees of accuracy
in some calculations. They should also make estimates and check that answers obtained from calculators.

3.1 Rounding Off Numbers to Decimal Places


The idea of rounding off a number to the nearest unit, 10, 100 and 1000 is revised. Then it is extended to rounding off
a number to a specified decimal place. Note that a number should be rounded off from its original value but not from
one of its rounded values. For instance, if x = 3.149, then
x = 3.15 (correct to 2 d.p.)
and x = 3.1 (correct to 1 d.p.).
It is wrong to round off 3.149 to 3.15, and then conclude that x = 3.2 (correct to 1 d.p.).

3.2 Rounding Off Numbers to Significant Figures


The most significant figure of a number is the first non-zero digit, reading from left to right. For a number like 0.00307,
some students may have the wrong concept that it has 5 significant figures (counting those two zeros after the decimal
points to be significant), while some students may wrongly interpret that it has only 2 significant figures (not knowing
that the zero after the most significant digit 3 is significant).

Another point that students may be confuse about is that the trailing zeros of an integer may or may not be significant.
For instance, the number 34 500 000 may have 3 to 7 significant figures.

3.3 Estimations
In estimating a sum or a measurement, different estimation strategies may yield different estimates. Students should note
that if each number in an expression is rounded off to 2 significant figures, the estimate of the expression may not be
correct to 2 significant figures.

In using the benchmark method to estimate a measurement, the benchmark should be carefully chosen such that its value
is known and it is easy to estimate the measurement with the benchmark.

If the answer is required to be correct to 3 significant figures, students should be reminded to round off the intermediate
values in a calculation correct to at least 4 significant figures.

Notes On Teaching
24
Chapter 4  Introduction to Algebra

Suggested Approach
The smooth transition from arithmetic expressions to algebraic expressions can be achieved through generalisation of arithmetic
expressions for some simple daily life scenarios. The basic notations in Algebra should be introduced step by step with examples.
It is advisable to ask students to compare and distinguish between arithmetic language and algebraic language.

Some students may confuse between algebraic expressions and formulae. It is better to teach algebraic expressions and its
substitution before formulae. In order to build the confidence of students and clarify their concept, the formulae used should
not be too complicated and the values of variables should vary from positive integers to rational numbers gradually.

Some students may have difficulty in translating a real-world situation into an algebraic expression. The use of model diagram
would help them formulate expressions and formulae.

4.1 The Use of Letters in Algebra


By exploring some daily-life examples, students should find that the use of letters is an easy way to express a generalised
arithmetic expression. Sufficient practice in translating word phrases into algebraic phrases should be given. Students
should distinguish between variables and numbers, and develop the habit of writing proper algebraic expressions.

4.2 Evaluation of Algebraic Expressions and Formulae


It is advisable to guide students to discuss the difference between algebraic expression and formula. Students should
discover that when the values of all but one variable in a formula are known, the value of the remaining variable can be
calculated by substitution. Some simple real-world examples can stimulate students’ interest in learning this topic.

Students may use a spreadsheet to explore the concept of variables and evaluate algebraic expressions. They can also
use a spreadsheet to compare and examine the difference between pairs of expressions such as 2n and 2 + n, n2 and 2n,
2n2 and (2n)2.

4.3 Translation of Real-world Situations into Algebraic Expressions


Ample examples involving one quantity is more than or less than another quantity by a certain value, and one quantity is
n times or a certain fraction of another quantity should be demonstrated with model diagrams. It takes time for students
to master the skill of formulating algebraic expressions inyo word problems.

25
Chapter 5  Algebraic Manipulation

Suggested Approach
Algebraic manipulation is crucial for students in learning mathematics and science. Careful elaboration of the concept is necessary
in order to build a strong foundation. Students should not skip steps in presenting solutions.

An algebraic expression can be considered as a machine, with the terms as its parts. Analogously, like terms and unlike terms
are like parts and unlike parts of the machine respectively. We may use an activity to ask students to identify coefficients of
given terms, like terms and unlike terms. The Algedisc application in AlgeTools can be used to explore the collection of like
terms and simplification of algebraic expressions.

Furthermore, students can work in groups to select and justify pairs of equivalent expressions. This will reinforce their technique
of simplification of algebraic expressions.

Numerical expressions can be used to introduce distributive property. Teachers may also use the geometrical interpretation as
shown below.

x + y x y

a a +a

Teachers may present various cases and help students discuss how to handle brackets in algebraic expressions.

Teachers may draw the analogy between prime factorisation of a whole number and the factorisation of an algebraic expression.

5.1 Like Terms and Unlike Terms


It should be emphasized that the sign of a term is attached to the coefficient and not the variables. Terms with the same
variables may not be like terms. For example, (x and x2) and (a2b and ab2) are two pairs of unlike terms.

Students may use algebra discs to make sense of and interpret linear expressions. They can use the discs to learn to
collect like terms.

5.2 Addition and Subtraction of Linear Expressions


Students may need to revise the addition and subtraction of integers and fractions. They should be careful when removing
brackets. The process of flipping algebra discs when dealing with the negative of an expression may help students to
understand and remember – (a + b – c) = – a – b + c.

5.3 Simplification of Linear Expressions


This section is confined to the expansion of linear algebraic expressions. Students should learn the skill of both distribution
from the left and distribution from the right.

5.4 Factorisation By Using Common Factors


Factorisation by drawing the common factor may be considered as the reverse process of expansion. After sufficient
practice, students should be able to locate the appropriate factor. However, they should be reminded to factorise an
expression completely. Some students may factorise 4ax + 6ay as a(4x + 6y), instead of the correct answer 2a(2x + 3y).

Students should develop the habit of expanding the factorisation result and check if the original expression can be
obtained. Instead of factoring ax + ay + a as a(x + y + 1), some students drop the 1 and get the wrong answer a(x + y).

Notes On Teaching
26
Chapter 6  Simple Equations in One Variable

Suggested Approach
From the mathematical sentence + 3 = 8, students can see that it is more convenient to replace the circle by a letter such as x
or y. Teachers can then introduce the concept of equations and solutions (i.e. roots). The process of solving linear equations can
be explored using the virtual balance in AlgeTools. When students understand the idea, they can apply the rules of transposing
terms to solve linear equations in one variable. Students should be encouraged to check the solutions in solving equations. Linear
equations should be introduced from simple ones (2-step solution) to more difficult ones (involving brackets and fractions).

Fractional equations are equations with variables in the denominators of the equation. Students are only required to solve simple
fractional equations that can be reduced to linear equations.

Some students may experience difficulty in solving word problems. They may try to use the AlgeBar applications in AlgeTools
to formulate linear equations to solve problems. They can also draw models to help them formulate the equation.

6.1 Simple Linear Equations in One Variable


Students should use the balancing concept to solve linear equations in one variable. In this section, they should not jump
steps in their solutions. Some students may confuse algebraic expressions with equations.

6.2 Equations Involving Brackets


Teachers can recall the distributive law when demonstrating the skill of solving linear equations involving brackets. A
common error of students is expanding –3(x – 4) as –3x – 12. For linear equations with numbers in the denominators,
it is better to multiply the whole equation by the LCM of the denominators first.

6.3 Simple Fractional Equations


The solution of a fractional equation cannot be a value that makes a denominator of the original equation zero. Students
should check the solutions obtained against the original equation.

6.4 Forming Linear Equations to Solve Problems


In solving word problems, diagrams and models will help students understand the problems. Students should be reminded
of the general steps. In particular, they should state the meaning of the unknown at the beginning. In the first few
problems, students may need more structured steps to help them formulate the problems.

27
Chapter 7  Angles and Parallel Lines

Suggested Approach
Teachers may motivate students by showing them angles, parallel lines and geometric shapes in our daily life. We will use
the dynamic geometry software package, The Geometer’s Sketchpad, to explore the properties of angles and parallel lines.
At this level, we will take an intuitive approach for geometry, no serious proofs are necessary. However, students should be
encouraged to attempt some elementary reasoning. They should write down the reasons in their working steps as this will help
them appreciate and remember the properties involved.

Students have learnt to use a protractor to draw and measure angles at the primary level. They will learn basic geometrical
constructions using various tools such as compasses, straightedges and set squares in this chapter. Teachers may tell them the
story of construction using primitive tools in ancient Greek and lead them to appreciate this art.

Students should use Sketchpad to construct and study the properties of perpendicular bisector of a line segment and bisector
of an angle.

7.1 Points, Lines and Planes


The terms point, line and plane are undefined terms. Students should be encouraged to use their intuitive ideas to describe
these terms. They should learn the notations of point, line, ray, line segment and plane. This section may be students’ first
encounter with Sketchpad. They may refer to the navigation guide of Sketchpad at the back of this book, to familiarise
themselves with its commands and operations.

7.2 Angles
An angle can be considered as a rotation. Teachers may recall the construction and measurement of an angle using a
protractor before introducing the terms acute angle, obtuse angle, etc. Students should be familiar with the different
notations of angles. The properties of angles can be explored using Sketchpad. Students should understand the definition
of vertically opposite angles and differentiate them from opposite angles. Sufficient examples and exercises can help
students develop their ability to apply the properties of angles.

7.3 Parallel Lines and Transversals


Parallel lines can be drawn easily with set squares. It should be noted that two lines cut by a transversal does not
necessarily mean the two lines are parallel. Students should identify pairs of corresponding angles, alternate angles and
interior angles on the same side when two lines are cut by a transversal.

For abler students, they can be challenged to prove the properties of alternate angles and interior angles formed by two
parallel lines and a transversal, given that their corresponding angles are equal.

7.4 Perpendicular Bisectors and Angle Bisectors


The idea of a perpendicular bisector and an angle bisector can be introduced through folding paper. Students may be
challenged to devise their own methods of drawing these bisectors before the formal ways of construction are taught.

Students should explore the properties of perpendicular bisector and angle bisector. Abler students may be asked to find
the incentre and the circumcentre of a triangle by construction.

Notes On Teaching
28
Chapter 8  Triangles And Polygons

Suggested Approach
Teachers may show that triangle is a basic shape of construction and buildings to motivate students’ interest in studying this
chapter. The general properties of triangles and polygons can be explored using Sketchpad and some other class activities. For
instance, students can form triangles with different lengths to discover the sum of two sides is greater than the third side and
the longest side is opposite the biggest angle.

Students can also explore the properties of a parallelogram using Sketchpad, and then extend the knowledge to study the
properties of rhombus, rectangle and square.

The formula for angle sum of interior angles and exterior angles of polygons can be obtained through inductive reasoning.
Students should learn to reason and justify whether a geometrical statement is true or not. For instance, does a regular polygon
with an exterior angle of 75° exist?

For the construction of triangles and quadrilaterals, students can use any tools. They may be asked if they will get a unique
figure of a triangle or a quadrilateral.

Students should recognise symmetric properties (rotational and line symmetry) in some special quadrilaterals and regular
polygons. Teachers may discuss with them whether some signboards and company logos have symmetry.

8.1 Triangles
Students should be able to classify triangles by the number of equal sides or types of its angles. Through construction,
they will learn triangle inequality, properties of isosceles triangle and equilateral triangle.

Students have learnt the angle sum of triangle is 180° at the primary level. They should be encouraged to draw the
exterior angles of a triangle in various cases to make sure that they can understand and identify exterior angles.

8.2 Quadrilaterals
Students should be familiar with the names of special quadrilaterals such as trapezium, parallelogram, rhombus, rectangles
and square. However, it is better to state their definitions again to consolidate students’ concept. Properties of these
special quadrilaterals can be explored using Sketchpad. Sufficient examples should follow to illustrate the application
of these properties.

8.3 Polygons
The sum of interior angles of a polygon can be found by dividing the polygon into triangles Alternatively, we can first
find the sum of exterior angles of a polygon, which is 360°. As the sum of interior angles and exterior angles is 180n°,
the sum of the interior angles is 180n° – 360° = (n – 2) × 180°. Students should note that the formula for the sum of
exterior angles is only valid for convex polygons.

8.4 Symmetry of Polygons


Students should understand the definitions of line symmetry and rotational symmetry. It should be noted that there is no
rotational symmetry of order 1.

They are encouraged to point out some shapes and objects which possess symmetry in their daily lives. The symmetry
properties of special quadrilaterals and regular polygons are discussed.

8.5 Construction of Triangles and Quadrilaterals


Students are encouraged to draw a rough sketch of the required figure before constructing it. They should write down
their steps of construction. They should learn that the shape of a quadrilateral is not fixed if only the lengths of its four
sides are given.

29
FULLY
WORKED
SOLUTIONS
1 Factors and Multiples

Class Activity 1
Objective: To classify whole numbers based on their number of factors.

Tasks
1. Do the following task with a partner. Consider the following whole numbers, list their factors in the table below.

Number Factors Number Factors


2 1, 2 12 1, 2, 3, 4, 6, 12
3 1, 3 13 1, 13
4 1, 2, 4 14 1, 2, 7, 14
5 1, 5 15 1, 3, 5, 15
6 1, 2, 3, 6 16 1, 2, 4, 8, 16
7 1, 7 17 1, 17
8 1, 2, 4, 8 18 1, 2, 3, 6, 9, 18
9 1, 3, 9 19 1, 19
10 1, 2, 5, 10 20 1, 2, 4, 5, 10, 20
11 1, 11 21 1, 3, 7, 21

2. Classify each of the above numbers according to the number of factors they have. You may want to use the table below
to help you organize.

Number of factors Numbers


1 None
2 2, 3, 5, 7, 11, 13, 17, 19
3 4, 9
4 6, 8, 10, 14, 15, 21
5 16
More than or equal to 6 12, 18, 20

3. Observe those numbers that have only two factors. List them out. What patterns do your observe about the factors of
those numbers? The numbers you have listed out have a special name. Do you know what they are called?

The numbers are 2, 3, 5, 7, 11, 13, 17 and 19. These numbers only have two factors, 1 and itself. They are called prime numbers.

4. Let us examine the numbers 0 and 1. How many factors do they have?

0 has infinitely many factors. 1 has only one factor, itself.

33
Try It! 5. Find the prime factorisation of 702 and write the answer
in index notation.
Section 1.1
1. Find the factors of 105. Solution
702
Solution
105 = 1 × 105 2 351
= 3 × 35
= 5 × 21 3 117
= 7 × 15
The factors of 105 are 1, 3, 5, 7, 15, 21, 35 and 105. 3 39

3 13
2. Determine whether the following are prime numbers or
composite numbers. \ 702 = 2 × 3 × 3 × 3 × 13
(a) 127 (b) 473 = 2 × 33 × 13
Solution
(a) 127 is not divisible by
Section 1.2
1, 2, 3, 5, 7, 11, ….
\  127 is a prime number. 6. Find the HCF of 252 and 360.
(b) 473 = 11 × 43
\  473 is a composite number. Solution
252 = 22 × 32 × 7
360 = 23 × 32 × 5
3. Find the prime factorisation of 585 using a factor tree \  HCF = 22 × 32
and compare it with those of your classmates. = 36

Solution
585 7. Find the HCF of 154, 330 and 396.

Solution
3 195 154 = 2 × 7 × 11
330 = 2 × 3 × 5 × 11
5 39 396 = 22 × 32 × 11
\  HCF = 2 × 11
3 13 = 22
\ 585 = 3 × 3 × 5 × 13
8. A rectangular piece of paper 35 cm by 28 cm is cut to
4. Find the prime factorisation of 2730. obtain identical squares. Find the largest possible length
of a side of each figure.
2730
35

2 1365
28
3 455

5 91

Solution
7 13
35 = 5 × 7
28 =4×7
\  HCF of 35 and 28 is 7.
The largest possible length of a side of each square is
7 cm.

Chapter 1  Factors and Multiples


34
Section 1.3 13. The dimensions of a 1st Local stamp are 40 mm × 30
9. Find the LCM of 40 and 150. mm. Wei Ming placed some 1st Local stamps in two
rows in different orientations as shown below. Find the
Solution minimum number of stamps in the first row such that
 40 = 23 × 5 the two rows are of the same length.
150 = 2 × 3 × 52
\  LCM = 23 × 3 × 52 Solution
= 600 40 = 23 × 5
30 = 2 × 5 × 3
LCM = 23 × 5 × 3
10. Find the LCM of 34 and 57. = 120
Solution
34
= 2 × 17
57
= 3 × 19
Section 1.4
\  LCM = 2 × 3 × 17 × 19 14. Find the value of  484 .
= 1938
Solution
11. Find the LCM of 54, 84 and 110. 484 = 22 × 112
= (2 × 11)2
Solution \ 484 = 2 × 11
  54 = 2 × 33 = 22
 84 = 22 × 3 × 7
110 = 2 × 5 × 11
15. The area of a square is 7225 cm2. Find the length of a
\  LCM = 22 × 33 × 5 × 7 × 11
= 41 580 side of the square.

Solution
12. The figure shows a gear system in which the numbers = 52 × 172
7225
of teeth on the big and small wheels are 20 and 16
respectively. The tooth X on the big wheel and the tooth \ 7225 = 5 × 17
Y on the small wheel are engaged at the start. = 85
The length of a side of the square is 85 cm.

16. Find the cube root of 1000.

Solution
= 23 × 53
1000
= (2 × 5)3
(a) Find the number of tooth contacts that the wheels 3
\ 1000 = 2 × 5
will make before X and Y are engaged again. = 10
(b) Find the number of revolutions that each wheel
will have made by then.
17. The volume of a cube is 2744 cm3. Find the length of a
Solution side of the cube.
= 22 × 5
(a) 20
16 = 24 Solution
\  LCM of 20 and 16 = 24 × 5 = 23 × 73
2744
= 80 = (2 × 7)3
The required number of tooth contacts is 80. 3
\ 2744 = 2 × 7
(b) Number of revolutions the big wheel has made
= 14
80
= The length of a side of the cube is 14 cm.
20
=4
Number of revolutions the small wheel has made
80
=
16
=5

35
Exercise 1.1 6. Find the largest multiple of 17 which is less than 1000.
Level 1 Solution
1. Write down all the factors of each of the following 17 × 58 = 986
numbers. 17 × 59 = 1003
(a) 15 (b) 28 The largest multiple of 17 which is less than 1000
(c) 32 (d) 43 is 986.

Solution
(a) 15 = 1 × 15 7. Find the smallest multiple of 19 which is greater than
= 3 × 5 500.
The factors of 15 are 1, 3, 5 and 15.
Solution
= 1 × 28
(b) 28 19 × 26 = 494
= 2 × 14 19 × 27 = 513
= 4 × 7 The smallest multiple of 19 which is greater than 500
The factors of 28 are 1, 2, 4, 7, 14 and 28. is 513.
= 1 × 32
(c) 32
= 2 × 16 8. Determine whether the following numbers are prime
= 4 × 8 numbers.
The factors of 32 are 1, 2, 4, 8, 16 and 32. (a) 103 (b) 229 (c) 817
= 1 × 43
(d) 43
Solution
The factors of 43 are 1 and 43.
(a) 103 is not divisible by 2, 3, 5, 7, 11, ….
\  103 is a prime number.
2. Write down the first four multiples of each of the
following numbers. (b) 229 is not divisible by 2, 3, 5, 7, 11, 13, ….
(a) 2 (b) 5 \  229 is a prime number.
(c) 11 (d) 23 (c)  817 = 19 × 43
\  817 is not a prime number.
Solution
(a) The first 4 multiples of 2: 2, 4, 6, 8.
9. Express the following in index notation.
(b) The first 4 multiples of 5: 5, 10, 15, 20. (a) 8 × 8 × 8
(c) The first 4 multiples of 11: 11, 22, 33, 44. (b) 3 × 3 × 3 × 3 × 3
(c) 7 × 7 × 9
(d) The first 4 multiples of 23: 23, 46, 69, 92. (d) 4 × 4 × 6 × 6
(e) 2 × 3 × 11 × 11 × 11
3. Determine whether 7 is a factor of 2395. (f ) 5 × 5 × 13 × 5 × 13 × 37

Solution Solution
1 (a) 8 × 8 × 8 = 83
2395 ÷ 7 = 342
7 (b) 3 × 3 × 3 × 3 × 3 = 35
\  7 is not a factor of 2395.
(c) 7 × 7 × 9 = 72 × 9
4. Determine whether 2816 is a multiple of 11. (d) 4 × 4 × 6 × 6 = 42 × 62
(e) 2 × 3 × 11 × 11 × 11 = 2 × 3 × 113
Solution
2816 = 11 × 256 (f) 5 × 5 × 13 × 5 × 13 × 37 = 53 × 132 × 37
\  2816 is a multiple of 11.
10. Find the values of the following.
5. Find the smallest prime factor of 377. (a) 172 (b) 53
2 2
(c) 2 × 11 (d) 34 × 25
Solution
377 = 13 × 29 Solution
The smallest prime factor of 377 is 13. (a) 172 = 289
(b) 53 = 125

Chapter 1  Factors and Multiples


36
(c) 22 × 112 = 4 × 121 = 484 Solution
4 5 (a)
(d) 3 × 2 = 81 × 32 = 2592
180

11. Find the prime factorisation of the following numbers


2 90
using the factor tree method.
(a)
2 45

54 3 15

6 3 5

\ 180 = 22 × 32 × 5

(b)
(b) 616
72
2 308
4
2 154

2 77

7 11
Solution
(a) \ 616 = 23 × 7 × 11
54
(c)
6 9 735

2 3 3 3 3 245

5 49
\ 54 = 2 × 3 × 3 × 3
= 2 × 33 7 7

(b)
\ 735 = 3 × 5 × 72
72
(d)
1350
4 18

2 2 2 9 6 225

3 3 2 3 15 15

3 5 3 5
\ 72 = 2 × 2 × 2 × 3 × 3
= 23 × 32 \ 1350 = 2 × 33 × 52

12. Find the prime factorisation of the following numbers


expressing your answers in index notation. Level 2
(a) 180 (b) 616 13. (a) List all the factors of 56.
(c) 735 (d) 1350 (b) List all the factors of 84.
(c) Hence find all the factors common to 56 and 84.

37
Solution (c) 2 and 8 are factors of 24, but 16 is not a factor of
(a) 56
= 1 × 56 24.
= 2 × 28 The statement is false.
= 4 × 14 [Note: Students are not required to present the proofs in
= 7 × 8 (a) and (b).]
The factors of 56 are 1, 2, 4, 7, 8, 14, 28 and 56.
(b) 84 = 1 × 84 16. Determine whether each statement below is true or false.
= 2 × 42 Give a specific counter example for those that are false.
= 3 × 28 (a) If two numbers are multiples of 11, then their sum
= 4 × 21 is a multiple of 11.
= 6 × 14 (b) If a number is a multiple of 2 and another number
= 7 × 12 is a multiple of 3, then their sum is a multiple of
The factors of 84 are 1, 2, 3, 4, 6, 7, 12, 14, 21, 5.
28, 42 and 84.
Solution
(c) The common factors of 56 and 84 are 1, 2, 4, 7,
(a) If a and b are multiples of 11,
14 and 28.
a = 11s
and b = 11t
for some integers s and t.
14. (a) Write down the first ten multiples of 2.
a + b = 11s + 11t
(b) Write down the first ten multiples of 3.
= 11(s + t)
(c) Hence write down the first three multiples common
a + b is a multiple of 11.
to 2 and 3.
The statement is true.
(d) What can you say about the numbers in (c)?
(b) 4 is a multiple of 2.
Solution 9 is a multiple of 3.
(a) The first ten multiples of 2 are But 4 + 9 = 13 is not a multiple of 5.
2, 4, 6, 8, 10, 12, 14, 16, 18 and 20. The statement is false.
(b) The first 10 multiples of 3 are
3, 6, 9, 12, 15, 18, 21, 24, 27 and 30. 17. (a) Find the missing numbers in the factor tree.
(c) The first three multiples common to 2 and 3 are (b) Can you find the number at the top of the tree
6, 12 and 18. without finding the other two numbers? Explain
briefly.
(d) The numbers in (c) are the first three multiples
of 6.

15. Determine whether each statement below is true or false. 3


Give a specific counter example for those that are false.
(a) If 6 is a factor of a number, then 3 is a factor of
the number. 11
(b) If 2 and 7 are factors of a number, then 14 is a
factor of the number.
(c) If 2 and 8 are factors of a number, then 16 is a 2 7
factor of the number.
Solution
Solution (a)
(a) If 6 is a factor of a number n, then n = 6t where t 462
is a whole number.
\  n = 3(2t)
i.e.  3 is a factor of the number n. 3
154
The statement is true.
(b) If 2 and 7 are factors of a number m, then
11
 m = 2 × 7 × s 14
where s is a whole number.
\  m = 14s
2 7
i.e.  14 is a factor of the number m.
The statement is true.

Chapter 1  Factors and Multiples


38
(b) Yes. (b) 41 = 1 × 41
The top number = 2 × 3 × 7 × 11 They are 12 possible designs.
= 462
22. A lock can only be opened by using a 3-digit number.
18 . Express each of the following as a single number in Jinlan sets this 3-digit number to be the largest prime
index notation. number under 1000. What is this number?
(a) 23 × 22 (b) 34 × 35
5 2
(c) 11 ÷ 11 (d) (53)2 Solution
The largest prime number under 1000 is 997.
Solution
(a) 23 × 22 = 25 (b) 34 × 35 = 39
23. Is it true that if a number is a multiple of 2 and another
(c) 115 ÷ 112 = 113 (d) (53)2 = 53 × 53 = 56 number is a multiple of 3, then their product is a multiple
of 6? Explain briefly using an example.
19 . Express each of the following in index notation, where Solution
all the bases are prime numbers. Let 2m be a multiple of 2 where m > 0.
(a) 12 × 15 (b) 54 × 98 Let 3k be a multiple of 3 where k > 0.
(c) 18 × 10 × 75 (d) 33 × 60 × 125 Product of 2m and 3k
= 2m × 3k
Solution = 6mk
(a) 12 × 15 = (22 × 3) × (3 × 5) 6mk is a multiple of 6.
= 22 × 32 × 5 The statement is true.
(b) 54 × 98 = (2 × 33) × (2 × 72)
= 22 × 33 × 72 24. The prime numbers 17 and 19 are called twin primes
2
(c) 18 × 10 × 75 = (2 × 3 ) × (2 × 5 ) × (3 × 5 ) 2 because they differ by 2. List three other pairs of twin
= 22 × 33 × 53 primes.

(d) 33 × 60 × 125 = (3 × 11) × (22 × 3 × 5) × 53 Solution
= 22 × 32 × 54 × 11 There are infinite number of pairs of twin primes. The
following are some examples:
3 and 5, 5 and 7, 11 and 13, 41 and 43, 71 and 73.
Level 3
20. The Olympic Games are held in 25. (a) Express each of the following even numbers as a
years that are multiples of 4. In sum of two prime numbers.
which years will the next three (i) 32 (ii) 78 (iii) 116
Olympic Games be held after the
(b) A mathematician proposed that “Every even
London Olympic Games in
number greater than 2 can be expressed as a sum
2012?
of two prime numbers.” Do you agree? Why? This
proposal is called Goldbach Conjecture. You may
Solution
surf the Internet to find more information about it.
After the year 2012, the next three consecutive Olympic
Games are held in the years 2016, 2020 and 2024. Solution
(a) (i) 32 = 13 + 19
21. A design is made by arranging a certain number of square (ii) 78 = 11 + 67
tiles to form a rectangular array. Find the number of (iii) 116 = 13 + 103
possible designs if there are (b) The statement “Every even number greater than 2
(a) 18 tiles, can be expressed as a sum of two prime numbers.”
(b) 41 tiles. is known as Goldbach Conjecture. No one can
prove or disprove it. However, it is shown that
Solution the statement is true for very large even numbers.
(a) 18 = 1 × 18 = 2 × 9 Therefore, we should agree with the statement.
=3×6
They are 6 possible designs.
Note: A design of 2 columns by 9 rows of tiles is
different from a design of 9 columns by 2
rows of tiles.

39
26 . The prime factorisation of a number is 24 × 35 × 72 × (e) 74 = 2 × 37
11. Write down 3 factors of the number that are greater 99 = 32 × 11
than 100. HCF = 1
Hint: 16 × 11 = 176 which is a factor greater than 100.
(f ) 120 = 23 × 3 × 5
Solution 225 = 32 × 52
The number = 24 × 35 × 72 × 11 HCF = 3 × 5
A product of some of its prime factors is a factor of the = 15
number. For example,
1st factor = 24 × 3 × 11 (g) 108 = 22 × 33
= 528, 240 = 24 × 3 × 5
2nd factor = 24 × 3 × 7 HCF = 22 × 3
= 336, = 12
3rd factor = 35
(h) 385 = 5 × 7 × 11
= 243
396 = 22 × 32 × 11
are factors greater than 100.
HCF = 11

27. The prime factorisation of two numbers are


2 × 32 × 73 × 13 and 3 × 72 × 133 × 17. A common Level 2
factor of these two numbers is 3 × 7 = 21. Write down 2. Find the HCF of each group of numbers.
3 other common factors of these two numbers. (a) 28, 63 and 91 (b) 60, 75 and 100
(c) 48, 84 and 144 (d) 66, 110 and 847
Solution (e) 14, 36 and 175 (f ) 70, 210 and 350
Some common factors of the two numbers are as follows:
3, 7, 13, 3 × 7, 72, 7 × 13, … Solution
i.e.  3, 7, 13, 21, 49, 91, … (a) 28 = 22 × 7
63 = 32 × 7
91 = 7 × 13
Exercise 1.2 HCF = 7
Level 1
(b) 60 = 22 × 3 × 5
1. Find the HCF of each pair of numbers. 75 = 3 × 52
(a) 8 and 12 (b) 18 and 27 100 = 22 × 52
(c) 45 and 72 (d) 21 and 84 HCF = 5
(e) 74 and 99 (f) 120 and 225
(g) 108 and 240 (h) 385 and 396 (c) 48 = 24 × 3
84 = 22 × 3 × 7
Solution 144 = 24 × 32
(a) 8 = 23   HCF = 22 × 3
12 = 22 × 3 = 12
HCF = 22
=4 (d) 66 = 2 × 3 × 11
110 = 2 × 5 × 11
= 2 × 32
(b) 18 847 = 7 × 112
27 = 33 HCF = 11
HCF = 32
=9 (e) 14 = 2 × 7
36 = 22 × 32
(c) 45 = 5 × 32 175 = 52 × 7
72 = 23 × 32 HCF = 1
HCF = 32
=9 (f ) 70 =2× 5×7
210 =2× 3×5×7
(d) 21
=3×7 350 =2× 52 × 7
84 = 22 × 3 × 7   HCF = 2 × 5×7
HCF = 3 × 7 = 70
= 21

Chapter 1  Factors and Multiples


40
Level 3 7. A charitable organisation distributes 420 packets of rice,
3. There are two metal bars of lengths 72 cm and 96 cm. 168 bottles of oil and 504 oranges equally among some
Both bars are exactly cut into small pieces of equal senior citizens. Find the greatest possible number of
lengths. Find the largest possible length of each small senior citizens.
pieces. Solution
420 = 22 × 3 × 5 × 7
Solution
168 = 23 × 3 × 7
72 = 23 × 32
504 = 23 × 32 × 7
96 = 25 × 3
The greatest possible number of senior citizens
HCF of 72 and 96 = 23 × 3 = 24
= HCF of 420, 168 and 504
The largest possible length of each small piece is 24 cm.
= 22 × 3 × 7
= 84
4. A rectangular piece of tin plate measuring 360 cm by
280 cm is cut into identical small squares of the largest 8. A botanist conducts a controlled experiment using 48
possible length. Find seeds A and 78 seeds B. Each type of seeds is divided
(a) the length of a side of each square, equally into as many groups as possible such that the
(b) the number of square tin plates formed. number of groups of seeds A and the number of groups
of seeds B are the same. Find
Solution (a) the number of groups of seeds A,
(a) 360 = 23 × 32 × 5 (b) the number of seeds A in each group,
280 = 23 × 5 × 7 (c) the number of seeds B in each group.
HCF of 360 and 280 = 23 × 5 = 40
The length of a side of each square is 40 cm.e is Solution
360 280 (a) 48 = 24 × 3
(b) × = 63 78 = 2 × 3 × 13
40 40
The number of square tin plates formed is 63. Number of groups of seeds A
= HCF of 48 and 78
= 2 × 3
5. 126 pieces of white chocolate and 84 pieces of dark
= 6
chocolate are divided into packs with equal number of
pieces of each kind of chocolate. What is the greatest (b) Number of seeds A in each group
possible number of packs needed? = 48 ÷ 6
Solution (c) Number of seeds B in each group
The number of packs should be a common factor of 126 = 78 ÷ 6
and 84. = 13
126 = 2 × 32 × 7
84 = 22 × 3 × 7 9. The product of two numbers is 8820. The HCF of these
The greatest number of packs numbers is 42. Find the greater number.
= HCF of 126 and 84
= 2 × 3 × 7 Solution
= 42 8820 = 22 × 32 × 5 × 72
HCF = 42 = 2 × 3 × 7
6. The total sales of a particular model of calculator on The two numbers can only be 2 × 3 × 7 and
three days are $180, $414 and $306 respectively. Find 2 × 3 × 5 × 7.
the greatest possible price of the calculator. The greatest number = 2 × 3 × 5 × 7
= 210
Solution
180 = 22 × 32 × 5
10. Find two different numbers such that their HCF is 18.
414 = 2 × 32 × 23
306 = 2 × 32 × 17
Solution
Greatest possible price of the calculator in $
The two numbers can be 2 × 18 and 3 × 18,
= HCF of 180, 414 and 306
i.e. 36 and 54.
= 2 × 32
The two numbers can also be 18 and 5 × 18,
= 18
i.e. 18 and 90.

41
11. Find three numbers such that the HCF of each pair of Exercise 1.3
these numbers is greater than 1 and the HCF of all three Level 1
numbers is 1.
Hint: For instance, the numbers 6, 10 and 15 satisfy the 1. Find the LCM of each pair of numbers.
conditions. (a) 12 and 15
(b) 6 and 28
Solution (c) 25 and 40
Let us consider three prime numbers such as 3, 5 and 7. (d) 23 and 32
We form the required three numbers by taking the pairwise (e) 24 and 54
products of these prime numbers: (f ) 60 and 75
3 × 5, 3 × 7 and 5 × 7, (g) 59 and 118
i.e.  15, 21 and 35. (h) 65 and 91

Solution
12. Find the greatest number that will divide 171, 255 and (a) 12 = 22 × 3
304 so as to leave the same remainder in each case. 15 = 3 × 5
LCM = 22 × 3 × 5
Solution = 60
255 – 171 = 84 = 22 × 3 × 7
304 – 171 = 133 = 7 × 19 (b) 6 = 2×3
304 – 255 = 49 = 72 28 = 22 × 7
LCM = 22 × 3 × 7
When 84, 133 and 49 are divided by the required number, = 84
the remainders should be zero.
The required number (c) 25 = 52
= HCF of 84, 133 and 49 40 = 23 × 5
= 7 LCM = 23 × 52
= 200
13. In a shop, the price (in dollars) of a model car in a shop (d) 23 = 1 × 23
is a whole number greater than 1. The sales of the model 32 = 25
cars on two days are $1518 and $2346. How many model LCM = 25 × 23
cars are sold on each day? = 736
(e) 24 = 23 × 3
Solution
54 = 2 × 33
1518 = 2 × 3 × 11 × 23
LCM = 23 × 33
2346 = 2 × 3 × 17 × 23
= 216
The price (in dollars) of a model car can be any common
factor of 1518 and 2346 that is greater than 1. (f ) 60 = 22 × 3 × 5
i.e. The price may be $2, $3, $6, $23, $46, $69 or $138. 75 = 3 × 52
LCM = 22 × 3 × 52
The corresponding number of model cars sold on each
= 300
day is shown in the following table.
(g) 59 = 1 × 59
Number of model cars sold 118 = 2 × 59
Price ($) LCM = 2 × 59
Day 1 Day 2
= 118
2 759 1173
(h) 65 = 5 × 13
3 506 782 91 = 7 × 13
LCM = 5 × 7 × 13
6 253 391
= 455
23 66 102
46 33 51 Level 2
69 22 34 2. Find the LCM of each group of numbers.
138 11 17 (a) 9, 12 and 30
(b) 13, 14 and 15
(c) 6, 8 and 20
(d) 28, 42 and 105

Chapter 1  Factors and Multiples


42
(e) 22, 132 and 253 Solution
(f  ) 4, 9 and 31 (a) 18 = 2 × 3 × 3
30 = 2 × 3 × 5
Solution HCF = 2 × 3
(a) 9 = 32 = 6
12 = 22 × 3 LCM = 2 × 3 × 3 × 5
30 = 2 × 3 × 5 = 90
LCM = 22 × 32 × 5
(b) 14 = 2 × 7
= 180
35 = 5 × 7
(b) 13 = 1 × 13 HCF = 7
14 = 2×7 LCM = 2 × 5 × 7
15 = 3×5 = 70
LCM = 2 × 3 × 5 × 7 × 13
(c) HCF × LCM = Product of the given numbers
= 2730
(d) Let the other number be a.
(c) 6 = 2×3
8 = 23 13 × 3640 = 520a
20 = 22 × 5 a = 91
LCM = 23 × 3 × 5
= 120
Level 3
(d) 28 = 22 × 7
5. A flash bulb on a Christmas tree flashes once every
42 = 2×3×7
10 seconds. Another bulb flashes once every 15 seconds.
105 = 3×5×7
If they are flashing together now, how long will it take
LCM = 22 × 3 × 5 × 7
for the two bulbs to flash together next?
= 420
(e) 22 = 2 × 11 Solution
132 = 22 × 3 × 11 10 = 2 × 5
253 = 11 × 23 15 =3×5
LCM = 22 × 3 × 11 × 23 LCM of 10 and 15 = 2 × 3 × 5
= 3036 = 30
The bulbs will next flash together after 30 seconds.
(f ) 4 = 22
9 = 32
31 = 1 × 31 6. John and Arun are running along a circular track. They
LCM = 22 × 32 × 31 take 48 seconds and 56 seconds to complete a lap
= 1116 respectively. They begin from the starting point at the
same time in the same direction.
(a) How long does it take for them to meet at the
3. The prime factorisation forms of two numbers are
starting point again?
24 × 35 × 53 × 72 and 23 × 36 × 5 × 78. Find
(b) How many laps will each boy have run by then?
(a) the HCF of these two numbers in prime factorisation
form, Solution
(b) the LCM of these two numbers in prime factorisation (a) 48 = 24 × 3
form. 56 = 23 × 7
LCM of 48 and 56 = 24 × 3 × 7
Solution
= 336
(a) HCF = 23 × 35 × 5 × 72
They will meet at the starting point again after
(b) LCM = 24 × 36 × 53 × 78 336 seconds.
(b) Number of laps that John has run
4. (a) Find the HCF and LCM of 18 and 30. = 336 ÷ 48
(b) Find the HCF and LCM of 14 and 35. =7
(c) What is the relationship between the product of the Number of laps that Arun has run
HCF and LCM of two numbers and the numbers = 336 ÷ 56
themselves? =6
(d) The HCF of two numbers is 13 and their LCM is
3640. If one number is 520, find the other number.

43
7. The thickness of a Science book is 20 mm and that of Solution
a Mathematics book is 28 mm. Books of each type are 30 = 2 × 3 × 5
stacked up in a separate pile. 24 = 23 × 3
(a) What should be the minimum height of each pile 50 = 2 × 52
such that both piles are of the same height? LCM of these three numbers
(b) Find the number of books in each pile. = 23 × 3 × 52
= 600
Solution
The cameras scan together every 600 seconds, i.e. 10
(a) 20 = 22 × 5
min. Hence, they will next scan together at 06 10.
28 = 22 × 7
LCM of 20 and 28 = 22 × 5 × 7 = 140
The minimum height of each pile should be 10. Doris has piano tutoring once every 6 days, swimming
140 mm. lessons once every 4 days and ballet lessons once every
8 days. If she has all these three activities on 1 April,
(b) Number of Science books in the pile
on which date will she have all of them together next?
= 140 ÷ 20
= 7
Solution
Number of Mathematics books in the pile
6 = 2 × 3
= 140 ÷ 28
4 = 22
= 5
8 = 23
LCM of these numbers = 23 × 3
8. Each student desk in a classroom has a rectangular
= 24
desktop that measures 60 cm by 45 cm. Some of these
desks are arranged, side by side, as shown in the diagram Doris has all the three activities once every 24 days.
to form a large square table for a class activity. She will next have all of them on 25 April.
60

45 11. Buses A, B and C are at a bus interchange at intervals


of 20 minutes, 30 minutes and 45 minutes respectively.
If all of them are at the interchange at 08 30, find the
time when they will meet at the interchange next.

Solution
20 = 22 × 5
Find 25 = 52
(a) the least length of a side of the square, 45 = 32 × 5
(b) the number of rows and columns of desks used to LCM of these numbers = 22 × 32 × 52
form the large square table. = 900
900 min = 900 ÷ 60 h
Solution = 15 h
= 22 × 3 × 5
(a) 60
45 = 32 × 5 All the three buses will be at the interchange once every
LCM of 60 and 45 = 22 × 32 × 5 15 hours.
= 180 08 30 + 15 00 = 23 30
The least length of a side of the square is 180 cm. They will next meet at the interchange at 23 30.
(b) The number of rows of desks = 180 ÷ 45
=4 12. Find three pairs of numbers such that the LCM of each
The number of columns of desks = 180 ÷ 60 pair of numbers is 24.
=3
Solution
24 = 23 × 3
9. A jewellery cabinet is monitored by three security
cameras. The first camera scans it every 30 seconds, the The pairs of numbers with LCM = 24 can be 23 and 3,
second scans every 24 seconds and the third scans every 22 and 23 × 3, 2 and 23 × 3, 23 and 2 × 3, etc.
50 seconds. The three cameras scan the cabinet together i.e. 8 and 3, 4 and 24, 2 and 24, 8 and 6, etc.
at 06 00. At what time will they scan the cabinet together
next?
13. Find two possible pairs of numbers such that the HCF and
LCM of each pair of numbers are 21 and 630 respectively.

Chapter 1  Factors and Multiples


44
Solution (c) 196 = 22 × 72
21 = 3 × 7 = (2 × 7)2
630 = 2 × 32 × 5 × 7 196 = 14
Some possible pairs of numbers are:
3 × 7 and 2 × 32 × 5 × 7, (d) 256 = 28
2 × 3 × 7 and 32 × 5 × 7, = 24 × 24
3 × 5 × 7 and 2 × 32 × 7. 256 = 24
i.e.  21 and 630, 42 and 315, 105 and 126.
= 16
(e) 441 = 32 × 72
14. Find the greatest 3-digit number which is divisible by
= (3 × 7)2
15, 20 and 24.
441 = 21
Solution
(f ) 676
= 22 × 132
15 = 3 × 5 = (2 × 13)2
20 = 22 × 5 676 = 26
24 = 23 × 3

LCM of these numbers = 23 × 3 × 5
= 120 2. Find the cube root of each of the following numbers.
A number which is divisible by 15, 20 and 24 is a multiple (a) 343 (b) 512
of 120. By trial and error, the required 3-digit number (c) 729 (d) 1331
= 120 × 8 (e) 4096 (f ) 8000
= 960
Solution
15. Find the smallest number that should be added = 73
(a) 343
3
to 1628 so that the sum is exactly divisible by 4, 343 = 7
5 and 6.
= 29
(b) 512
Solution
= 23 × 23 × 23
3
4 = 22 512 = 23
5 = 5 =8
6 = 2 × 3 = 36
(c) 729
LCM of these numbers = 22 × 3 × 5 = 32 × 32 × 32
= 60 3
2 729 = 32
1628 ÷ 60 = 27 =9
15
The required smallest number (d) 1331 = 113
= 60 × 28 – 1628 3
1331 = 11
= 52
(e) 4096 = 212
= 24 × 24 × 24
Exercise 1.4
3
4096 = 24
Level 1 = 16
1. Find the value of each of the following numbers. (f ) 8000 = 26 × 53
(a) 36 (b) 121 = (22 × 5) × (22 × 5) × (22 × 5)
3
(c) 196 (d) 256 8000 = 22 × 5
= 20
(e) 441 (f ) 676

Solution Level 2
(a) 36 = 22 × 32 3. Find the positive square roots of the following numbers
= (2 × 3)2 given in prime factorisation form. Express your answers
36 = 6 in index notation.
(a) 54 × 72 (b) 26 × 1110
(b) 121 = 112
121 = 11

45
Solution (b) The required square root
(a) 54 × 72 = (52 × 7) × (52 × 7)
= 212 × 56
4 2 2
5 × 7 = 5 × 7
= (26 × 53)2
6 10 3 5 3 5
(b) 2 × 11 = (2 × 11 ) × (2 × 11 ) = 26 × 53
26 × 1110 = 23 × 115
8. (a) Find the square of 76 × 193.
4. Find the cube roots of the following numbers given in (b) Find the cube root of the result in (a).
prime factorisation form. Express your answers in index Express your answers in index notation.
notation.
(a) 23 × 196 (b) 312 × 59 Solution
8. (a) (a) The required square = (76 × 193)2
Solution = 712 × 196
(a) 23 × 196 = (2 × 192)3 3
3
(b) The required cube root = 712 × 196
3 6 2
2 × 19 = 2 × 19 3
= (74 × 192)3
(b) 312 × 59 = (34 × 53)3 = 74 × 192
3
312 × 59 = 34 × 53
Level 3
5. (a) Find the HCF of 63 and 117. 9. The area of a square tin plate is 7056 cm2. Find the length
(b) Find the positive square root of the HCF in (a). of a side of the plate.

Solution Solution

(a) 63 = 3 × 7 2 7056 = 24 × 32 × 72
117 = 32 × 13 = (22 × 3 × 7)2
HCF of 63 and 117 = 32 7056 = 22 × 3 × 7
=9 = 84
The length of a side of the plate is 84 cm.
(b) The positive square root of 9 = 9
=3
10. The area of a square frame is 2601 cm2. Find the perimeter
of the frame.
6. (a) Find the LCM of 24 and 108.
(b) Find the cube root of the LCM in (a). Solution
2601 = 32 × 172 = (3 × 17)2
Solution
(a) 24 = 23 × 3 2601 = 3 × 17 = 51
108 = 22 × 33 The length of a side of the frame is 51 cm.
LCM of 24 and 108 = 23 × 33 51 × 4 cm = 204 cm
= 216 The perimeter of the frame is 204 cm.

3
(b) The cube root of 216 = 216 11. (a) Find the prime factorisation of 129 600.
=6 (b) In an experiment, the speed of a bullet is found to
be 129 600 m/s. Find the value of the speed of
7. (a) Find the cube of 24 × 52. the bullet in index notation.
(b) Find the positive square root of the result in (a).
Express your answers in index notation. Solution
(a) 129600 = 26 × 34 × 52
Solution
(a) The required cube = (24 × 52)3 (b) Speed of the bullet
= 212 × 56 = 129 600
= 26 × 34 × 52
= (23 × 32 × 5)2
= (23 × 32 × 5) m/s

Chapter 1  Factors and Multiples


46
12. The volume of a piece of glass cube is 1728 cm3. Find (b) (i) By testing,
the length of a side of the cube. 5832 = 183.
5 r 32 is a perfect cube when r = 8.
Solution 3
(ii) 5832 = 18
1728 = 26 × 33
= (22 × 3)3 (c) The numbers 5232, 5532 and 5832 are even
3
1728 = 22 × 3 numbers and the sum of the digits of each number
is a multiple of 3.
= 12
The length of a side of the cube is 12 cm.
Revision Exercise 1
13. A piece of wire is cut and soldered to form the framework 1. Find the smallest number that has 2, 5 and 7 as its prime
of a cube. The volume of the cube is 10 648 cm3. Find factors.
(a) the length of a side of the cube,
(b) the length of the wire used. Solution
Solution
The required number = 2 × 5 × 7
(a) 10 648 = 23 × 113 = 70
= (2 × 11)3

3
10 648 = 2 × 11 = 22 2. Find the prime factorisation of the greatest 3-digit number.
The length of a side of the cube is 22 cm. Solution
(b) 22 × 12 cm = 264 cm The greatest 3-digit number = 999
The length of the wire used is 264 cm. = 33 × 37

14. (a) Find the prime factorisation of 21 952. 3. Determine whether each number is prime or composite.
(b) The radius of a snowball is found to be (a) 649 (b) 721
3
21 952 mm. Find the value of the radius in index
Solution
notation.
(a) 649 = 11 × 59
649 is a composite number.
Solution
(a) 21 952 = 26 × 73 (b) 721 = 7 × 103
(b) Radius of the snowball 721 is a composite number.
3
= 21 952
3 4. Determine whether each statement is true or false.
= 26 × 73
3
(a) If 3 and 5 are factors of a number, then 15 is a
= (22 × 7)3 factor of the number.
= (22 × 7) mm (b) If 246 is a multiple of a number, then 123 is a
multiple of the number.

15. It is given that 6 is a factor of 5 r 32 where r represents Solution


a missing digit. (a) If 3 is a factor of a number n, then
(a) Find all the possible values of r. n = 3 × t
(b) Suppose 5 r 32 is a perfect cube. where t is a whole number.
(i) Find the value represented by r. If 5 is also a factor of n, then 5 should be a
(ii) Find the cube root of the number. factor of t.
(c) Study all the possible numbers 5 r 32 formed in \  t = 5 × m for some integer m.
(a). Suggest some of their common properties. i.e. n = 3 × 5 × m = 15 × m
\  15 is a factor of n.
Solution The statement is true.
(a) By testing the divisibility of 5 r 32 by 6
for r = 0, 1, 2, … 9, (b) 246 = 3 × 82
we find that 246 is a multiple of 82, but 123 is not a multiple
5232 = 6 × 872, of 82.
5532 = 6 × 922, The statement is false.
5832 = 6 × 972.
The possible values of r are 2, 5 and 8.

47
5. (a) Complete the following factor trees. 6. Given three numbers 12,40 and 45, find
(i) (a) their HCF,
150 (b) their LCM,
(c) Find the greatest 4-digit number which is a common
multiple these numbers.
25
Solution
(a) 12 = 22 × 3
40 = 23 × 5
45 = 32 × 5
(ii) HCF = 1
(b) LCM = 23 × 32 × 5
= 360
(c) 27 × 360 = 9720
3 28 × 360 = 10 080
The greatest 4-digit number which is a common
multiple of 12, 40 and 45 is 9720.
5 4
7. (a) Find the prime factorisation of
2 (i) 12
(ii) 144
(iii) 5040
(b) The HCF and LCM of two numbers are 12 and
(b) Write down the prime factorisation of the number
5040 respectively. If one of the numbers is 144,
at the top in each tree.
find the other number.
(c) Find the HCF of the numbers in (b).
(d) Find the LCM of the numbers in (b). Solution
(a) (i)  12 = 22 × 3
Solution
(ii)  144 = 24 × 32
(a) (i)
(iii) 5040 = 24 × 32 × 5 × 7
150
(b) The other number = 22 × 3 × 5 × 7
= 420
25 6
8. (a) Express 240 as a product of its prime factors.
5 5 (b) Find the HCF of 75 and 240.
2 3
(c) Each box of lollies has 240 pieces. Each pack of
cookies has 75 pieces. Mrs Tan buys the same
(ii)
60 number of lollies and cookies. Find the least number
of boxes of lollies that she would have bought.

3 Solution
20
(a) 240 = 24 × 3 × 5
(b) 75 = 3 × 52
5 4
HCF of 75 and 240
= 3 × 5
2 = 15
2
(c) The least number of lollies that Mrs Tan would
2 have bought is the LCM of 75 and 240,
(b) 150 = 2 × 3 × 5 ,
The number = 24 × 3 × 52
 60 = 22 × 3 × 5
= 1200
(c) HCF = 2 × 3 × 5 The least number of boxes bought
= 30 = 1200 ÷ 240
= 5
(d) LCM = 22 × 3 × 52

= 300

Chapter 1  Factors and Multiples


48
9. The product of two numbers is 3388 and their HCF is 13. Ahmad, Brian and Clara go to ABC Restaurant for
11. Find all the possible pairs of the numbers. dinner on regular intervals of 6 days, 8 days and
15 days respectively. If they go to the restaurant
Solution on 1 January, what is the next date that all of
3388 = 22 × 7 × 112 them will go there?
HCF =11
If one number is 11, Solution
the other number = 22 × 7 × 11 6 = 2 × 3
= 308 8 = 23
If one number is 11 × 22, 15 = 3 × 5
the other number = 7 × 11 LCM of their numbers = 23 × 3 × 5
= 77 = 120
The possible pairs of numbers are (11 and 308) and (44 The three persons go to the restaurant once every 120
and 77). days. Hence, the next day will be 1 May (if it is not a
leap year), or 30 April (if it is a leap year).
10. The dimensions of a rectangle are (2 5 × 7) cm by
(2 × 52 × 73) cm. 14. A box contains an assortment of 3 types of chocolate
(a) Find the area of the rectangle. Express your answer bars. It has 18 bars with almonds, 24 bars with hazelnuts
in prime factorisation form. and 30 bars with peanuts. The chocolate bars are shared
(b) A square has the same area as the rectangle. Find among some students. Each student has only one type of
the length of a side of the square. chocolate bar and every student has the same number of
chocolate bars. If each student has the greatest number
Solution of chocolate bars.
(a) Area of the rectangle (a) how many chocolate bars does each student get?
= (25 × 7) × (2 × 52 × 73) (b) how many students will get chocolate bars with
= (26 × 52 × 74) cm2 peanuts?
(b) Length of a side of the square
Solution
= 26 × 52 × 74 (a) 18 = 2 × 32
= 23 × 5 × 72 24 = 23 × 3
= 1960 cm 30 = 2 × 3 × 5
HCF of 18, 24 30 = 2 × 3
=6
11. (a) Find the value of 2601 . Each student gets 6 chocolate bars.
3
(b) Find the value of 375 × 243 .
(b) 30 ÷ 6 = 5
(c) Find the HCF of the numbers in (a) and (b). Five students will get chocolate bars with peanuts.

Solution
15. A rectangular board measures 630 cm by 396 cm. It is
(a) 2601 = 51
divided into small squares of equal size.
(b)
3
375 × 243 (a) (i) Find the largest possible length of the side of
= 45 a square.
(ii) Find the least number of squares.
(c) 51 = 3 × 17 (b) (i) Find the second largest possible length of the
45 = 32 × 5 side of a square.
HCF = 3 (ii) Find the number of squares in this case.

12. A bell rings every 25 minutes while another bell rings Solution
every 40 minutes. Suppose they rang together at 6 a.m., = 2 × 32 × 5 × 7
(a) (i) 630
when will they ring together next? 396 = 22 × 32 × 11
HCF of 630 and 396 = 2 × 32
Solution = 18
25 = 52 The largest possible length is 18 cm.
40 = 23 × 5 630 396
LCM of 25 and 40 = 23 × 52 (ii) × = 770
18 18
= 200 The least number of squares is 770.
200 minutes = 3 hours 20 minutes
The bells will next ring together at 9.20 a.m.

49
(b) (i) The second largest possible length is 9 cm.
630 396
(ii) × = 3080
9 9
The required number of squares is 3080.

16. The Singapore Flyer is a giant observation wheel with


seating capacity of 784 passengers. The number of
passengers that each capsule can carry is equal to the
total number of capsules on the Singapore Flyer.
(a) Find the prime factorisation of 784 and express
your answer in index notation.
(b) Hence find the total number of capsules on the
Singapore Flyer.

Solution
(a) 784 = 24 × 72
(b) Total number of capsules
= 24 × 72
= 22 × 7
= 28

Chapter 1  Factors and Multiples


50
2 Real Numbers

Class Activity 1
Objective: To identify the use of negative numbers in the real world.

Task
Take a look at your surroundings. Where can you find the use of negative numbers? You may take photos of such scenarios
using a camera. Show and discuss with your classmates what the ‘–’ sign means in these scenarios you have captured.

Suggested Answers:
• Time zone
• Temperature
• Hockey scores
• Golf scores
• Music Chart
• Stock Market
• Timeline
• Altitude
• Credit card balances
• Latitudes
• Angles
• Polarities in electricity

Class Activity 2
Objective: To perform addition of integers using algebra discs.

Tasks
1. Adding two integers with the same sign
For example, evaluate (–3) + (–2).


–1 –1 Place 3 –1 discs in the first row.

–1 –1 –1 Place 2 –1 discs in the second row.



\ (–3) + (–2) = –5 Count the total number of –1 discs.

In fact, we can also write (–3) + (–2) = –3 – 2 or (–3) + (–2) = (–2) + (–3)
= –5 = –2 – 3
= –5
Here, we add two negative numbers together, thus the answer is negative.

51
Evaluate the following.
(a) 2 + 3

1 1 Place 2 1 discs in the first row.



1 1 1 Place 3 1 discs in the second row.

\ 2 + 3 = 5 Count the total number of 1 discs.

(b) 5 + 1

1 1 1 1 1 Place 5 1 discs in the first row.



1 Place 1 1 discs in the second row.

\ 5 + 1 = 6 Count the total number of 1 discs.

(c) (–2) + (–4)

–1 –1 Place 2 –1 discs in the first row.



–1 –1 –1 –1 Place 4 –1 discs in the second row.

\ (–2) + (–4) = –6 Count the total number of –1 discs.

(d) (–3) + (–5)

–1 –1 –1 Place 3 –1 discs in the first row.



–1 –1 –1 –1 –1 Place 5 –1 discs in the second row.

\ (–3) + (–5) = –8 Count the total number of –1 discs.

2. Adding two integers with different signs


For example, evaluate (–5) + 2.

–1 –1 –1 –1 –1 Place 5 –1 discs in the first row.

1 1 Place 2 1 discs in the second row.

\ (–5) + 2 = –3 Count the discs left after taking away the zero pairs.

We can also write (–5) + 2 = –5 + 2 or (–5) + 2 = 2 + (–5)


= –3 = 2 – 5
= –3

Here, we add a positive number and a negative number together. Since there are more –1 discs than 1 discs, the
answer is negative.

Evaluate the following.

(a) 7 + (–3)

1 1 1 1 1 1 1 Place 7 1 discs in the first row.

–1 –1 –1 Place 3 –1 discs in the second row.

\ 7 + (–3) = 4 Count the discs left after taking away the zero pairs.

Chapter 2  Real Numbers


52
(b) (–4) + 6

–1 –1 –1 –1 Place 4 –1 discs in the first row.

1 1 1 1 1 1 Place 6 1 discs in the second row.

\ (–4) + 6 = 2 Count the discs left after taking away the zero pairs.

(c) 3 + (–5)

1 1 1 Place 3 1 discs in the first row.

–1 –1 –1 –1 –1 Place 5 –1 discs in the second row.

\ 3 + (–5) = –2 Count the discs left after taking away the zero pairs.

(d) (–8) + 2

–1 –1 –1 –1 –1 –1 –1 –1 Place 8 –1 discs in the first row.

1 1 Place 2 1 discs in the second row.

\ (–8) + 2 = –6 Count the discs left after taking away the zero pairs.

3. (a) Think of two integers ▲ + ■ (including negative integers).

Calculate both ▲ + ■ and ■ + ▲.

(b) Repeat (a) with 2 other different integers. Do this two more times.

Suggested answer for (a) and (b):

Consider two integers a and b where a > 0 and b > 0.

Case ▲+■ ■+▲


1 (+a) + (+b) = a + b (+b) + (+a) = b + a
2 (–a) + (–b) = –a – b (–b) + (–a) = –b – a
3 (+a) + (–b) = a – b (–b) + (+a) = –b + a
4 (–a) + (+b) = –a + b (+b) + (–a) = b – a
5 (+a) + 0 = a 0 + (+a) = a
6 (–a) + 0 = –a 0 + (–a) = –a

(c) From (a) and (b), what do you observe?

The results are always the same. The order of addition does not affect the result.

53
Class Activity 3
Objective: To perform subtraction of integers using algebra discs.

Tasks
1. Subtracting a positive integer
For example, evaluate 3 – 7.

1 1 1 1 1 Place 3 1 discs in the first row.

–1 –1 Place 7 –1 discs in the second row.

\ 3 – 7 = – 4 Count the discs left after taking away the zero pairs.

In fact, we can also write 3 – 7 = 3 + (–7)


= – 4
or 3 – 7 = 3 + (–7)
= (–7) + 3
= –7 + 3
= – 4

Here, we add a positive number and a negative number together. Since there are more –1 discs than 1 discs, the
answer is negative.

For example, evaluate –2 – 4.



–1 –1 Place 2 1 discs in the first row.

–1 –1 –1 –1 Place 4 –1 discs in the second row.

\  –2 – 4 = –6 Count the total number of –1 discs.

We can also write –2 – 4 = (–2) + (– 4)


= –6

Here, we add two negative numbers together, thus the answer is negative.

Evaluate the following.

(a) 5 – 2

1 1 1 1 1 Place 5 1 discs in the first row.

–1 –1 Place 2 –1 discs in the second row.

\ 5 – 2 = 3 Count the discs left after taking away the zero pairs.

(b) 3 – 4

1 1 1 Place 3 1 discs in the first row.

–1 –1 –1 –1 Place 4 –1 discs in the second row.

\ 3 – 4 = –1 Count the discs left after taking away the zero pairs.

Chapter 2  Real Numbers


54
(c) –3 – 1


–1 –1 –1 Place 3 –1 discs in the first row.
–1 Place 1 –1 discs in the second row.

\ 3 – 1 = –4
Count the total number of –1 discs.

(d) –6 – 3


–1 –1 –1 Place 6 –1 discs in the first row.
–1 –1 –1 –1 –1 Place 3 –1 discs in the second row.

\  –6 – 3 = –9
Count the total number of –1 discs.

2. Subtracting a negative integer


For example, evaluate 5 – (–2).
1 1 1 1 1 1 1 1 1 1

– –1 –1 1 1

Take the negative of –2


(Flip over the discs inside the box)

\ 5 – (–2) = 7

We can also write 5 – (–2) = 5 + 2   Change signs as the discs are flipped.
=7
Here, we add two positive numbers together, thus the answer is positive.

For example, evaluate (–3) – (–8).



–1 –1 –1 –1 –1 –1

– –1 –1 –1 –1 –1 –1 –1 –1 ➜ 1 1 1 1 1 1 1 1

Take the negative of –8 Remove the zero pairs


(Flip over the discs inside the box)

➜ 1 1 1 1 1

\ (–3) – (–8) = 5

We can also write (–3) – (–8) = (–3) + 8 Change signs as the discs are flipped.
= –3 + 8
=5

Here, we add a negative number and a positive number together. Since there are more 1 discs than –1 discs, the
answer is positive.

55
Evaluate the following.

(a) 6 – (–3)

1 1 1 1 1 1 1 1 1 1 1

– –1 –1 –1 1 1 1

Take the negative of –3


(Flip over the discs inside the box)

\ 6 – (–3) = 6 + 3  Change signs as the discs are flipped.
=9

(b) 3 – (–4)

1 1 1 1 1 1

– –1 –1 –1 –1 1 1 1 1

Take the negative of –4


(Flip over the discs inside the box)

\ 3 – (–4) = 3 + 4  Change signs as the discs are flipped.
=7

(c) –4 – (–7)

–1 –1 –1 –1 –1 –1 –1 –1

– –1 –1 –1 –1 –1 –1 –1 ➜ 1 1 1 1 1 1 1

Take the negative of –7 Remove the zero pairs


(Flip over the discs inside the box)

➜ 1 1 1

\  –4 – (–7) = –4 + 7  Change signs as the discs are flipped.
=3

(d) –7 – (–5)

–1 –1 –1 –1 –1 –1 –1 –1 –1 –1 –1 –1 –1 –1

– –1 –1 –1 –1 –1 ➜ 1 1 1 1 1

Take the negative of –7 Remove the zero pairs


(Flip over the discs inside the box)

➜ –1 –1

\  –7 – (–5) = –7 + 5  Change signs as the discs are flipped.
= –2

Chapter 2  Real Numbers


56
3. (a) Think of two integers ▲ + ■ (including negative integers). Calculate both ▲ – ■ and ■ – ▲.
(b) Repeat (a) with 2 other different integers. Do this two more times.

Suggested answer for (a) and (b):

Consider two integers a and b where a > 0 and b > 0.

Case ▲+■ ■+▲


1 (+a) – (+b) = a + b (+b) – (+a) = b + a
2 (–a) – (–b) = –a + b (–b) – (–a) = –b + a
3 (+a) – (–b) = a + b (–b) – (+a) = –b – a
4 (–a) – (+b) = –a – b (+b) – (–a) = b + a
5 (+a) – 0 = a 0 – (+a) = a
6 (–a) – 0 = –a 0 – (–a) = –a

(c) From (a) and (b), what do you observe?

The results are not the same. The order of subtraction affects the result.

Class Activity 4
Objective: To perform multiplication of integers using algebra discs.

Tasks
1. Multiplying a positive integer by an integer
For example, evaluate 2 × 3 (–5).

–1 –1 –1 –1 –1 –1 –1 –1 –1 –1

–1 –1 –1 –1 –1 –1 –1 –1 –1 –1

Take 2 groups of –5 Ungroup the discs



\ 2 × 3 (–5) = –10   Observe that the answer is negative. Here, we multiply a positive
number by a negative number.

Evaluate the following.

(a) 2 × 4

1 1 1 1 1 1 1 1

1 1 1 1 1 1 1 1

Take 2 groups of 4 Ungroup the discs  

\ 2 × 4 = 8

57
(b) 3 × 6

1 1 1 1 1 1 1 1 1 1 1 1

1 1 1 1 1 1 ➜ 1 1 1 1 1 1

1 1 1 1 1 1 1 1 1 1 1 1

Take 3 groups of 6 Ungroup the discs  


\ 3 × 6 = 18
(c) 3 × (–4)

–1 –1 –1 –1 –1 –1 –1 –1

–1 –1 –1 –1 ➜ –1 –1 –1 –1

–1 –1 –1 –1 –1 –1 –1 –1

Take 3 groups of –4 Ungroup the discs  

\ 3 × (–4) = –12

(d) 4 × (–5)

–1 –1 –1 –1 –1 –1 –1 –1 –1 –1

–1 –1 –1 –1 –1 ➜ –1 –1 –1 –1 –1

–1 –1 –1 –1 –1 –1 –1 –1 –1 –1

Take 4 groups of –5 Ungroup the discs  


\ 4 × (–5) = –20

2. Multiplying a negative integer by an integer


For example, evaluate (–3) × 4.
(–3) × 4 is the same as 4 × (–3), so we can write (–3) × 4 = 4 × (–3).

–1 –1 –1 –1 –1 –1

–1 –1 –1 –1 –1 –1

–1 –1 –1 –1 –1 –1

–1 –1 –1 –1 –1 –1

Take 4 groups of –3 Ungroup the discs

\ (–3) × 4 = 4 × (–3) Observe that the answer is negative. Here, we


= –12 multiply a negative number by a positive number.

Chapter 2  Real Numbers


58
Evaluate the following.

(a) (–1) × 5
(–1) × 5 is the same as 5 × (–1), so we can write (–1) × 5 = 5 × (–1).

–1 –1

–1 –1

–1 –1

–1 –1

–1 –1

Take 5 groups of –1 Ungroup the discs

\ (–1) × 5 = 5 × (–1)
= –5

(b) (–2) × 3
(–2) × 3 is the same as 3 × (–2), so we can write (–2) × 3 = 3 × (–2).

–1 –1 –1 –1

–1 –1 ➜ –1 –1

–1 –1 –1 –1

Take 3 groups of –2 Ungroup the discs

\ (–2) × 3 = 3 × (–2)
= –6

3. Multiplying two negative integers

For example, evaluate (–2) × (–3).



– –1 –1 –1 1 1 1

– –1 –1 –1 1 1 1

Take the negative of 2 groups of –3 Ungroup the discs


(Flip over the discs of each group)

\ (–2) × (–3) = 6   Observe that the answer is positive. Here, we multiply a negative
number by a negative number.

59
Evaluate the following.

(a) (–3) × (–4)



– –1 –1 –1 –1 1 1 1 1

– –1 –1 –1 –1 ➜ 1 1 1 1

– –1 –1 –1 –1 1 1 1 1

Take the negative of 3 groups of –4 Ungroup the discs


(Flip over the discs of each group)

\ (–3) × (–4) = 12

(b) (–4) × (–2)

– –1 –1 1 1

– –1 –1 1 1

– –1 –1 ➜ 1 1

– –1 –1 1 1

Take the negative of 3 groups of –4 Ungroup the discs


(Flip over the discs of each group)

\ (–4) × (–2) = 8 

4. (a) Think of two integers ▲ and ■ (including negative integers). Calculate both ▲ × ■ and ■ × ▲.

(b) Repeat (a) with 2 other different integers. Do this two more times.

Suggested answer for (a) and (b):

Consider two integers a and b where a > 0 and b > 0.

Case ▲+■ ■+▲


1 (+a) × (+b) = ab (+b) × (+a) = ab
2 (–a) × (–b) = ab (–b) × (–a) = ab
3 (+a) × (–b) = –ab (–b) × (+a) = –ab
4 (–a) × (+b) = –ab (+b) × (–a) = –ab
5 (+a) × 0 = 0 0 × (+a) = 0
6 (–a) × 0 = 0 0 × (–a) = 0

(c) From (a) and (b), what do you observe?

The results are always the same. The order of multiplication does not affect the result.

Chapter 2  Real Numbers


60
Class Activity 5
Objective: To find out more about rational numbers and represent them on a number line.

Questions
1. Express the following numbers as decimals. You may use a calculator to get the result.
1 1 . 5 . 4 .
(a) – = –0.5 (b) = 0.3 (c) = 0.83 (d) – = –0.4
2 3 6 9

7 8 .. 3 . . 9
(e) = 0.7 (f ) = 0.72 (g) – = –0.2142857 (h) = 0.36
10 11 14 25

2. State which numbers in Question 1 can be represented as

(a) terminating decimals,


1 7 9
– , ,
2 10 25

(b) recurring decimals.


1 8 5 3 4
– , , , – , –
3 11 6 14 9

3. What can you say about the decimal representation of a rational number?

The decimal representation of a rational number can be either a terminating decimal or a recuming deeimal.

4. Write a decimal which is non-terminating and non-recurring.

–1.14159265358979...

5. Represent the rational numbers in Question 1, the number –1.2 and your number in Question 4 on the following number
line.
1 4 7 1 9 7 8 5
– – –
–1.14159265358979... 2 9 11 3 25 10 11 6

–1 0 1

61
Class Activity 6
Objective: To identify different types of real numbers and represent them on a number line.

Questions
Consider the following numbers:
7 22 6
–3, – 2 , , –�, , , 0, 6 , –2.4
8 7 3

1. Which of the numbers above are integers?


6
–3, , and 0
3

2. Which of the numbers above are rational numbers?


7 22 6
–3, , , , 0 and –2.4
8 7 3

3. Which of the numbers above are irrational numbers?

– 2 , –� and 6

4. Represent all the numbers on a number line.


7 6 22
–� 3 –2.4 – 2 0 8 3 6 7

–3 –2 –1 0 1 2 3

Chapter 2  Real Numbers


62
Try It! (b) (17) + (–9) Since there are more positive ones than
(17) + (–9) negative ones, the answer is positive.
Section 2.1
= 17 – 9
1. An aeroplane is 1320 m above sea level and a submarine = 8
is 56 m below sea level. Represent their altitudes using
positive and negative numbers. (c) (–23) + 12 Since there are more negative ones than
(–23) + 12 positive ones, the answer is positive.
Solution = –23 + 12
Suppose the altitude above sea level is positive. = –11
Altitude of the aeroplane = 1320 m (d) 0 + (+5) Since the sum is zero and positive ones, the
Altitude of the submarine = –56 m 0 + (+5) answer is positive.
= 0 + 5
1 = 5
2. (a) Represent the numbers –1, –3.5 and 2 on a
2
number line.
(b) Arrange the given numbers in descending order. 5. Use algebra discs or AlgeDisc TM application in
AlgeToolsTM to help you do the following. Can you
Solution
predict whether the answer is positive or negative before
(a) The representation of the numbers –1, –3.5 and evaluating it? Show your steps in getting the answers.
1
2 is shown below. (a) 13 – (–21) (b) –9 – (–4)
2
1 (c) 0 – (–6) (d) 7 + (–3)
–3.5 –1 2–2
(e) –4 + (–10) (f) –11 + (–5)
–4 –3 –2 –1 0 1 2 3
Solution
1 (a) 13 – (–21) Since both numbers are positive, the answer
(b) The descending order of the numbers is 2 , –1,
2 = 13 + 21 is positive.
–3.5.
= 34
3. (a) List 3 numbers that are greater than or equal to –27. (b) –9 – (–4) Since there are more negative ones than
Express the relationships using the in equality sign = –9 + 4 positive ones, the answer is negative.
‘’. = –5
(b) List 3 numbers that are less than or equal to –11. (c) 0 – (–6) Since both numbers are positive, the answer
Express the relationships using the inequality sign = 0 + 6 is positive.
‘’. = 6
Solution (d) 7 + (–3) Since there are more positive ones than
(a) Answer depends on numbers chosen. = 7 – 3 negative ones, the answer is positive.
–26  –27 = 4
–2  –27 (e) –4 + (–10) Since both numbers are negative, the answer
0  –27 = –4 – 10 is negative.
(b) Answer depends on numbers chosen. = –14
–15  –11 (f) –11 + (–5)
Since both numbers are negative, the answer
–20  –11 = –11 – 5 is negative.
–31  –11 = –16

Section 2.2 6. In July, the time zone in Sydney is GMT+10 and the
time zone in Vancouver is GMT– 8.
4. For each of the following, can you predict whether the
(a) By how many hours is the local time in Vancouver
answer is positive or negative before evaluating it? Show
behind the local time in Sydney?
your steps in getting the answers.
(b) When it is at 4 p.m. on 3 July in Vancouver, what
(a) (–6) + (–14) (b) 17 + (–9)
is the date and the local time in Sydney?
(d) (–23) + 12 (d) 0 + (+5)
Solution
Solution
(a) The required hours
(a) (–6) + (–14) Since both numbers are negative, the answer
= 10 – (–8)
(–6) + (–14) is negative.
= 18 h
= –6 – 14
= –20

63
(b) By counting forward 18 hours from 4 p.m. on 3 July, (c) (– 8) × (–5) – (–36) ÷ 9 = 40 – (–4)
Sydney is at 10 a.m. on 4 July. = 40 + 4
= 44
18 hours
11. Evaluate the following.
(a) (–6)3 ÷ (–12) + [(–8) – (–3)]2 × (–2).
4 p.m. 8 p.m. 12 a.m. 4 a.m. 8 a.m. 12 p.m. (b) (5 – 9)2 ÷ [(–5) – (–3)]3 – (–7) × (–6) + (–20).
3 July 4 July
Solution
(a) (– 6)3 ÷ (–12) + [(–8) – (–3)]2 × (–2)
7. A submarine descended 23 m. It then ascended 47 m. = (– 6)3 ÷ (–12) + (–5)2 × (–2)
Finally it descended again 35 m. Find the overall change = (–216) ÷ (–12) + 25 × (–2)
of its level. = 18 – 50
= –32
Solution
Overall change of level (b) (5 – 9)2 ÷ [(–5) – (–3)]3 – (–7) × (–6) + (–20)
= –23 + 47 – 35 = (– 4)2 ÷ [(–5) + 3]3 – 42 – 20
= – 11 = 16 ÷ [–2]3 – 42 – 20
= 16 ÷ (–8) – 42 – 20
Section 2.3 = –2 – 42 – 20
= –64
8. Find the values of the following.
(a) 8 × (–12) (b) (–3) × (–6)
(c) (–5) × 6 × (– 4)
Section 2.4
Solution 1 8 5
12. Find the value of 1 + – – – .
(a) 8 × (–12) = –8 × 12 4 3 9
= –96
Solution
(b) (–3) × (–6) = 3 × 6
1 8 5 1 8 5
= 18 1 + – – – = 1 – +
4 3 9 4 3 9
(c) (–5) × 6 × (– 4) = – (5 × 6) × (–4)
5 8 5
= 30 × 4 = – +
4 3 9
= 120
45 – 96 + 20
=
36
9. Find the values of the following. 31
(a) (+16) ÷ (+2) (b) (–75) ÷ (–5) =–
36
(c) (+63) ÷ (–7) (d) (–54) ÷ 6
2 2
Solution 13. Find the value of –6 × .
3 5
(a) ( + 16) ÷ (+2) (b) (–75) ÷ (–5)
16 –75 Solution
=
= –5 2 2 20 2
2
–6 × =– ×
= 8 = 15 3 5
3 5
8
(c) (+63) ÷ (–7) (d)
(–54) ÷ (6) =–
3
–63 –54
= –7 = –6
= –9
= –9 3 1 1
14. Find the value of – ÷ – + – .
4 5 3
10. Find the values of the following. Solution
(a) 32 ÷ (– 4) × (–7)
3 1 1 3 –3 – 5
(b) (–8) – (–2) + (–28) ÷ 4 – ÷ – + – = – ÷
4 5 3 4 15
(c) (–8) × (–5) – (–36) ÷ 9
3 15
=– × –
Solution 4 8
(a) 32 ÷ (– 4) × (–7) = (–8) × (–7) 45
=
= 56 32

(b) (–8) – (–2) + (–28) ÷ 4 = (–8) – (–2) + (–7) =1


13
= –8 + 2 – 7 32
= –13
Chapter 2  Real Numbers
64
3
15. In a play group, of the children are Chinese, are
2 3.8
8 5 7 26.6
Indian and the rest are Malay. Of the Chinese children, 21
2
are boys. Find the fraction of the children who are 56
9
(a) Malay, (b) Chinese girls. 56
00
Solution
0.266 ÷ 0.07 = 3.8
(a) The fraction of Malay children
3 2
=1– – (b) 0.148 ÷ 0.005
8 5 Move the decimal points 3 places to the
9 148
= = right to make 0.005 a whole number.
40 5

(b) The fraction of Chinese girls 29.6


3 2 7 148.0
= × (1 – ) 10
8 9
=
3
×
7 48
8 9 45
7
= 30
24
30

000
16. Find the values of the following.
0.148 ÷ 0.005 = 29.6
(a) 21.357 + 9.24 (b) 16.054 – 8.39

Solution
(a) 21.357 + 9.24 = 30.597 Section 2.5
9
21.357 19. Convert into a decimal.
16
+ 9.240
30.597 Solution
9 ÷ 16 = 0.5625
(b) 16.054 – 8.39 = 7.664
16.054 0.5625
– 8.390 16 9.0000
7.664 80
100
96
17. Find the values of the following. 40
(a) 3.47 × 1.2 32
(b) 2.93 × 0.07 8
8
Solution 0
3.47
(a) 2 decimal places
× 1.2 1 decimal place 7
20. Convert into a decimal.
694 22
347 Solution
4.164 3 decimal places
0.31818
(b) 2.93 × 0.07 = 0.2051 22 7.00000
2.93 2 decimal places 66
× 0.07 2 decimal places 40
0.2051 4 decimal places 22
180
176
18. Find the values of the following. 40
(a) 0.266 ÷ 0.07 (b) 0.148 ÷ 0.005 22
180
Solution 176
(a) 0.266 ÷ 0.07 4
0.266
=
0.07
Move the decimal points 2 places to the
26.6
= right to make 0.07 a whole number.
7

65
Exercise 2.1 Solution
Level 1 (a) 0 4

1. Consider a gain in mass to be positive. Write down the –1 0 1 2 3 4


following changes in mass using positive and negative 04
numbers.
(a) John is 2 kg heavier than before. (b) –2.5 0
(b) Meidi is 3 kg lighter.
–3 –2 –1 0 1
Solution –2.5  0
(a) Change in John’s mass = 2 kg
(b) Change in Meidi’s mass = –3 kg
7. Represent each pair of numbers on a number line and
write down their relation using the ‘’ sign.
2. Consider depositing money into a bank account as
(a) –3, 1 (b) –5, –1
a positive number. Write down the amounts for the
following transactions.
(a) A withdrawal of $2800 Solution
(b) A deposit of $1650 (a) –3 1

– 4 –3 –2 –1 0 1 2
Solution
(a) Amount = –$2800 1  –3
(b) Amount = $1650 (b) 1
–1 –2
–5

–5 –4 –3 –2 –1 0
3. (a) If –4 km/h means 4 km/h below the speed limit,

what does 12 km/h mean?


1
–1  –5
(b) If –3 km denotes a distance of 3 km due South, 2
what is the meaning of +5 km?
8. Fill in the blanks with ‘’ or ‘’.
Solution (a) 2 7 (b) –3 –10
(a) 12 km/h means 12 km/h above the speed limit. (c) 3 –2 (d) –14 27
(e) –5 0 (f ) –11 –6
(b) 5 km denotes a distance of 5 km due North.
Solution
4. If –7 °C denotes a temperature drop of 7 °C, what does (a) 2  7 (b) –3 . –10
5 °C denote? (c) 3 . –2 (d) –14  27
Solution (e) –5  0 (f ) –11  –6
5 °C denotes a temperature rise of 5 °C.

Level 2
5. State the numbers represented by the points A, B and C Describe the meaning of each quantity, in Questions 9 to 11.
on the number line.
9. The adjustment of the hourly wage of a worker is –$5.
A B C
–4 –3 –2 –1 0 1 2 3 4 Solution
The hourly wage is decreased by $5.
Solution
1
The numbers represented by A, B and C are – 4, – and 10. The movement of a lift is positive 2 levels.
2
2 respectively.
Solution
The lift moves up by 2 levels.
6. Represent each pair of numbers on a number line and
write down their relation using the ‘’ sign.
(a) 0, 4 (b) –2.5, 0 11. The change in mass of a lady is –1 kg.

Solution
The lady’s mass has reduced by 1 kg.

Chapter 2  Real Numbers


66
1
12. (a) Represent the numbers –2.7, 1 and –0.4 on a Deposit/
3 Date Balance
number line. Withdrawal
(b) Arrange the given numbers in ascending order. 03 Jan 2012 $3000.00 $3000.00
05 Jan 2012 –$200.00 $2800.00
Solution
(a) The representation is shown below. 11 Jan 2012 $150.00 $2950.00
1
18 Jan 2012 –$400.00 $2550.00
–2.7 –0.4 1 –3
In the table, ‘+’ means deposit and ‘–’ means
–3 –2 –1 0 1 2 withdrawal.

(b) The numbers in ascending order are
–2.7, –0.4, 1 .
1 17. The bank account balances of Rahim, Shimin and Samy
3 are –$3250, $760 and –$2180 respectively. Based on
1
these figures, who has the
13. (a) Represent the numbers 5, –3 and 0.9 on a number (a) most money,
2
line. (b) least money?
(b) Arrange the given numbers in (a) in descending
order. Solution
(a) Shimin has the most money.
Solution
(a) The representation is shown below. (b) Rahim has the least money.

–31–2 0.9 5
18. The table shows the maximum temperatures in four cities
– 4 –3 –2 –1 0 1 2 3 4 5 6 on a certain day.
(b) The numbers in descending order are City Maximum temperature (°C)
5, 0.9,
1
–3 . Beijing –6
2
Hong Kong 12
14. Arrange the numbers –50, 210, 0 and –300 in ascending Singapore 29
order. Tokyo –11
Arrange these cities in the ascending order of their
Solution
temperatures.
The numbers in ascending order are –300, –50, 0, 210.
Solution
15. Arrange the numbers –22, 4, 7 and –9 in descending The required order of the cities is Beijing, Tokyo, Hong
order. Kong, Singapore.

Solution
19. The altitudes of some places are as follows:
The numbers in descending order are 7, 4, –9, –22.
Death Valley, USA: –86 m,
Mount Fuji, Japan: 3376 m,
Mount Faber, Singapore: 105 m,
Level 3
Turfan Depression, China: –154 m.
16. The following shows a page of a savings account Arrange the above places in the ascending order of their
passbook. altitudes.
Date Deposit Withdrawal Balance Solution
03 Jan 2012 $3000.00 $3000.00
–154 < –86 < 105 < 3376
05 Jan 2012 $200.00 $2800.00 The places in the ascending order of their altitude are
11 Jan 2012 $150.00 $2950.00 Turfan Depression, Death Valley, Mount Faber, Mount
18 Jan 2012 $400.00 $2550.00 Fuji.
Design a page that shows the deposits and withdrawals 20. Tan was one of the finalists in the Men’s 400 m
under the same column. Run in the 72nd Singapore Open Track and Field Meet
2010. The results of the other 6 runners in the race
compared to Tan’s are shown below.

67
Hara: 1.74 s faster than Tan Solution
MD: 3.91 s slower than Tan Yes, it is possible to draw a vertical number line as shown
Quek: 7.55 s slower than Tan below.
Sapari: 1.35 s slower than Tan
Sathiyanesan: 0.25 s faster than Tan
Subasinghe: 1.92 s faster than Tan 4
(a) If –1 s means 1 second faster than Tan, represent
the results of the above six runners using positive 3
or negative numbers. 2
(b) Who was the winner of the race?
(c) Who finished last in the race? 1
(d) What was the position of Tan?
0
(e) If Tan’s time was 50.45 s, find the winner’s
time. –1
–2
Solution
–3
(a) Hara: –1.74 s
MD: + 3.91 s –4
Queck: + 7.55 s
Sapari: + 1.35 s
Sathiyanesan: – 0.25 s An example in real life would be a thermometer.
Subasinghe: – 1.92 s
(b) The winner was Subasinghe.
Exercise 2.2
(c) Quek finished last in the race.
Level 1
(d) Tan was slower than Hara, Sathiyanesan and 1. Evaluate the following.
Subasinghe. (a) 3 + 5 (b) (–2) + (–7)
Tan was in the 4th position. (c) (– 4) + 8 (d) 6 + (–11)
(e) The winner’s time (e) 0 + (–2) (f ) (–5) + 17
= 50.45 – 1.92 (g) (–12) + (–25) (h) 24 + (–30)
= 48.53 s
Solution
(a) 3 + 5 = 8
21. Discuss whether the following numbers exist. If they
exist, write down their values. (b) (–2) + (–7) = –(2 + 7)
(a) The largest positive integer = –9
(b) The smallest positive integer (c) (–4) + 8 = 8 – 4
(c) The largest negative integer =4
(d) The smallest negative integer
(d) 6 + (–11) = –(11 – 6)
= –5
Solution
(a) The largest positive integer does not exist. (e) 0 + (–2) = –2
(b) The smallest positive integer exists. It is 1. (f) (–5) + 17 = 17 – 5
= 12
(c) The largest negative integer exists. It is –1.
(g) (–12) + (–25) = –(12 + 25)
(d) The smallest negative integer does not exist.
= – 37
(h) 24 + (–30) = –(30 – 24)
22. Is it possible to draw a vertical number line? If so, draw = –6
a vertical number line. Are there examples in real life
where vertical number lines can be used? Discuss.
2. Evaluate the following.
(a) 13 – 6 (b) (–17) – 8
(c) 9 – (–7) (d) –3 – (–2)
(e) (–5) – (–5) (f ) (–18) – 6
(g) (–9) – (–3) (h) 11 – (–24)

Chapter 2  Real Numbers


68
Solution Solution
(a) 13 – 6 = 7 (a) 7 + (– 4) = 3
(b) (–17) – 8 = –(17 + 8) (b) 11 + (–16) = –5
= –25
(c) –9 – (4) = –13
(c) 9 – (– 7) = 9 + 7
(d) –8 – (–18) = 10
= 16
(e) –8 – (14) = 6
(d) – 3 – (– 2) = –3 + 2
= –1 (f) 4 – (–3) = 7
(e) (–5) – (–5) = (–5) + 5
=0
Level 3
(f) (–18) – 6 = – (18 + 6) 5. A helicopter flying 120 m above sea level detects a
= –24 submarine. If the submarine is 39 m below sea level,
(g) –9 – (–3) = – 9 + 3 what is the vertical distance between the helicopter and
= –6 the submarine?

(h) 11 – (–24) = 11 + 24 Solution


= 35 The required vertical distance = [120 – (–39)]
= 159 m

Level 2
6. The bank account of a company has overdraft facility. Its
3. Evaluate the following. balance for yesterday was –$390. The company deposits
(a) 3 – (–8) + (– 4) $600 into the account today and it will withdraw $450
(b) (–2) + 5 + (–3) tomorrow. Find the balance in the account
(c) (–9) + (–2) – (–7) (a) today, (b) tomorrow.
(d) 4 – (–2) – 4
(e) 7 + (–12) – 6 Solution
(f ) –19 – (–15) + 10 (a) Today’s balance = –$390 + $600 = $210

Solution (b) Tomorrow’s balance = $210 – $450 = –$240


(a) 3 – (–8) + (– 4) = 3 + 8 – 4
=7 7. A car travels 16 km due south, 33 km due north and
then 12 km due south. What is its final position from
(b) (–2) + 5 + (–3) = –2 + 5 – 3
the starting point?
=0
(c) (–9) + (–2) – (–7) = –9 – 2 + 7 Solution
= – 4 Suppose the movement due north is positive.
Final position = [(–16 + 33 + (–12)]
(d) 4 – (–2) – 4 = 4 + 2 – 4
= (–16 + 33 – 12)
=2
= 5 km
(e) 7 + (–12) – 6 = 7 – 12 – 6 Its final position is 5 km north from the starting point.
= –11
(f ) –19 – (–15) + 10 = –19 + 15 + 10 8. The following table records the maximum and minimum
=6 daily temperatures of a city on 5 consecutive days.
Maximum Minimum
Day
4. Find the missing numbers. temperature (°C) temperature (°C)
(a) 7 + ( ) = 3 Mon –2 –10
(b) 11 + ( ) = –5 Tue 6 –4
(c) –9 – ( ) = –13
Wed 0 –3
(d) –8 – ( ) = 10
(e) –8 + ( ) = 6 Thu –1 –5
(f) 4 – ( )=7 Fri 10 3
(a) Which day has the greatest temperature difference?
(b) Which day has the least temperature difference?

69
Solution (c) Check the results of your classmates. What can you
conclude?
Day Temperature difference (°C)
Mon –2 – (–10) = 8 Solution
Tue 6 – (– 4) = 10    (a) Left side Right side
Wed 0 – (–3) = 3 3 + (–5) (–5) + 3
Thu –1 – (–5) = 4 [3 + (–5)] + 6 3 + [(–5) + 6]
Fri 10 – 3 = 7 3 – (–5) (–5) – 3
(a) Tuesday has the greatest temperature difference. [3 – (–5)] – 6 3 – [(–5) – 6]
(b) Wednesday has the least temperature difference. (b) The results are as follows:

9. A lift, initially at the 11th floor, goes up 7 floors and then Left side Right side
goes down 10 floors. Find the level it finally stops at. –2 –2
4 4
Solution
The required level = 11 + 7 + (–10) 8 –8
= 8th floor 2 14
The values on the left side is equal to those on the
10. Ali has been running a small business for 4 years. The right side for the first two rows.
following table shows his profits (positive) and losses Left side = –Right side for the third row.
(negative) in these years. Left side  Right side for the 4th row.
Year Profit/Loss (c) For any three integers a, b and c,
1 –$23 000 (i) a + b = b + a
(ii) (a + b) + c = a + (b + c)
2 –$6000
(iii) a – b = –(b – a)
3 –$9000 (iv) (a – b) + c  a – (b – c)
4 –$17 000
(a) Find his total profit or loss in these 4 years.
(b) If his targeted total profit is $18 000 in the first 5 Exercise 2.3
years, what should be his profit in the 5th year? Level 1
1. Evaluate the following.
Solution (a) 8 × (–9) (b) (–5) × (– 4)
(a) Total (c) (–6) × 7 (d) 3 × 17
= (–$23 000) + (–$6000) + $9000 + $17 000 ( e ) (–12)2 ( f) (– 4)3
= –$3000 (g) (–3)2 × 5 (h) (–2)3 × (–9)
His total loss is $3000.
(b) $18 000 – (–$3000) = $21 000 Solution
His profit should be $21 000. (a) –72 (b) 20
(c) – 42 (d) 51
11. (a) Fill the same shapes with the same integers in the (e) 144 (f) –64
following table.
(g) 45 (h) 72
Left side Right side
2. Evaluate the following.
+ +
(a) (–38) ÷ (–2) (b) 132 ÷ (–11)

( + )+ + ( + ) (c) 65 ÷ 5
0
(d) (–57) ÷ 3

– – (e) (f) (–144) ÷ (–9)


–7
3 162
(g) (– 6) (h)
(–3)3
( – )– – ( – ) Solution
(b) Evaluate each expression and compare the results (a) 19 (b) –12
on the left side with those on the right side. What
(c) 13 (d) –19
do you notice?

Chapter 2  Real Numbers


70
(e) 0 (f) 16 (b) Megan’s mass after 3 months
= 65 + (–6)
(g) –216 (h) –6
= 59 kg

Level 2 5. A shop sells 4 mobile phones at a loss. If the loss per


phone is $40, what is the total loss incurred?
3. Evaluate the following.
(a) (–6) × 3 × (–1)
Solution
(b) (–84) ÷ 7 × 5
Total loss = $40 × 4
(c) (–37) × 0 – (–8)
= $160
(d) 63 ÷ (–9) + (–2) × (–10)
(e) (– 47) – 33 ÷ (–3) – 3 × (–7)
(f ) 196 ÷ [(–8) + (–6)] × (–2)
  
6. The speed of a car decreases steadily from 24 m/s to 9
(g) [(–23) + 14] × (–2)2 m/s in 5 seconds. Find the change in speed per second.
(h) (– 45) × 6 ÷ (–3)3 Note: In physics, the rate of change of speed over time
( i ) (–7) × (–8) × 0 × 34 – 4 × (–5)2 is called acceleration.
( j) (–6)3 ÷ 32 ÷ [–9 – (–8)]3
Solution
Solution Change in the speed per second
(a) (–6) × 3 × (–1) = (–18) × (–1) = (9 – 24) ÷ 5
= 18 = – 3 m/s2
(b) (–84) ÷ 7 × 5 = (–12) × 5
= –60
Level 3
(c) (–37) × 0 – (–8) = 0 – (–8)
7. Tom borrowed a sum of money from his father 7 months
=8
ago. He returned $400 per month to his father. Currently
(d) 63 ÷ (–9) + (–2) × (–10) = (–7) + 20 he owes his father $2500.
= 13 (a) How much will he owe his father 3 months from
now?
(e) (– 47) – 33 ÷ (–3) – 3 × (–7)
(b) How much money did he borrow from his father?
= (– 47) – (–11) + 21
= – 47 + 11 + 21 = –15
Solution
(f ) 196 ÷ [(–8) + (–6)] × (–2) = 196 ÷ (–14) × (–2) (a) Amount owed = $(2500 – 400 × 3) = $1300
= (–14) × (–2)
(b) Amount borrowed = $[2500 + (– 400) × (–7)]
= 28
= $(2500 + 2800)
(g) [(–23) + 14] × (–2)2 = (–9) × (–2)2 = $5300
= (–9) × 4
= –36
8. The volume of water in a tank is decreasing at the rate
(h) (– 45) × 6 ÷ (–3)3 = (– 45) × 6 ÷ (–27) of 3 litres per minute. At 9:00 a.m., the volume of water
= (–270) ÷ (–27) is 120 litres. Find the volume of water at the following
= 10 times.
(a) 9:30 a.m.
(i) (–7) × (–8) × 0 × 34 – 4 × (–5)2 = 0 – 4 × 25
(b) 8:50 a.m.
= –100
( j) (–6)3 ÷ 32 ÷ [–9 – (–8)]3 = (–6)3 ÷ 32 ÷ (–1)3 Solution
= (–216) ÷ 9 ÷ (–1) (a) Time from 9:00 a.m. to 9:30 a.m.
= 24 = 30 minutes
Volume of water
4. After joining a fitness programme, Megan loses her mass = 120 – 3 × 30
by 2 kg every month. = 30 litres
(a) Find the change in her mass in 3 months. (b) Time from 9:00 a.m. to 8:50 a.m.
(b) If Megan’s original mass was 65 kg, find her mass = –10 minutes
after 3 months.
Volume of water
= 120 – 3 × (–10)
Solution
= 150 litres
(a) Change in mass = (–2) × 3
= –6 kg

71
9. Mr Lim has 400 shares of Stock A and 500 shares of (b) The minimum score = (–2) × 5 = –10 marks
Stock B. He gains $3 per share from Stock A and loses
(c) The score is 3 marks if there is 1 correct answer
$2 per share from Stock B. How much does he gain or
and 4 unanswered questions.
lose from these two stocks taken together?
(d) The score is –3 marks if there are 3 incorrect
Solution answers, 1 correct answer and 1 unanswered
The net gain = $[400 × 3 + 500 × (–2)] question.
= $(1200 – 1000)
= $200
13. (a) Fill the same shapes with the same integers in the
following table.
10. Jamilah jogs along a road. Starting from a bus station,
Left side Right side
she jogs at 4 m/s due north for 20 minutes, then at
5 m/s due south for 15 minutes and finally at 3 m/s due × ×
south for 10 minutes. Find her final position from the
station. ( × )× × ( × )
Solution ÷ ÷
Suppose the movement due north is positive.
Final position ( ÷ )– ÷ ( ÷ )
= [4 × 20 × 60 + (–5) × 15 × 60 + (–3) × 10 × 60]
= (4800 – 4500 – 1800) (b) Evaluate each expression and compare the results
= –1500 m on the left side with those on the right side. What
Her final position is 1500 m south from the station. do you notice?
(c) Check the results of your classmates. What can you
conclude?
11. The highest point of the Bukit Timah Hill (BTH) is about
164 m above ground level.
Solution
When completed in 2013, the Jurong Rock Cavern
(JRC) will be the first underground oil storage facility    (a) Left side Right side
in Southeast Asia. The base of the JRC is 132 m below
ground level. 4 × (–12) (–12) × 4
(a) Find the vertical distance between the top of the [4 × (–12)] × 6 4 × [(–12) × 6]
BTH and the base of the JRC.
(b) How high is an office building above ground level 4 ÷ (–12) (–12) ÷ 4
if its top is vertically midway between the top of [4 ÷ (–12)] ÷ 6 4 ÷ [(–12) ÷ 6]
the BTH and the base of the JRC?

(b) The results are as follows.
Solution
(a) The required distance Left side Right side
= 164 – (–132) – 48 – 48
= 296 m
–288 –288
(b) The required level 1
1 – –3
= –132 + × 296 3
2
= 16 m 1
– –2
18

12. A mathematics quiz consists of 5 ‘true/false’ questions. Left side = Right side for the first two rows.
The marking scheme is as follows: 3 marks for every
correct answer, –2 marks for every incorrect answer, 0 1
Left side = for the third row.
Right side
mark for every question not answered.
(a) What is the maximum score of the quiz? Left side  Right side for the fourth row.
(b) What is the minimum score of the quiz? (c) For any three integers a, b and c,
(c) Write about a situation where a student scores (i) a × b = b × a
3 marks for the quiz. (ii) (a × b) × c = a × (b × c)
(d) When would a a student score –3 marks for the quiz? 1
(iii) a ÷ b = provided a  0 and b  0.
b÷a
Solution (iv) (a ÷ b) ÷ c  a ÷ (b ÷ c) provided b  0 and
(a) The maximum score = 3 × 5 = 15 marks c  0.

Chapter 2  Real Numbers


72
Exercise 2.4 (g)
9 3
× 1 × 3 =
1 9
×
8
×
10
Level 1 32 5 3 32 5 3
3
1. Evaluate the following, giving your answers in the =
2
simplest form. 1
1 2 3 1 =1
(a) + (b) – + – 2
5 7 4 2
–4 3
3 11 5 6 8 35 21
(c) 2 – 3 (d) –1 – – + – (h) =– ÷
4 12 6 5 51 8 4
4
35 4
24 45 7
2
7 =– ×
(e) – × (f ) – – 8 21
25 16 11 11 5
=–
–4 3 6
9 3 1 8
(g) ×1 ×3 (h)
32 5 3 51 3 1 8 3
4 (i) –1 ÷ –1 = ÷
5 3 5 4

3 1 1 1 8 3
(i) –1 ÷ –1 (j) 3 + 2 ÷ –1 = ×
5 3 2 4 5 4
6
=
5
Solution
1
1 2 7+2×5 17 =1
(a) + = = 5
5 7 35 35
3 1 3 1 1 1 5 5
(b) – + – =– + (j) 3 + 2 ÷ –1 =3+ ÷ –
4 2 4 2 2 4 2 4

5 4
=3– ×
3+2 2 5
=–
4
=3–2
5
= – =1
4
1
= –1
4
2. Find the values of the following.
3 11 11 47 (a) 32.7 + 9.84 (b) 25.02 – 9.765
(c) 2 – 3 = –
4 12 4 12 (c) 11.47 × 2.8 (d) 3.45 × 1.06
11 × 3 – 47 (e) 10.68 ÷ 0.6 (f) 0.1935 ÷ 0.009
=
4
–14 Solution
=
12 (a) 42.54 (b) 15.255
1
= –1 (c) 32.116 (d) 3.657
6

5 6 5 6
(e) 17.8 (f) 21.5
(d) –1 – – + – = –1 + –
6 5 6 5

–30 + 25 – 36
= Level 2
30
–41 3. Evaluate the following, giving your answers in the
= simplest form.
30
11
= –1 (a) – + +
1 1 1
× –
2
30
2 3 6 5
24 45 24 45
(e) – × = – × 5 1 1 1
25 16 25 16 (b) 1 × – ÷ –1
7 4 9 4
27
=–
10
2 9 7 2
7 (c) – × – + – ×
= –2 3 8 4 21
10
2 1 1 1 1
(f ) –
7

7
=
49

7 (d) – – – ÷ – –
4 6 3 2
11 11 121 11
49 – 77 1 2
3
1 1
= (e) –2 – – × –2
121 2 3 4 4
28

=–
121

73
1 1 1 2 1 1 1 2
(f) 1 – –8 × –1 ÷ –1 (f) 1 – –8 × –1 ÷ –1
2 3 5 15 2 3 5 15

3 25 6 17
Solution = – – × – ÷ –
2 3 5 15
1 1 1 2 –3 + 2 + 1 2
(a) – + + × – = × – 3 17
2 3 6 5 6 5 = – 10 ÷ –
2 15
2
=0× – 17 15
5 = – × –
2 17
=0
15
=
2
5 1 1 1 12 9–4 5 1
(b) 1 × – ÷ –1 = × ÷ – =7
7 4 9 4 7 36 4 2

12 5 4
=– × × 1
7 36 5
4. Of the Secondary 1 students in a school, of them live
5
4 4
= – in Clementi and of them live in Queenstown. Find the
21 9
fraction of the students who neither live in Clementi nor
2 9 7 2 Queenstown.
(c) – × – + – ×
3 8 4 21

2 9 7 2 Solution
= × – × 1 4
3 8 4 21 The required fraction = 1 – –
5 9
3 1 45 – 9 – 20
= – =
4 6 45
16
=
9–2 =
45
12

7 2 1
= 5. In a villa, of the land is garden and of the garden is
12 5 3
a swimming pool. Find the fraction of the land which is
1 1 1 1 occupied by the swimming pool.
(d) – – – ÷ – –
4 6 3 2
Solution
1 1 2–3 2 1
= – + ÷ The required fraction = ×
4 6 6 5 3
–3 + 2 2–3 2
= ÷ =
12 6 15

–1 1 5
= ÷ – 6. Jim spent of his savings to buy a pair of sports shoes.
12 6 8
1 1
= ×6 He donated of the remainder to the Red Cross Society.
12 3
1 Find the fraction of his savings which was donated.
=
2
Solution
3 5 1
1 2 1 1 The required fraction = 1 – ×
(e) –2 – – × –2 8 3
2 3 4 4
3 1
= ×
5 8 1 9 8 3
= – – – × – 1
2 27 4 4
=
8
5 5 9
= – – × –
2 108 4
5 5 Level 3
=– +
2 48 7. Alex, Boris and Carol invested in a business together.
–115 1
= Based on the amount each invested, Alex owns of the
48 3
2
19 business and Boris owns .
= –2 7
48

Chapter 2  Real Numbers


74
(a) What fraction of the business does Carol own? (b) Time difference = 25 – 24
(b) Find the total investment in the business if Alex = 1 min
has invested $6000 more than Boris.
(c) LCM of 20, 24 and 25 = 600
They all meet again at the starting point after 600
Solution
minutes.
(a) The fraction that Carol owns 600
1 2 Number of loops Huili completed =
= 1– × 20
3 7
= 30
21 – 7 – 6
=
21 600
8 Number of loops Jenny completed =
= 24
21
= 25
(b) Fraction that Alex owns more than Boris 600
1 2 Number of loops Rohana completed =
= – 25
3 7
= 24
7–6
=
21
3 9
=
1 10. Consider the rational numbers and .
4 11
21
(a) Which number is larger?
(b) Find two rational numbers between them.
3
8. There are two identical jugs, A and B. Jug A is full of (c) Compare your answers in (b) with those of your
7
8 classmates.
water and Jug B is full. What fraction of the capacity
11 (d) Can you suggest how many rational numbers lie
of a jug of water should be poured from B to A so that 3 9
between and ? Give a reason for your answer.
they both have the same volume of water? 4 11

Solution Solution
1 8 3 1 56 – 33 3 3 × 11 33
– = (a) = =
2 11 7 3 77 4 4 × 11 44
1 23 9 9×4 36
= = =
2 77 11 11 × 4 44
23
= 9
154 is larger.
11
8 23 89
– =
11 154 154 17 34 35 3
89 (b) = and are two numbers between and
22 44 44 4
\  of the capacity in Jug B should be poured into
154 9
Jug A. .
11
a+c a
In fact, is a rational number between and
b+d b
9. Huili, Jenny and Rohana walk around a circular c
1 2 5 .
track. They take h, h and h to complete a loop d
3 5 12
respectively. 3×9 12 4
i.e.  = = is also a rational number
(a) Express the time taken by each of them to walk 4 × 11 15 5
one lap of the track in minutes. 3 9
between and .
(b) Find the difference in time taken by Jenny and 4 11
Rohana to walk one lap of the track, giving your
(c) The answers from different students may be
answer in minutes.
different.
(c) How many laps will each girl have to walk before
all three of them meet again at the starting point?
(d) There are an infinite number of rational numbers
3 9
Solution between and .
4 11
1
(a) Time taken by Huili = × 60 min = 20 min This is because, as illustrated in (b), there is a
3
2 rational number between any two rational numbers.
Time taken by Jenny = × 60 min = 24 min
5
5
Time taken by Rohana = × 60 min = 25 min
12

75
3
Exercise 2.5 (m) +
4
1.69 = 0.512 × 1.3
5
Level 1 = 0.6656
1. Express the following rational numbers as decimals.
2 1 2 9
3 2 ( n) – 4 + – ×1 = – 4 + – ×
(a) (b) –1 3 8 3 8
4 5
3
(c)
2
(d) –
13 = – 4 –
4
9 11
3
7 17 = – 4
(e) – (f)
  – 4
12 20
= – 4.75

Solution
3 2
(a) = 0.75 (b) –1 = –1.4 Level 2
4 5
2 13 3. Evaluate the following and express the answers in
(c) = 0.2 (d) – = –1.18 decimals.
9 11
7 17 2 1 3 1
(e) – = –0.583 (f ) – = –0.85 (a)
5
+ –
3
(b) – 4 – –5
4 2
12 20

2. Evaluate the following using a calculator. Solution


(a) 292 (b) 153 2 1 2 1
(a) + – = –
5 3 5 3
(c) 6 5
(d) 15 129
6–5
(e) 59 049 (f  ) 141.61 =
15
3 3
(g) 22 + 8 (h) 46.656 =
1
2
15
(i) –11 × [–5 + (–4)] (j) (–7) × (–8) ÷ (–5)
= 0.06
(k) [1 + 2 × (–3)] ÷ [1 – 2 × (–3)]
3 3 1 3 1
(l)
3
+
1
(m)
4
+ 1.69 (b) – 4 – –5 = – 4 + 5
4 2 4 2
4 6 5
2 1 19 11
(n) – 4 + – × 1 =– +
3 8 4 2

–19 + 22
Solution
=
4
(a) 292 = 841 = 0.75
3
(b) 15 = 3375
(c) 65 = 7776 4. Evaluate the following using a calculator.
2
3 1
(d) 15 129 = 123 (a) 372 – 352 (b) 2005 – –3
7
(e) 59 049 = 243 3 1
3
3.21 + 2
(c) –5 – –6 (d) 5
(f ) 141.61 = 11.9 4 2
4.7 – 2.2
3
(g) 22 + 8 = 3.107 232 506 (e) –1 × 0.5 –
1 2
÷2
1
3 9 3 3
(h) 46.656 = 3.6
2
1 1 1 3
(i) –11 × [–5 + (–4)] = –11 × (–9) (f  ) –3 + 1 + –2 × –
8 6 12 2
= 99
Solution
(j) (–7) × (–8) ÷ (–5)2 = 56 ÷ 25
=2
6 (a) 372 – 352 = 12
25 2
3 1
(k) [1 + 2 × (–3)] ÷ [1 – 2 × (–3)] = (–5) ÷ 7 (b) 2005 – –3 = 2.732 150 08
7
5
= – 3
7 3 1 27
(c) –5 – –6 = 0.421 875 or
4 2 64
= –0.714 285
3 1 11
(l) + = = 0.916
4 6 12

Chapter 2  Real Numbers


76
3.21 + 2 Solution
19
(d) 5 = 0.76 or Let a, b, c, d, e, f, g, h and i represent the missing values
25
4.7 – 2.2 in the table.
1 2 1 + a b c
(e) –1 × 0.5 – ÷ 2 = 0.079 365 079
9 3 3
2 d e f
1 1 1 3
2 g 3 – 4 –7
(f ) –3 + 1 + –2 × – = –9.093 75
8 6 12 2 –3 h i –5
3
= –9
32
–3 + c = –5
  c = –2
5. Weimin thinks that 4.9 = 5. Do you agree or disagree
with him. Why? g + c = –7
g + (–2) = –7
Solution g = –5
1
4 = 4.3 a + g = 3
3

a + (–5) = 3
2
= 0.6 a = 8
3
1 2 b + g = – 4
∴ 4 3 + 3 = 4.9

b + (–5) = – 4
Based on the above working, Weimin is correct. b = 1
d = 2 + a
Revision Exercise 2 =2+8
1. The world is divided into 24 standard time zones. Relative = 10
to Singapore in April, Sydney’s time is 2 hours, London’s e = 2 + b
time is –7 hours and New York’s time is –12 hours. If it =2+1
is Tuesday, 12 April, 9 a.m. in Singapore, find the local =3
time in
(a) Sydney, f = 2 + c
(b) London, = 2 + (–2)
(c) New York. =0
h = –3 + a
Solution = –3 + 8
(a) 9 + 2 = 11 =5
The local time in Sydney is Tuesday, 12 April,
11 a.m. i = –3 + b
= –3 + 1
(b) 9 – 8 = 1 = –2
The local time in London is Tuesday, 12 April,
1 a.m. The values in the table are as follows:
(c) 9 – 12 = –3 + 8 1 –2
The local time in New York is Monday, 11 April, 2 10 3 0
9 p.m.
–5 3 – 4 –7
–3 5 –2 –5
2. Complete the following addition table.

3. Complete the following multiplication table.


+
× –3
2
15
3 – 4 –7
–2 8
–3 –5
7 –42

77
Solution (b)
()
2
7

6
Let a, b, c, d, e, f, g, h and i represent the missing values
in the table. 7–
–2 6 (–2)2
× –3 a b
–3 –2 –1 0 1 2 3 4 5 6
c 15 d e 7
(c) –2 
–2 f 8 g 6
7 h i –42 7
2
(d)  (–2)2
6
(–2) × a = 8
a = –  4
5. Liquid air consists of a mixture of liquid nitrogen and
7 × b = –  42
liquid oxygen. The boiling point of nitrogen is –196 °C
b = – 6
and that of oxygen is –183 °C. When liquid air heats
c × (–3) = 15
up, which gas will evaporate first? Explain your answer
c = –5 briefly.
Note: This property is used in industry for the production
d = c × a
of oxygen using the method called fractional
= (–5) × (–4)
distillation.
= 20
e = c × b Solution
= (–5) × (–6) –196 < –183
= 30 When liquid air heats up, it will reach the temperature
–196 °C first. Hence nitrogen will evaporate first.
f = (–2) × (–3)
=6
6. (a) A jet fighter which is 324 m above sea level
g = (–2) × b
fires a missile to hit a submarine which is
= (–2) × (– 6)
58 m below sea level. Find the vertical distance
= 12
travelled by the missile.
h = 7 × (–3) (b) If a bird is midway between the jet fighter
= –21 and the submarine, how high is the bird above
i = 7 × a sea level?
= 7 × (– 4)
Solution
= –28
(a) Vertical distance travelled
The values in the table are as follows: = 324 – (–58)
= 382 m
× –13 –4 –6
(b) Height of the bird
–5 15 20 30 1
= –58 + × 382
2
–2 6 8 12
= 133 m
7 –21 –28 –42
7. The temperatures in some cities at noon on a particular
7
2
day are as follows:
4. (a) Find the value of (–2)2 and .
6
2
Beijing –7 °C
7 7
(b) Represent the numbers –2, , (–2)2 and on a Hong Kong 16 °C
6 6
number line. Singapore 28 °C
7
(c) Write down an inequality connecting –2 and . Stockholm T °C
6
(d) Write down an inequality connecting (–2)2 and Toronto –2 °C
2
7
6 . (a) Find the difference between the temperatures in
Singapore and Toronto.
Solution (b) The temperature in Stockholm is 26 °C lower than
(a) (–2)2 = 4 that in Hong Kong.
2 (i) Find the temperature in Stockholm.
7 49
=
6 36

Chapter 2  Real Numbers


78
(ii) Determine whether the temperature in Beijing Solution
is higher or lower than that in Stockholm. (a) –5  –3  3  10
What is the difference in their temperatures? \  Ali had the lowest score in Round 1.
(b) Ali’s total score = (–5) + (–1) + (–2)
Solution
= –8
(a) The required difference
= 28 – (– 2) Ben’s total score = (+3) + (–2) + (–1)
= 30 °C =0
(b) (i) Temperature in Stockholm Chetan’s total score = (+10) + (+2) + (–3)
= 16 – 26 =9
= –10 °C
Dave’s total score = (–3) + (– 4) + (+5)
(ii) –10 < –7 = –2
and – 7 – (–10) = 3
(c) The winner was Ali.
The temperature in Beijing is higher than that in
Stockholm. Their difference is 3 °C.
10. Evaluate the following.
(a) (–16) × (–3) – (–8) × 5
8. A quality control supervisor measures the actual volumes (b) [–2 + (–7)]3
of 6 packets of fruit juice. Each packet of fruit juice is 1 1 4
supposed to contain 375 ml of juice. The table shows (c) 3 × –5 ÷ 1
2 7 5
the inspection results. 2
2 1 1
(d) × –
Amount below 3 4 8
or above the
–5 +12 –6 –9 +7 –2 Solution
required
volume (ml) (a) (–16) × (–3) – (–8) × 5 = 48 + 40
Packet 1 2 3 4 5 6 = 88

(a) Find the actual volumes of juice in packet 1 and (b) [–2 + (–7)]3 = (–9)3
packet 2. = –729
(b) Find the total volume of juice in the 6 packets.
1 1 4 7 36 9
(c) 3 × –5 ÷ 1 =– × ÷
Solution 2 7 5 2 7 5
(a) Volume of juice in Packet 1 = (375 – 5) 5
= –18 ×
= 370 ml 9
= –10
Volume of juice in Packet 2 = (375 + 12)
= 387 ml 2 2
2 1 1 2 1
(b) (–5) + (+12) + (–6) + (–9) + (+7) + (–2) (d) × – = ×
3 4 8 3 8
= –5 + 12 – 6 – 9 + 7 – 2
2 1
= –3 = ×
3 64
Total volume of juice = [375 × 6 + (–3)]
1
= 2247 ml =
96

9. Four persons participate in a golf tournament. Their


11. A standard triathlon is made up of a swim, followed by
scores (which are the numbers of strokes below or above
a bicycle ride, followed by a run. The running distance
a standard value) in 3 rounds are shown in the following
20
table. is of the whole journey, the cycling distance is 4
103
Ali Ben Chetan Dave times of the running distance and the swimming distance
Round 1 –5 +3 +10 –3 is 1.5 km. Find
Round 2 –1 –2 +2 –4 (a) the fraction of the race which a competitor has to
swim,
Round 3 –2 –1 –3 +5
(b) the distance covered in the race.
(a) Who had the lowest score in Round 1?
(b) Find the total score of each person in 3 rounds.
(c) The winner in the tournament was the person with
the lowest total score. Who was the winner?

79
Solution
(a) The required fraction
20 20
=1– –4×
103 103
103 – 20 – 80
=
103
3
=
103

(b) The distance covered in the race


3
= 1.5 ÷
103
103
= 1.5 ÷
3
= 51.5 km

3
12. On a shelf of science books, of the books are biology,
3 8
of the remaining books are chemistry and the rest are
8
physics.
(a) Find the fraction of physics books on the shelf.
(b) Express the fraction of physics books as a decimal.
(c) If there are 10 physics books, find the total number
of books on the shelf.

Solution
3 5
(a) 1 – =
8 8
5
of the books are physics books.
8

5
(b) = 0.625
8

(c) Total number of books


5
= 10 ÷
8
= 16

Chapter 2  Real Numbers


80
3 Approximation and
Estimation

Class Activity 1
Objective: To identify the use of significant digits in real-life situations.

Questions
1. The exact thickness of a piece of glass is 0.004 503 m.

(a) State the thickness of the piece of glass in metres correct to 2 decimal places.

0.00 m (correct to 2 d.p.)

(b) If you give the rounded off figure in (a) to a handyman, will it make any sense to him?

No, it will not.

(c) What could be a meaningful estimation of the thickness that can be given to the handyman?

0.004 m

2. Mr Tan bought an apartment for $508 175.62.

(a) If his friend asked him about the price of the apartment, what would he tell his friend?

$508 000 (correct to the nearest 1000)

(b) Share and discuss your answer in (a) with your classmates. Explain why you have chosen the answer.

It is not necessary to give the exact amount, a rounded off price is sufficient.

3. Find the land area of Singapore. Is your answer an exact value or a rounded off value?

714 km2. It is a rounded-off value.

81
Class Activity 2
Objective: To identify the use of significant digits in real-life situations.

Questions
1. Mrs Fu wants to buy a can of milk powder, a pack of apples and a piece of fish fillet at a supermarket. She has only $20
in her purse.

a can of milk powder a pack of apples a piece of fish fillet


$12.40 $2.99 $3.50

(a) How would you help her to estimate, without using calculator, whether she has enough money for these three items?

$12.40 + $2.99 + $3.50


≈ $12 + $3 + $4  (correct to the nearest dollar)
= $19

She has enough money based on the estimation.

(b) Explain your method briefly in the estimation above.

Each price is rounded off to the nearest dollar and then added up.

2. The monthly salaries of five employees in a company are $3150, $2980, $3040, $2890 and $2950.

(a) Estimate the total monthly salary of these employees without using a calculator.

$3000 + $3000 + $3000 + $3000 + $3000


= $15 000  (correct to the nearest thousand)

Estimate total monthly salary is $15 000.

(b) Explain your method briefly in the estimation above.

Each salary is rounded off to the nearest thousand before adding.

Chapter 3  Approximation and Estimation


82
Class Activity 3
Objective: To apply estimation strategies for measurements.

Questions
1. (a) Simply by looking at the line segment AB, estimate its length.

A B

The actual length of line segment AB is 11 cm.

(b) (i) Suggest a way of estimating the length of AB using a $1 coin.

The length of AB can be estimated by determining how many $1 coins can fit into the line segment AB.

(ii) What is your new estimate of the length of AB?

The actual length of line segment AB is 11 cm.

(c) Measure the length of AB using a ruler.

length of AB = 11 cm

(d) Compare your estimates in (a) and (b).

The estimates in (b) is closer to the actual value of 11 cm than the estimate in (a).

2. Suggest a way of estimating the width of your classroom using one of your body parts and share your method with your
classmates.

Use the foot to walk the width of the classroom. The number of steps taken multiplied by the length of the foot will give an estimation of the width.

3. One of the HDB blocks in Bishan has 50 storeys. Suggest a way to estimate the height of the block.

The height of the block can be estimated by estimating the height of one storey and multiplying by 50. The height of one storey can be estimated

by having a person standing in front of the block and determine how many persons can fit into one storey.

83
Class Activity 4
Objective: To compare follow-through errors arising from intermediate values that are rounded off to different degrees of accuracy.

Questions
Alif, Balan, Cai Ling and Danny went to a supermarket to buy chicken fillets and salmon fillets for their friend’s
farewell party. They bought a pack of chicken fillets and a pack of salmon fillets which weigh 0.543 kg and
0.376 kg respectively.


1 kg of chicken fillets 1 kg of salmon fillets
$8.75 $23.45

They were not sure if they were within the budget of what they had set aside for this purchase. Thus, each of them made a
quick estimate as follows. Use a table to help you organise your workings and answers as you work through Questions 1–3.

1. Alif rounded off each number to 1 decimal place and then worked out the estimate. What was his estimated value?

Solution
$8.75 × 0.543 + $23.45 × 0.376
≈ $8.8 × 0.5 + $23.5 × 0.4 Round off each value to 1 d.p.
= $4.4 + $9.4
= $13.8 (correct to 1 d.p.)

His estimated value was $13.8.

2. Balan rounded off each number to 1 significant figure and then worked out the estimate. What was his estimated value?

Solution
$8.75 × 0.543 + $23.45 × 0.376
≈ $9 × 0.5 + $20 × 0.4 Round off each value to 1 s.f.
= $4.5 + $8
= $10 (correct to 1 s.f.)

His estimated value was $10.

3. Cai Ling rounded off each number to 2 significant figures and then worked out the estimate. What was her estimated
value?

Cai Ling calculated in the following way:


Price of chicken fillets = $8.75 3 0.543
≈ $8.8 3 0.54 Round off each number correct to 2 s.f.
= $4.752
= $4.8 (correct to 2 s.f.)
Price of salmon fillets = $23.45 3 0.376
≈ $23 3 0.38 Round off each number correct to 2 s.f.
= $8.74
= $8.7 (correct to 2 s.f.)
Total price ≈ $4.8 + $8.7
= $13.5 (correct to 3 s.f.)

Chapter 3  Approximation and Estimation


84
Solution
$8.75 × 0.543 + $23.45 × 0.376
≈ $8.8 × 0.54 + $23 × 0.38 Round off each value to 2 s.f.
≈ $4.75 + $8.74
= $13 (Correct to 2 s.f.)

Her estimated value was $13.

4. Calculate the exact total price of the items and determine whether or not

(a) Alif’s estimate is correct to 1 decimal place of the exact value,

Exact price = $13.6 (correct to 1 d.p.)

Alif’s estimate = $13.8 (correct to 1.d.p)

Alif’s estimate is not correct to 1 decimal place of the exact value.

(b) Balan’s estimate is correct to 1 significant figure of the exact value,

Exact price = $10 (correct to 1 s.f.)

Balan’s estimate = $10 (correct to 1.s.f.)

Balan’s estimate is correct to 1 significant figure of the exact value.

(c) Cai Ling’s estimate is correct to 2 significant figures of the exact value,

Exact price = $14 (correct to 2 s.f.)

Cai Ling’s estimate = $13 (correct to 2 s.f.)

Cai Ling’s estimate is not correct to 2 significant figures of the exact value.

(d) Danny’s estimate is correct to 3 significant figures of the exact value.

Exact price
= $8.75 × 0.543 + $23.45 × 0.376
= $13.56845

Exact price = $13.6 (correct to 3 s.f.)

Danny’s estimate = $13.5 (correct to 3 s.f.)

Danny’s estimate is not correct to 3 significant figures of the exact value.

5. Compare and discuss their estimated values. What can you conclude?

Alif’s and Danny’s estimates are the closest to the exact value. Using rounded off values for intermediate calculations gives different results.

85
Class Activity 5
Objective: To make estimates and check if the answers are reasonable.

Questions
The manager of an event management company sold 1324 concert tickets at $18.95 each. He used a calculator to work out the
revenue. As the calculator displayed a sum of over $250 000, he jumped up in excitement.

1. Find an approximate value of the revenue. Describe the method that you use.

Price of each ticket = $18.95


= $20 (correct to 1 s.f.)

Number of tickets sold = 1324


= 1000 (correct to 1 s.f.)

≈ $20 × 1000
Revenue
= $20 000

The method is to round off each number to 1 significant figure before working out the final answer.

2. Based on your answer in Question 1, do you expect the manager to remain excited for long? Explain your reason.

No, the manager should not be excited for long. This is because his calculated revenue is wrong and the actual sum should be much less.

3. Can you tell the degree of accuracy of your answer in Question 1? Explain your answer.

= $18.95 × 1324
Revenue
= $25 089.80
= $30 000 (correct to 1 s.f.)

The answer in question 1 is not correct to 1 significant figure. In fact, if each value of two numbers is rounded to 1 significant figure, the product of

the rounded numbers is not necessarily equal to the product of the given numbers correct to 1 significant figure.

Chapter 3  Approximation and Estimation


86
Try It! Solution
(a) 0.070 185 = 0.070  (correct to 2 s.f.)
Section 3.1
1. Given that the exact population of Hong Kong at a certain (b) 0.070 185 = 0.070 2 (correct to 3 s.f.)
moment is 7 147 382, state the population correct to the
nearest 6. Mrs Tan spent $17 345 on buying a watch. Round off
(a) 1000, (b) million. the price of the watch to
(a) 3 significant figures,
Solution (b) 4 significant figures.
(a) 7 147 382
= 7 147 000  (correct to the nearest 1000) Solution
The population is 7 147 000, correct to the nearest (a) $17 345 = $17 300 (correct to 3 s.f.)
1000.
(b) $17 345 = $17 350 (correct to 4 s.f.)
(b) 7 147 382
= 7 000 000  (correct to the nearest million)
The population is 7 000 000, correct to the nearest 7. State the number of significant figures in each number.
million. (a) 3.12 (b) 0.050 060
(c) 3847 (d) 3800

2. Round off 9.0035 to Solution


(a) 2 decimal places, (b) 3 decimal places. (a) 3.12 has 3 significant figures.

Solution (b) 0.050 060 has 5 significant figures.


(a) 9.0035 = 9.00 (correct to 2 d.p.) (c) 3847 has 4 significant figures.
(b) 9.0035 = 9.004 (correct to 3 d.p.) (d) 3800 has 2, 3 or 4 significant figures.

3. The mass of a piece of A4 paper is 0.004 872 kg. Round 8. The number of passengers who arrived in Singapore
off the mass to by civil aircrafts in December 2010 was 2 016 952.
(a) 3 decimal places, Find the average number of passengers who arrived
(b) 5 decimal places. per day in that month, giving your answer correct to
3 significant figures.
Solution
(a) 0.004 872 = 0.005 (correct to 3 d.p.) Solution
(b) 0.004 872 = 0.004 87 (correct to 5 d.p.) The average number of passengers arriving per day
= 2016952 ÷ 31
= 65100 (correct to 3 s.f.)
7 9
4. (a) Evaluate 3 × –1 .
11 20

(b) Round off the answer to 4 decimal places. Section 3.3


9. 1382 people paid $21.50 each to watch a soccer match
Solution
in a stadium. Estimate the revenue received from the
7 9 40 29 match by rounding off the given numbers correct to
(a) 3 × –1 = – ×
11 20 11 20 (a) 1 significant figure, (b) 2 significant figures.
58
=–
11 Solution
3
= –5
11 (a) Revenue received
3 = $21.50 × 1382
(b) –5 = –5.272 727 … ≈ $20 × 1000 Each number is correct to 1 s.f.
11
= –5.272 7  (correct to 4 d.p.) = $20000

(b) Revenue received


Section 3.2 = $21.50 × 1382
5. Round off 0.070 185 to ≈ $22 × 1400 Each number is correct to 2 s.f.
(a) 2 significant figures, = $30800
(b) 3 significant figures.

87
10. The masses of 6 boys are 53 kg, 56 kg, 61 kg, 52 kg, 59 3
14. Estimate the value of 990 ÷ 50, giving your answer
kg and 54 kg. Using a cluster value, estimate
correct to 2 significant figures.
(a) the total mass of the boys,
(b) the average mass of the boys. Solution
3 3
Solution 990 ÷ 50 ≈ 1000 ÷ 49
(a) Total mass of the boys = 10 ÷ 7
= 53 + 56 + 61 + 52 + 59 + 54 = 1.4  (correct to 2 s.f.)
= 55 + 55 + 55 + 55 + 55 + 55
≈ 55 × 6
= 330 kg 15. (a) Calculate 56.48 × 13.87 correct to 3 significant
figures.
(b) Average mass of the boys (b) (i) Round off 56.48 and 13.87 to 3 significant
≈ 330 ÷ 6 figures.
= 55 kg Take cluster value = 55 kg
(ii) Calculate the product of the answers in (i)
and give your answer correct to 3 significant
11. In the diagram, the boy is 1.2 m tall. Estimate figures.
(a) the height of the tree,
(b) the height of the lamp post. Solution
(a) 56.48 × 13.87 = 783.377 6
Solution = 783  (correct to 3 s.f.)
(a) We take the height of the boy from the ground as = 56.5 
(b) (i) 56.48 (correct to 3 s.f.)
a benchmark for measurement. 13.87 = 13.9  (correct to 3 s.f.)
Height of the tree
≈ 2 times the height of the boy from the ground (ii) 56.5 × 13.9
= 2×1.2 m = 785.35
= 2.4 m = 785   (correct to 3 s.f.)
(b) Height of the lamp post
≈ 3 times the height of the boy from the ground 16. 2260 kJ of heat energy is required during the process of
= 3×1.2 m converting 1 kg of water of steam at 100°C. Find the
= 3.6 m amount of heat energy required to convert 0.487 kg of
water to steam at 100°C, correct to 4 significant figures.
12. There is a stack of 10 books on a table. If each book is Solution
about 2 cm thick, estimate the height of the stack. Amount of heat required
= 0.487 × 2260
Solution
= 1100.62 kJ
Height of the stack = 1101 kJ   (correct to 4 s.f.)
= 10 × thickness of a book
≈ 10 × 2
= 20 cm

13. Estimate the value of 79.5 – 3.21 × 29.52 by rounding


off each number correct to 1 significant figure.

Solution
79.5 – 3.21 × 29.52
≈ 80 – 3 × 30   Each number is correct to 1 s.f.
= 80 – 90
= –10

Chapter 3  Approximation and Estimation


88
Exercise 3.1 8. (a) Evaluate 315 using a calculator.
Level 1 (b) Round off the answer to the nearest million.
1. Round off the following numbers to the nearest integer. Solution
(a) 13.4 (b) 321.8 (a) 315 = 14 348 907
(b) 14 348 907
Solution
= 14 000 000   (correct to the nearest million)
(a) 13.4 = 13  (correct to the nearest integer)
(b) 321.8 = 322 (correct to the nearest integer) 9. Express the following correct to 4 decimal places.
5 10
(a) (b)
2. Round off the following numbers to the nearest 100. 9 11
(a) 7289 (b) 13 562
Solution
5
Solution (a) = 0.555 55 … = 0.5556 (correct to 4 d.p.)
9
(a) 7289 = 7300  (correct to the nearest 100)
10
(b) 13 562 = 13 600 (correct to the nearest 100) (b) = 0.909 090 … = 0.9091 (correct to 4 d.p.)
11

3. Round off the following numbers to 1 decimal place.


10. Evaluate the following and give the answers correct to
(a) 23.69 (b) 0.72
3 decimal places.
2 1 1
Solution (a) × (–8 + 12) (b) 4 – 5
3 7 8
(a) 23.69 = 23.7  (correct to 1 d.p.)
2 7 172 – 82
(b) 0.72 = 0.7 (correct to 1 d.p.) (c) –4 ÷1 (d)
9 12 13

4. Round off the following numbers to 2 decimal places. Solution


(a) 10.7543 (b) 2.9968 2 2
(a) × (–8 + 12) = ×4
3 3
Solution 8
=
(a) 10.7543 = 10.75  (correct to 2 d.p.) 3

(b) 2.9968 = 3.00 (correct to 2 d.p.) = 2.667  (correct to 3 d.p.)

1 1 29 41
(b) 4 – 5 = –
5. Round off the following numbers to 3 decimal places. 7 8 7 8
(a) 0.040 25 (b) 17.926 53 232 – 287
=
56
Solution 55
=–
(a) 0.040 25 = 0.040  (correct to 3 d.p.) 56

(b) 17.926 53 = 17.927  (correct to 3 d.p.) = – 0.982  (correct to 3 d.p.)



2 7 38 12
6. Round off the following numbers to 4 decimal places. (c) –4 ÷ 1 =– ×
9 12 9 19
(a) 3.004 056 (b) 8.471 345 8
=–
3
Solution = –2.667  (correct to 3 d.p.)
(a) 3.004 056 = 3.0041 (correct to 4 d.p.)
(b) 8.471 345 = 8.4713 (correct to 4 d.p.) 172 – 82 289 – 64
(d) =
13 13

225
Level 2 =
13
7. (a) Evaluate (–13) × [(–17) + (–12)]. 15
(b) Round off the answer to the nearest 10. =
13

Solution = 1.154  (correct to 3 d.p.)


(a) (–13) × [(–17) + (–12)] = (–13) × (–29)
= 377
(b) 377 = 380  (correct to the nearest 10)

89
Level 3 (b) Record time
11. The number of people who attended an outdoor event is = 50.99 s
12 756. Round off this number to the nearest = 51.0 s (correct to the nearest 0.1 s)
(a) hundred, (b) thousand.
16. The height of a basketball player is 2.1036 m when
Solution measured with a precise instrument.
(a) 12 756 = 12 800  (correct to the nearest 100) Express his height correct to
The required number of people is 12 800, correct (a) 2 decimal places,
to the nearest 100. (b) 3 decimal places.
(b) 12 756 = 13 000  (correct to the nearest 1000)
The required number of people is 13 000, correct Solution
to the nearest 1000. (a) His height = 2.1036 m
= 2.10 m  (correct to 2 d.p.)
12. The Upper Peirce Reservoir has a water storage capacity (b) His height = 2.1036 m
of 27 800 000 m3. Round off the capacity to the nearest = 2.104 m  (correct to 3 d.p.)
1 000 000 m3.
17. The length of a piece of A4-size paper is
Solution 29.7302 cm.
Water capacity = 27 800 000 m3 Express the length correct to the nearest
= 28 000 000 m3 (a) cm, (b) 0.1 cm,

(correct to the nearest 1000 000) (c) 0.001 cm.

13. Asia is the largest continent in the world. Given that its Solution
land area is 44 579 000 km2, round off the area to the (a) The length = 29.7302 cm
nearest 100 000 km2. = 30 cm (correct to the nearest cm)
(b) The length = 29.7302 cm
Solution = 29.7 cm
Area = 44 579 000 km2 (correct to the nearest 0.1 cm)
= 44 600 000 km2 (correct to the nearest 100 000)
(c) The length = 29.7302 cm
2
= 29.730 cm 
14. The area of an apartment is 119.3 m . Round off this (correct to the nearest 0.001 cm)
area to the nearest
(a) m2, (b) 10 m2.
18. Sulin wants to determine the thickness of a $1 coin using
Solution a ruler with millimetre scale. The height of a stack of 25
(a) 119.3 = 119  (correct to the nearest whole $1 coins is 58 mm.
number) (a) Find the thickness of a $1 coin, based on Sulin’s
The required area is 119 m2, correct to the nearest method of measurement, correct to 0.1 mm.
m2. (b) Is Sulin’s method better than to measuring the
height of one coin? Explain briefly.
(b) 119.3 = 120  (correct to the nearest 10)
The required area is 120 m2, correct to the nearest Solution
10 m2. (a) Thickness of a coin
= 58 ÷ 25
15. I n t h e 2 0 1 0 S i n g a p o r e N a t i o n a l S w i m m i n g = 2.32
Championships, Yeo Kai Quan Danny, set the national = 2.3 mm  (correct to the nearest 0.1 mm)
record time of 50.99 seconds for the men’s 100 m (b) Yes, the rounding error due to the precision of the
freestyle event. Express the record time to the nearest measuring instrument is smaller.
(a) second, (b) tenth second.

Solution 19. The length of a black ant is 0.003 25 m.


(a) Record time (a) Express the length correct to 2 decimal places.
= 50.99 s (b) Is it a good idea to present the rounded figure in
= 51 s (correct to the nearest second) (a) as the length of the ant? Give your reason.
(c) If you want to tell a primary school pupil the rough
length of the ant, what would you say?

Chapter 3  Approximation and Estimation


90
Solution Exercise 3.2
(c) Length of the black ant Level 1
= 0.00325
= 0.00 m (correct to 2 d.p.) 1. Round off each number to 1 significant figure.
(a) 13.67 (b) 0.0392
(b) The rounded figure is NOT good. It gives an
impression that the length is zero. Solution
(a) 13.67 = 10 (correct to 1 s.f.)
(c) We should round off the length correct to 3 decimal
places and say the length of the ant is about 0.003m. (b) 0.0392 = 0.04 (correct to 1 s.f.)

20. A number is rounded off to 38 000. 2. Round off each number to 3 significant figures.
(a) What are the possible degrees of accuracy of the (a) 69 352 (b) 13.047
approximation?
(b) Suggest a value of the actual number corresponding Solution
to each of your possible degrees of accuracy. (a) 69 352 = 69 400 (correct to 3 s.f.)
(b) 13.047 = 13.0  (correct to 3 s.f.)
Solution
(a) 38 000 can be correct to the nearest thousand,
hundred, ten or whole number. 3. Round off each number to 5 significant figures.
(a) 4 253 607 (b) 8.004 036 25
(b) Some possible values are:
(i) 38 125 Solution
= 38 000  (correct to the nearest 1000) (a) 4 253 607 = 4 253 600  (correct to 5 s.f.)
(ii) 38 047 (b) 8.004 036 25 = 8.0040  (correct to 5 s.f.)
= 38 000  (correct to the nearest 100)
(iii) 38 003 4. State the number of significant figures in each number.
= 38 000  (correct to the nearest 10) (a) 38 (b) 72 009
(iv) 38 000.49 (c) 15.340 (d) 6800
= 38 000  (correct to the nearest whole
number) Solution
(a) 38 has 2 significant figures.
21. Michael’s time for the 100 m sprint is recorded as (b) 72 009 has 5 significant figures.
9.96 seconds. State two possible values of his actual time
for the 100 m sprint. (c) 15.340 has 5 significant figures.
(d) 6800 has 2, 3 or 4 significant figures.
Solution
Two possible values of his actual time for the 100 m
sprint are 9.957 seconds and 9.962 seconds. Level 2
6
5. (a) Convert the fraction into a decimal.
7
22. Find some examples of the use of decimals in the (b) Express the answer in (a) correct to 4 significant
newspaper. Are the numbers exact or estimated? If they figures.
are estimated, what are their degrees of accuracy?
Solution
Solution 6
(a) = 0.857 142
Some examples of the use of decimals are: 7
stock price index  (correct to 2 d.p.), (b) 0.857 142 = 0.8571  (correct to 4 s.f.)
maximum daily temperature (correct to the nearest
0.1°C),
unemployment rate  (correct to the nearest 0.1%). 6. Evaluate (– 4.937) + (–3.625) and give your answer
correct to 3 significant figures.

Solution
(– 4.937) + (–3.625) = –(4.937 + 3.625)
= –8.562
= –8.56  (correct to 3 s.f.)

91
1 2 1 1 11. The length and breadth of a rectangular playground are
7. (a) Evaluate 1 × + –2 × – .
4 9 2 6 30.1 m and 25.6 m respectively. Find the area of the
(b) Express the answer in (a) as a decimal correct to playground, correct to 3 significant figures.
4 significant figures.
(c) Find the square root of the answer in (a). Solution
Area of playground = 30.1 × 25.6
Solution = 770.56
= 771 m2  (correct to 3 s.f.)
1 2 1 1 5 2 5 1
(a) 1 × + –2 × – = × + ×
4 9 2 6 4 9 2 6

5 5 12. The numbers of private cars in Singapore in the years
= +
18 12 2009 and 2010 were 566 608 and 584 399 respectively.
10 + 15 The number of resident households in 2009 was 1 119
= 600. Giving your answers correct to 3 significant figures,
36
25 find
= (a) the increase in the number of private cars in the
36
25 year 2010,
(b) = 0.6944  (correct to 4 s.f.) (b) the average number of cars per household in 2009.
36

2
25 5 Solution
(c) =
36 6 (a) The increase = 584 399 – 566 608
5 = 17791
= = 17800 (correct to 3 s.f.)
6

(b) Average number of cars per household


0.049 23
8. Evaluate , correct to 2 significant figures. = 566 608 ÷ 111 9600
23.56 –
3
13.67 = 0.506 (correct to 3 s.f.)
Solution
0.049 23 13. A number is rounded off to 0.0506, correct to 3 significant
= –0.0059 (correct to 2 s.f.)
23.56 – figures. Give 3 possible values of this number.
3
13.67

Solution
Level 3
Three possible values of the number are
9. In 2010, the number of people employed in Singapore 0.050 587, 0.050 592 and 0.050 618.
was 3105.9 thousand.
(a) How many significant figures are there in this
number? 14. The length and width of the floor of a room are 6.7 m and 3.8
(b) Write this number in full correct to 4 significant m respectively, correct to 2 significant figures. Is it possible
figures. that the area of the floor of the room is 26 m2, correct to
2 significant figures? Explain your answer.
Solution
(a) 3105.9 thousand has 5 significant figures. Solution
Let the room be x m long and y m wide.
(b) The required number Then  6.65  x  6.75
= 3 106 000  (correct to 4 s.f.) 3.75  y  3.85
  Area = xy
10. The land area of Singapore is 712.4 km2. The population   6.65 × 3.75  Area  6.75 × 3.85
of Singapore in the year 2010 was 5 076 700. On average, 24.937 5  Area  25.987 5
how many people lived on 1 km2 of land? Give your It is possible that the exact area is 25.9 m2.
answer correct to 3 significant figures. Then the area will be 26 m2, correct to 2 significant
figures
Solution
The required number of people 15. The numbers of visitors to Sentosa in 2000, 2005 and
= 5 076 700 ÷ 712.4 2010 are 3392.5 thousand, 4989.7 thousand and 19 087.3
= 7130  (correct to 3 s.f.) thousand respectively.
(a) Round off the number of visitors in 2000 to the
nearest thousand.
(b) Round off the number of visitors in 2005 to the
nearest million.

Chapter 3  Approximation and Estimation


92
(c) On average, how many visitors did Sentosa receive (h) 1878 ÷ 29.2 ÷ 3.14
every month in 2010? Give your answer correct to ≈ 2000 ÷ 30 ÷ 3 Each number is correct to 1 s.f.
4 significant figures. ≈ 22  (correct to 2 s.f.)
(d) It is observed that the number of visitors to Sentosa
(i) (16.9 – 5.47) × 7.09
is much higher in 2010 than that in 2000 or 2005.
≈ (20 – 5) × 7 Each number is correct to 1 s.f.
Give a possible reason for this observation.
= 105
Solution ( j) 6.252 ÷ 4.38
(a) Number of visitors in 2000 ≈ 40 ÷ 4 Each number is correct to 1 s.f.
= 3392500 = 10
= 3393000 (correct to the nearest thousand)
Note: The above answers may not be the same as the
(b) Number of visitors in 2005 actual answers that are correct to 1 s.f.
= 4989700
Refer to 1(f ):
= 5000000 (correct to the nearest million)
13.24 × 4.83 × 6.09
(c) Average number of visitors per month in 2010 = 389.45 (correct to 5 s.f.)
= 19087300 ÷ 12 = 400  (correct to 1 s.f.)
= 1591000 (correct to 4 s.f.)
(d) One possible reason could be the opening of 2. Determine whether each estimated value is resonably
Universal Studios Singapore in 2010. close to the exact value of the expression on the left-hand
side.
(a) 347 – 482 + 659 ≈ –500
Exercise 3.3 (b) 23.92 × 4.801 ≈ 1200
Level 1
Solution
1. Without using a calculator, estimate each expression (a) 347 – 482 + 695
by rounding off each number in the expression to 1 ≈ 300 – 500 + 700
significant figure. = 500
(a) 2315 + 5967 (b) 45 301 – 18 432 The estimated value, –500, is NOT reasonable.
(c) 84 × 26 (d) 2853 ÷ 21.5
(e) 125 + 3.91 × 27.48 (f ) 13.24 × 4.83 × 6.09 (b) 23.92 × 4.801
(g) 38 467 × 0.002 96 (h) 1878 ÷ 29.2 ÷ 3.14 ≈ 20 × 5
(i) (16.9 – 5.47) × 7.09 ( j) 6.252 ÷ 4.38 = 100
The estimated value, 1200, is NOT reasonable.
Solution
(a) 2315 + 5967 3. Estimate, correct to 1 significant figure, the value of each
≈ 2000 + 6000 Each number is correct to 1 s.f.
of the following expressions. You may use a calculator
= 8000 in this question.
(b) 45 301 – 18 432 (a) 38 467 × 0.002 02
≈ 50 000 – 20 000 Each number is correct to 1 s.f. (b) 1840 ÷ 177 ÷ 39.6
= 30 000 (c) (159 – 60.3) × 7.58
(d) 6.032 ÷ 1.48
(c) 84 × 26
≈ 80 × 30 Each number is correct to 1 s.f. (e) 14.76 × 80.2
= 2400 2.99
23.33 × 1.991
(d) 2853 ÷ 21.5 (f)
(0.202)3
≈ 3000 ÷ 20 Each number is correct to 1 s.f.
= 150 Solution
(e) 125 + 3.91 × 27.48 (a) 38 467 × 0.002 02
≈ 100 + 4 × 30 Each number is correct to 1 s.f. = 80 (correct to 1 s.f.)
= 220 (b) 1840 ÷ 177 ÷ 39.6

(f ) 13.24 × 4.83 × 6.09 = 0.3 (correct to 1 s.f.)
≈ 10 × 5 × 6 Each number is correct to 1 s.f. (c) (159 – 60.3) × 7.58
= 300 = 700 (correct to 1 s.f.)
(g) 38 467 × 0.002 96 (d) 6.032 ÷ 1.48
≈ 40 000 × 0.003 Each number is correct to 1 s.f. = 20 (correct to 1 s.f.)
= 120

93
(e) 14.76 × 80.2 = 20 (correct to 1 s.f.) (c) The answers in (a) and (b) are NOT the same.
2.99 The answers in (a) is more accurate.
23.33 × 1.991
(f) = 6000 (correct to 1 s.f.) 6. Consider the expression 1.425 × 84.73 ÷ 6.25.
(0.202)3
(a) Find the exact value of the expression.
(b) Express the answer in (a) correct to 3 significant
Level 2 figures.
4. Estimate the value of each expression. (c) (i) Compute the expression after each number
is rounded off to 1 significant figure, giving
(a) 160 the answer correct to 1 significant figure.
3 (ii) Is the answer in (c)(i) same as the answer in
(b) 350
(a) correct to 1 significant figure?
(d) (i) Compute the expression after each number
(c) 891 × (23.25 – 9.63)
is rounded off to 3 significant figures, giving
(d)
3
521 ÷ 0.0418 the answer correct to 3 significant figures.
(ii) Is the answer in (d)(i) same as that in (b)?

Solution Solution
(a) 1.425 × 84.73 ÷ 6.25
(a) 160 ≈ 169
= 19.31844
= 13
(b) Answer in (a) = 19.3 (correct to 3 s.f.)
3 3
(b) 350 ≈ 343
(c) (i) The expression
=7 ≈ 1 × 80 ÷ 6 (Round off each number)
= 13.3
(c) 891 × (23.25 – 9.63) = 10 (correct to 1 s.f.)
≈ 900 × (20 – 10) (ii) The answer in (c)(i) is NOT the same as the
= 30 × 10 answer in (a) correct to 1 significant figure,
which is 20.
= 300
(d) (i) The expression
(d)
3
729 ÷ 0.041 8 ≈ 1.43 × 84.7 ÷ 6.25 (Round off each number)
= 19.37936
3
≈  512 ÷ 0.04 = 19.4 (correct to 3 s.f.)
= 8 ÷ 0.04 (ii) The answer in (d)(i) is NOT the same as the
= 200 answer in (b).

5. Consider the expression 4.5012 – 3.2167 – 6.7183. 7. Evaluate the expression


(a) Evaluate the expression correct to 3 significant 2.973
figures. 0.998 × 60.26 – × 33.44,
9.015
(b) Evaluate the expression after each number in the giving your answer correct to
expression has been rounded off to 3 significant (a) 3 significant figures,
figures, giving the answer correct to 3 significant (b) 1 significant figure.
figures.
(c) Are the answers in (a) and (b) the same? If not, Solution
2.973
which answer is more accurate? (a) 0.998 × 60.26 – × 33.44
9.015
= –37.03895
Solution = –37.0 (correct to 3 s.f.)
(a) 4.5012 – 3.2167 – 6.7183 (b) The answer
= –5.433 8 = –40 (correct to 1 s.f.)
= –5.43  (correct to 3 s.f.)
(b) 4.5012 – 3.2167 – 6.7183 8. Suppose you had $3458 in your wallet. You spent $361,
= 4.50 – 3.22 – 6.72  (correct to 3 s.f.) $86, $405 and $299 on 4 items during a shopping trip.
= –5.44 Estimate the amount of money left by rounding off each
given value to the nearest $100.

Chapter 3  Approximation and Estimation


94
Solution 13. The diagram shows a $1 coin placed on an ez-link card.
$(3458 – 361 – 86 – 405 – 299) The coin is 2.2 cm in diameter. Estimate the length of
≈ $(3000 – 400 – 100 – 400 – 300) the ez-link card using the coin.
= $1800
The estimated amount of money left is $1800. Solution
Length of the ez-link card
≈ 4 times the diameter of a $1 coin
9. A group of soldiers in a parade are arranged in a
= 4 × 2.2
rectangular array of 32 rows by 28 columns. Estimate
= 8.8 cm
the number of soldiers in the group.

Solution 14. The diagram shows a scale floor plan of a living room.
32 × 28 ≈ 30 × 30 = 900 Given that the length of the sofa is 2 m, estimate
The estimated number of soldiers is 900. (a) the length of the room,
(b) the length of the cabinet,
(c) the width of the table.
10. Mrs Li buys 4 bottles of peanut oil and 3 boxes of
detergent from a supermarket. The price of a bottle of
Solution
peanut oil is $3.85 and that of a box of detergent is
(a) Length of the room
$17.95. Estimate the total price of Mrs Li’s purchases.
≈ 3 times the length of the sofa
=3×2
Solution
=6m
$(3.85 × 4 + 17.95 × 3) ≈ $(4 × 4 + 20 × 3)
= $76 (b) Length of the cabinet
The estimated total price is $76. ≈ 1.5 times the length of the sofa
= 1.5 × 2
=3m
11. The distance covered by an athlete in 4 attempts of a
long jump events are (c) Width of the table
7.58 m, 7.62 m, 7.54 m and 7.63 m. 1
≈ times of the length of the sofa
Without using a calculator, 2
(a) estimate the total distance in these 4 jumps. 1
= ×2
(b) estimate the average distance of these 4 jumps. 2
=1m
Solution
(a) Take cluster value = 7.6 m
15. A row of 12 chairs can fit along the wall AB of a room.
Total distance in the 4 jumps
If each chair is 45 cm wide, estimate the length of the
= 7.58 + 7.62 + 7.54 + 7.63
wall AB.
≈ 7.6 + 7.6 + 7.6 + 7.6
= 30.4 m Solution
(b) Average distance of the 4 jumps Length of the wall
≈ 30.4 ÷ 4 ≈ 12 times the width of a chair
= 7.6 m = 12 × 45
= 540 cm
12. The duration of 5 badminton matches in minutes is as 16. There were 100 cars in the queue at one traffic congestion
follows: on the Ayer Rajah Expressway (AYE). Assuming that each
17.3, 21.2, 18.9, 22.6 and 19.5. car was 4.5 m long, estimate the length of the queue.
Without using a calculator, estimate the total duration of What else would you assume in making this estimation?
these 5 matches.
Solution
Solution Assume that the gap between two adjacent cars is 0.5 m.
Take cluster value = 20 min Length of the queue
Total playtime of the 5 matches ≈ 100 × 4.5 + 99 × 0.5
= 17.3 + 21.2 + 18.9 + 22.6 + 19.5 = 499.5 m
≈ 20 + 20 + 20 + 20 + 20 = 500 m (correct to the nearest 10 m)
= 100 min

95
17. A rectangular field is 64.19 m long and 25.68 m (d) We should round up every mass to the nearest kg.
wide. If the sum of the round up masses is less than the
(a) Find its area correct to 2 significant figures. maximum loading, the lift will not be overloaded.
(b) (i) Express the length correct to 2 significant
figures.
19. Give two examples of estimation used in real-world
(ii) Express the breadth correct to 2 significant
situations.
figures.
(iii) Using the approximate values in (i) and (ii),
Solution
estimate the area of the field correct to 2
Population figures, death toll in a major disaster,
significant figures.
expenditures in a week, etc, are some examples of
(c) Are the answers in (a) and (b) (iii) the same? If
estimation used in real-world situations.
not, which answer is more accurate?

Solution
(a) Area
= 64.19 × 25.68
Revision Exercise 3
= 1600 m2  (correct to 2 s.f.) 1 1 1 1
1. Evaluate 3 × 7 + –1 ÷ 2 and give your answer
2 3 8 4
(b) (i) 64 m (a) in the exact value,
(ii) 26 m (b) correct to 3 significant figures.

(iii) 64 × 26 Solution
= 1664 1 1 1 1
(a) 3 × 7 + –1 ÷ 2
= 1700 m2  (correct to 2 s.f.) 2 3 8 4
7 22 9 4
(c) They are not the same. = × – ×
2 3 8 9
The answer in (a) is more accurate. 77 1
= –
3 2
18. The maximum loading of a lift is 8 persons or =
154 – 3
600 kg. There are 8 persons whose masses (in kg) are 6
71.8, 73.4, 75.3, 74.8, 76.4, 73.9, 78.4 and 77.4. 151
=
(a) Estimate the total mass of the 8 persons by rounding 6
off each mass to 2 significant figures. 1
= 25
(b) Based on the estimation in (a), is it safe to let these 6
8 persons take the lift at the same time?
(c) Explain why the lift stops working when these 8 1
(b) 25 = 25.16
persons take the lift at the same time. 6
(d) Devise an estimation method which will = 25.2  (correct to 3 s.f.)
ensure that the lift will not be overloaded if
the estimated total mass is not greater than 2. (a) Find the LCM of 28, 36 and 120.
600 kg. (b) Round off the LCM to the nearest 100.

Solution Solution
(a) Total mass (a)  28 = 22 × 7
= 71.8 + 73.4 + 75.3 + 74.8 + 76.4 + 73.9 + 78.4  36 = 22 × 32
+ 77.4 120 = 23 × 3 × 5
≈ 72 + 73 + 75 + 75 + 76 + 74 + 78 + 77 LCM of these = 23 × 32 × 5 × 7
(Round off each number correct to 2 s.f.) = 2520
= 600 kg
(b) 2520 = 2500  (correct to the nearest 100)
(b) Since the estimated total mass is less than or equal
to the maximum loading, it is safe to let these 8
persons take the lift at the same time. 3. Evaluate
3 196.42 ,
(c) Total mass (a) correct to 2 decimal places,
0.047 58
= 601.4 kg
> maximum loading 259 – 6.253
Thus  the lift stops working. (b)  , correct to 3 significant figures.
2012 – 34.67

Chapter 3  Approximation and Estimation


96
Solution Solution
3 196.42 (a) In the diagram,
(a) = 16.04  (correct to 2 d.p.)
0.047 58 length of car = 1.6 cm,
length of each lot = 2.4 cm,
259 – 6.253
(b) = 1.46  (correct to 3 s.f.) width of each lot = 1 cm.
2012 – 34.67
actual length of each lot
2.4
≈ 4.5 ×
4. Evaluate 1.6
(a) (3.159 ÷ 22.36)2 × (13.17 – 9.08), correct to the = 6.75 m
nearest integer, = 6.8 m  (correct to 1 d.p.)
3
5 – 1.236
actual width of each lot
(b) , correct to 4 significant figures. 1
1.432 ≈ 4.5 ×
1.6
Solution = 2.8125 m
(a) (3.159 ÷ 22.36)2 × (13.17 – 9.08) = 2.8 m  (correct to 1 d.p.)
= 0.08164
= 0  (correct to the nearest integer) The dimensions of each lot is about
6.8 m by 2.8 m.
3
5 – 1.236 (b) Area of the car park
(b) = 0.002318  (correct to 4 s.f.)
1.432 ≈ (2 × 6.8) × (3 × 2.8)
= 114.24
= 114 m2  (correct to the nearest m2)
5. In the year 2010, the population of Singapore was 5 076
700 and the total number of mobile phone subscribers was
7 288 600. Find the average number of mobile phones 8. Meili forgot to put the decimal point in the answer of
subscribed by each person, correct to 1 decimal place. each calculation. Show how you can use estimation to
put the decimal point correctly.
Solution (a) 637 × 0.54 = 34 398
Average number of mobile phones subscribed per person (b) 2198 × 0.0047 = 103 306
= 7288600 ÷ 5076700
= 1.4 (correct to 1 d.p.) Solution
(a) 637 × 0.54
6. The prices of 3 items are $16.95, $23.40 and $5.15. ≈ 600 × 0.5 Round off each correct to 1 s.f.
(a) Estimate the total price of these 3 items by rounding = 300
off each price to 1 significant figure. \ 637 × 0.54 = 343.98
(b) If Mr Cai has $50 in his pocket, would he have (b) 2198 × 0.0047
enough money to buy these 3 items? Explain briefly. ≈ 2000 × 0.005 Round off each correct to 1 s.f.
= 10
Solution \ 2198 × 0.0047 = 10.3306
(a) Total price
= $(16.95 + 23.40 + 5.15)
≈ $(20 + 20 + 5) 9. A rectangular sign board measures 2.13 m by 4.08 m.
Round off each number to 1 s.f. (a) Give an estimate of its area.
= $45 (b) Calculate the area of the board correct to 3
significant figures.
(b) Since $45 < $50, Mr Cai should have enough money (c) The cost per square metre of the board is $56.90.
to buy these 3 items. Find the cost of the board correct to the nearest
dollar.
7. The diagram shows a car park which consists of
6 identical parking lots. The car parked in one of the lots Solution
is 4.5 m long and 1.7 m wide. Estimate (a) 2.13 × 4.08
(a) the dimensions of each lot, ≈ 2 × 4 m2  Round off each correct to 1 s.f.
(b) the area of the car park. = 8
Its estimated area is 8 m2.

(b) Area of the board
= 2.13 × 4.08
= 8.690 4
= 8.69 m2  (correct to 3 s.f.)

97
(c) Cost of the board
= $56.90 × 8.690 4
= $494.483 76
= $494  (correct to the nearest dollar)

10. A bookshop purchases 185 calculators at a cost of $25.13


each. Two of the calculators are damaged and the rest are
sold at $34.90 each. Find, correct to the nearest dollar,
(a) the total cost of the calculators,
(b) the total revenue collected from the sales of the
calculators,
(c) the profit made from the sales of the calculators.

Solution
(a) Total cost = $25.13 × 185
= $4649.05
= $4649  (correct to the nearest dollar)
(b) Total revenue
= $34.90 × (185 – 2)
= $6386.70
= $6387  (correct to the nearest dollar)
(c) Profit
= $(6386.70 – 4649.05)
= $1737.65
= $1738  (correct to the nearest dollar)

Chapter 3  Approximation and Estimation


98
4 Introduction to Algebra

Class Activity 1
Objective: To compare algebraic expressions 2n, 2 + n, n2, 2n2, (2n)2 and 4n2.

Tasks
A B C D E F G
1 Algebraic Expressions
2
3 n 2n 2+n n^2 2n^2 (2n)^2 4n^2
4 -5 -10 -3 25 50 100 100
5 - 4 -8 -2 16 32 64 64
6 -3 -6 -1 9 18 36 36
7 -2 -4 0 4 8 16 16
8 -1 -2 1 1 2 4 4
9 0 0 2 0 0 0 0
10 1 2 3 1 2 4 4
11 2 4 4 4 8 16 16
12 3 6 5 9 18 36 36
13 4 8 6 16 32 64 64
14 5 10 7 25 50 100 100

1. In a spreadsheet, enter the headings and the data for the first column, then generate the values for other columns using
formulae as shown.
Note: In a spreadsheet program, * stands for multiplication and ^ stands for power of a number.

2. Work with a partner and try to represent your answers clearly and logically for the following questions.

99
Questions
1. Look at the values under the columns of 2n and 2 + n. Can we say 2n = 2 + n? Explain your answer.

The answers are different. 2n ≠ 2 + n. (≠ means not equal to)

2. Discuss the differences between 2n and n2 when the value of n changes from negative numbers to positive numbers.

The sign for 2n changes from negative to positive while the sign for n 2 is always positive.

3. Do you think the following pairs of expressions are related? Explain your answer.
(a) 2n2 and (2n)2

The value of (2n )2 is twice the value of 2n 2. We say that (2n)2 = 2(2n 2).

(b) (2n)2 and 4n2

The value of (2n )2 is equal to the value of 4n 2. We say that (2n)2 = 4n 2.

(c) 2n2 and 4n2

The value of 4n 2 is twice the value of 2n 2. We say that 4n 2 = 2(2n 2).

Chapter 4  Introduction to Algebra


100
Try It! Solution
(a) d × e × 1 × e = 1 × d × e × e
Section 4.1
= de2
1. Mary is 9 years old. Find her age after
(a) 3 years, (b) 10 years, (b) 4m × 5n × m × m = 4m × m × m × 5n
(c) t years. = 4m3 × 5n
= 20m3n
Solution 7t
Mary’s present age = 9 years (c) 7t ÷ y × 2 = × 2
y
(a) Age after 3 years = (9 + 3) 14t
=
= 12 years y

(b) Age after 10 years = (9 + 10)


5. Express the following statements algebraically.
= 19 years
(a) Multiply the sum of c and d by u.
(c) Age after t years = (9 + t) years (b) Subtract the quotient of a divided by b from the
cube of v.
2. A worker’s salary is $200 less than one-third of a
Solution
manager’s salary. Find the worker’s salary when the
(a) Sum of c and d = c + d
manager’s salary is
Multiply c + d by u = (c + d)u
(a) $7200, (b) $9000,
The required expression is (c + d)u.
(c) $m.
a
(b) Quotient of a divided by b =
Solution b
1 Cube of v = v3
Worker’s salary = × Manager’s salary – $200 a a
3 Subtract from v3 = v3 –
b b
(a) When the manager’s salary is $7200, a
The required expression is v3 – .
1 b
worker’s salary = $ × 7200 – $200
3
= $2200
Section 4.2
(b) When the manager’s salary is $9000, 6. When x = 4 and y = 3, find the values of
worker’s salary = $
1
× 9000 – $200 (a) 2x – 4y, (b) xy2 + 2xy + 7.
3
= $2800 Solution
(a) When x = 4 and y = 3,
(c) When the manager’s salary is $m,
2x – 4y = 2(4) – 4(3)
1 = 8 – 12
worker’s salary = $ × m – $200
3 = –4
m
=$ – 200 (b) When x = 4 and y = 3,
3
xy2 + 2xy + 7 = (4)(3)2 + 2(4)(3) + 7
= 36 + 24 + 7
3. The price of a cup is $5 and the price of a plate is $12.
= 67
Find the total price of
(a) 3 cups and 2 plates,
3
(b) n cups and m plates. 7. When x =  , y = 6 and z = –5, find the values of the
4
following expressions.
Solution 10xy – z + 2
(a) z(8x + yz) (b)
(a) Total price of 3 cups and 2 plates xy2 – 3
= $(5 × 3 + 12 × 2)
Solution
= $39
3
(a) When x = , y = 6 and z = –5,
(b) Total price of n cups and m plates 4
= $(5 × n + 12 × m) z(8x + yz)
= $(5n + 12m) 3
= (–5)[8 × + 6 × (– 5)]
4
= (–5)[6 – 30]
4. Simplify the following.
= (–5)(–24)
(a) d × e × 1 × e
= 120
(b) 4m × 5n × m × m
(c) 7t ÷ y × 2

101
10 ×
3
× 6 – (–5) + 2 Section 4.3
10xy – z + 2 4
(b) = 11. The mass of Wendy is 5 kg less than that of Tom.
xy2 – 3 3
× 62 – 3 (a) Let w kg be the mass of Wendy. Express the mass
4
52 of Tom in terms of w.
= 24 (b) Let t kg be the mass of Tom. Write a formula
13 connecting w and t.
= 6
Solution
8. The perimeter, P m, of a rectangular field of l m by b m (a) Mass of Tom = (w + 5) kg
is given by the formula P = 2(l + b). Find the perimeter (b) t = w + 5
of a rectangular field whose dimensions are 60 m by 45
m.
12. There are 3 times as many children as adults.
Solution (a) Let n be the number of adults. Express the number
P = 2 (l + b) of children in terms of n.
When l = 60 and b = 45, (b) Let m be the number of children. Write a formula
P = 2 (60 + 45) connecting m and n.
= 210
Solution
\ The perimeter is 210 m.
(a) Number of children = 3 × n
1
= 3n
9. Given the formula D = n(n – 3), find the value of D
2 (b) m = 3n
when
(a) n = 5, (b) n = 12.
13. In a department store, the price of a ceramic pan is $216
Solution while the price of a stainless pan is $s.
(a) When n = 5, (a) Express the number of stainless pans that can be
1 bought for $216 in terms of s.
D = n(n – 3)
2 (b) If s = 18, find the number of stainless pans that
1 can be bought for $216.
= × 5 × (5 – 3)
2 (c) Mrs Muthu bought n ceramics pans as rewards for
=5 her staff. Let $m be the amount she paid for the
pans. Write a formula connecting m and n.
(b) When n = 12,
(d) Find the value of m when n = 7.
1
D = × 12 × (12 – 3)
2
Solution
= 54 (a) Number of stainless pans that can be bought
216
2 2
= s
v –u
10. If s = , find the value of s when u = 3, v = 12 and
2a (b) When s = 18,
a = 6. number of stainless pans that can be bought
216
= 18
Solution
When u = 3, v = 12 and a = 6, =12
2 2
v –u (c) m = 216n
s =
2a
(d) When n = 7,
122 – 32
= m = 216(7)
2×6
= 1512
135
=
12
1
= 11
4

Chapter 4  Introduction to Algebra


102
14. In an examination, Alan’s score is 5 marks more than
three quarters of Budi’s score, and Chetan’s score is 37
marks less than twice of Alan’s score. Let a, b and c be
the scores of Alan, Budi and Chetan respectively.
(a) Express a and c in terms of b.
(b) If Budi’s score is 80, find the scores of Alan and
Chetan.

Solution
3
(a) a = b + 5
4
c = 2a – 37
3
= 2 b + 5 – 37
4
3
= b + 10 – 37
2
3
= b – 27
2

(b) When b = 80,


3
Alan’s score, a = (80) + 5
4
= 65
3
Chetan’s score, c = (80) – 27
2
= 93

103
Exercise 4.1   5. The number of Singapore stamps in a stamp album is 23
Level 1 more than twice the number of British stamps. Find the
number of Singapore stamps in the album when there
1. A man’s monthly income is $3600. Find his savings when are
his expenditure is (a) 10 British stamps,
(a) $2500, (b) $3400, (b) p British stamps.
(c) $x.
Solution
Solution
(a) Number of Singapore stamps = 2 × 10 + 23
(a) His savings = $(3600 – 2500) = $1100 = 43
(b) His savings = $(3600 – 3400) = $200
(c) His savings = $(3600 – x) (b) Number of Singapore stamps = 2 × p + 23
= 2p + 23
2. Mr Lin is 5 cm taller than his wife. Find Mr Lin’s height
if the height of his wife is 6. The time taken by an aeroplane to travel from City A to
(a) 160 cm, (b) 168 cm, City B is 20 minutes more than one-eighth of the time
(c) h cm. taken by a train. Find the time taken by the aeroplane if
the time taken by the train is
Solution (a) 12 hours, (b) t hours.
(a) Mr Lin’s height = (160 + 5)
= 165 cm Solution
(a) Time taken by the aeroplane
(b) Mr Lin’s height = (168 + 5)
1 20
= 173 cm = × 12 +
8 60
(c) Mr Lin’s height = (h + 5) cm 3 1
= +
2 3
3. Find the number of days in 5
= 1 hours
(a) 5 weeks, (b) 12 weeks, 6
(c) n weeks.
(b) Time taken by the aeroplane
Solution 1 20
= ×t+
8 60
(a) Number of days in 5 weeks = 7 × 5
= 35 t 1
= + hours
8 3
(b) Number of days in 12 weeks = 7 × 12

= 84
7. Simplify the following.
(c) Number of days in n weeks = 7 × n
(a) a × 5 (b) b × b × 4
= 7n
(c) 2c ÷ d (d) e ÷ f × g
(e) 6h × 3k (f ) 9m ÷ 9n
4. John’s score is three quarters that of Mary’s score. (g) 3p × p × 5p (h) 4q × 5r × q
Find John’s score if Mary’s score is (i) s ÷ 6 + 1 × t ( j) u + 6v ÷ 9w
(a) 76, (b) 92, (c) s.
Solution
Solution (a) a × 5 = 5a (b) b × b × 4 = 4b2
3
(a) John’s score = × 76 2c eg
4 (c) 2c ÷ d = (d) e ÷ f × g =
d f
= 57
9m m
3 (e) 6h × 3k = 18hk (f ) 9m ÷ 9n = =
(b) John’s score = × 92 9n n
4
= 69 (g) 3p × p × 5p = 15p3 (h) 4q × 5r × q = 20q2r
3
(c) John’s score = ×s (i) s÷6+1×t=
s
+ t
4
6
3s
= 6v 2v
4 ( j) u + 6v ÷ 9w = u + =u+
9w 3w

Chapter 4  Introduction to Algebra


104
8. Express the following word statements algebraically. Solution
(a) Add 5 to the product of h and k. (a) Amount Mr Tan has to pay = $11 × (5 – 2)
(b) Subtract 3m from the quotient of n divided by p. = $33
(c) Divide the sum of 2t and 3u by v.
(b) Amount Mr Tan has to pay = $p × (n – 2)
(d) Multiply the product of y and z by 7y.
= $(n – 2)p
Solution
(a) hk + 5 13. There are 4 rotten eggs in a carton. The good eggs in
n the carton are shared among some households. Find the
(b) – 3m number of eggs that each household gets if there are
p
2t + 3u
(a) 100 eggs in the carton and 12 households,
(c) (2t + 3u) ÷ v = (b) n eggs in the carton and m households.
v

(d ) (yz) × 7y = 7y2z Solution


(a) Number of eggs per household = (100 – 4) ÷ 12
=8
Level 2
(b) Number of eggs per household = (n – 4) ÷ m
9. The price of a pineapple is $2 and the price of a n–4
watermelon is $5. Find the total price of =
m
(a) 5 pineapples and 1 watermelon,
(b) x pineapples and y watermelons. Level 3
14. A car salesman has a basic salary of $2000 a month. For
Solution every car sold, he gets $800 as commission. Let n be the
(a) Total price = $(2 × 5 + 5 × 1) number of cars that he sells in a month.
= $15 (a) Find his monthly salary when n = 18.
(b) Express his monthly salary in terms of n.
(b) Total price = $(2 × x + 5 × y)
= $(2x + 5y) Solution
(a) When n = 18,
10. The mass of Book A is 759 g and the mass of Book B his monthly salary = $(2000 + 800 × 18)
is 400 g. Find the total mass of = $16 400
(a) 4 copies of Book A and 3 copies of Book B, (b) His monthly salary = $(2000 + 800 × n)
(b) p copies of Book A and q copies of Book B. = $(2000 + 800n)
Solution
(a) Total mass = (759 × 4 + 400 × 3) 15. A grocer has p cartons of oranges. Each carton contains
= 4236 g q oranges and r of them are rotten. Express, in terms of
p, q and r,
(b) Total mass = (759 × p + 400 × q)
(a) the total number of oranges in all the cartons,
= (759p + 400q) g
(b) the total number of good oranges.

11. A lady has $500 in her pocket. Find the amount left if Solution
she spends (a) Total number of oranges = q × p = pq
(a) $120 on a skirt and $30 on her dinner, (b) Total number of good oranges = (q – r) × p
(b) $k on a skirt and $d on her dinner. = p(q – r)

Solution
(a) Amount left = $(500 – 120 – 30) 16. The length of a metal bar is L cm. It is melted and recast
= $350 into a thin bar whose length is k cm longer than n times
its original length. The thin bar is then cut into m equal
(b) Amount left = $(500 – k – d)
sticks. Express, in terms of k, L, m and n, the length of
(a) the thin bar, (b) each stick.
12. Mr Tan goes to a movie with his family. He has 2 free
tickets. Find the amount he has to pay for the tickets if Solution
(a) there are 5 family members and each ticket costs (a) Length of the thin bar = (L × n + k)
$11, = (nL + k) cm
(b) there are n family members and each ticket costs (b) Length of each stick = (nL + k) ÷ m
$p. nL + k
= cm
m

105
17. Mariani is x years old. Suhaini, her brother, is 19 years (c) Work out the approximate number of breaths you
older than her. Their mother is 3 times as old as Mariani. take per day, correct to the nearest thousand.
Their father is twice as old as Suhaini. Write down the
expressions, in terms of x, for Solution
(a) Suhaini’s age, (a) The number of breaths in a minute may be 15, 16,
(b) their mother’s age, 17, 18, 19 or 20.
(c) their father’s age. (b) (i) Number of breaths in an hour = n × 60
= 60n
Solution
(a) Suhaini’s age = (x + 19) years (ii) Number of breaths in a day = 60n × 24
= 1440n
(b) Their mother’s age = 3x years
(c) The approximate number of breaths taken per day
(c) Their father’s age = 2(x + 19) years varies.
For example, when n = 16,
18. A test paper consists of Section A and Section B with a 1440n = 1440 × 16
number of questions in each section. Find the total score = 23 040
of the paper in each case. = 23 000  (correct to the nearest 1000)
(a) Each question in Section A carries 5 marks while
each question in Section B carries 15 marks. There
are 8 questions in Section A and 4 questions in Exercise 4.2
Section B. Students are required to answer all the Level 1
questions. 1. Find the value of 3x – 1 when
(b) Each question in Section A carries a marks while 4
(a) x = 1, (b) x = .
each question in Section B carries b marks. There 5
are n questions in Section A and m questions in
Solution
Section B. Students are required to answer all the
(a) When x = 1,
questions.
3x – 1 = 3(1) – 1
(c) Each question in Section A carries a marks while
=2
each question in Section B carries b marks. There
are n questions in Section A and m questions in 4
(b) When x = ,
Section B. Students are required to answer all 5
the questions in Section A and (m – 1) out of m 4
questions in Section B. 3x – 1 = 3 –1
5
7
=
Solution 5
(a) Total score = (5 × 8 + 15 × 4)
= 100 marks 2. Find the value of 25 – 4y when
5
(b) Total score = (a × n + b × m) (a) y = 0, (b) y= .
2
= (an + bm) marks
(c) Total score = [a × n + b × (m – 1)] Solution
= [an + b(m – 1)] marks (a) When y = 0,
25 – 4y = 25 – 4(0)
= 25 – 0
19. Describe a real-life situation that could be represented
= 25
by the expression 2n + 4m.
5
(b) When y = ,
Solution 2
The number of legs of n chickens and m rabbits is 5
25 – 4y = 25 – 4
2n + 4m. 2
The number of wheels of n bicycles and m cars is = 25 – 10
2n + 4m.
= 15

20. (a) How many breaths do you take in a minute? 3. Find the value of 3a + 4b when
(b) Suppose you take n breaths in a minute, how many 1 1
(a) a = 7 and b = 5, (b) a = and b = .
3 2
breaths do you take in
(i)  an hour? (ii)  a day?

Chapter 4  Introduction to Algebra


106
Solution (e) When e = 2,
(a) When a = 7 and b = 5, 1
x = 3
3a + 4b = 3(7) + 4(5) e –1

= 21 + 20 1
= 3
= 41 2 –1
1
1 1 =
(b) When a = and b = , 7
3 2

3a + 4b = 3
1
+4
1
(f ) When f = –1,
3 2 3f + 4
=1+2 x = 5f – 2
=3 3(–1) + 4
= 5(–1) – 2
1
4. Find the value of (2m + n)(m – n + 1) when =–
7
(a) m = 5 and n = 3,
(b) m = 11 and n = 6.
6. Find the value of y in the following formulae.
(a) y = 3g + 4h; given g = 1, h = 5
Solution
(a) When m = 5 and n = 3, (b) y = m(3 + 2n); given m = 4, n = 6
(2m + n)(m – n + 1) = [2(5) + 3](5 – 3 + 1) p2 – q2
= 13 × 3 (c) y = ; given p = –7, q = 3
2
= 39 r s
(d) y = + ; given r = 6, s = –3
(b) When m = 11 and n = 6, s r
(2m + n)(m – n + 1) = [2(11) + 6](11 – 6 + 1) (e) y = uv2; given u = –2, v = –9
= 28 × 6
(tx)2
= 168 (f ) y = ; given t = 4, x = 2
3t – 5x

5. Find the value of x in the following formulae. Solution


(a) x = 2a + 1; given a = 3 (a) When g = 1, h = 5,
(b) x = 32 – 5b; given b = 4 y = 3g + 4h
(c) x = c2 – 2c; given c = 5 = 3(1) + 4(5)
(d) x = 4d(d + 5); given d = –8 = 23
1
(e) x = ; given e = 2 (b) When m = 4, n = 6,
e3 – 1
y = m(3 + 2n)
3f + 4
(f ) x = ; given f = –1 = 4[3 + 2(6)]
5f – 2
= 60
Solution (c) When p = –7, q = 3,
(a) When a = 3, p2 – q2
x = 2a + 1 y =
2
= 2(3) + 1 (–7)2 – 32
=7 =
2
(b) When b = 4, 40
=
x = 32 – 5b 2
= 32 – 5(4) = 20
= 12
(d) When r = 6, s = –3,
(c) When c = 5, r s
+ y =
x = c2 – 2c s r
= 52 – 2(5) 6 (–3)
= +
= 15 –3 6
1
(d) When d = –8, = –2
2
x = 4d(d + 5)
(e) When u = –2, v = –9,
= 4(–8)(–8 + 5)
y = uv2
= 96
= (–2) ×(–9)2
= –162

107
(f ) When t = 4, x = 2, Solution
(tx)2 (a) When x = 9, y = 3 and z = 6,
y = 3t – 5x x2 – 2xy 92 – 2(9)(3)
(4 × 2) 2 z3 – 4y2
= 63 – 4(3)2
= 3(4) – 5(2)
27
64 =
180
=
2 3
= 32 =
20

(b) When x = 8, y = –1 and z = –2,


Level 2 x2 – 2xy 82 – 2(8) (–1)
z3 – 4y2
=
7. Find the value of b2 – 4ac when (–2)3 – 4(–1)2

(a) a = 1, b = 5 and c = 3, 2
= –6
3 3
(b) a = 2, b = 7 and c = .
4 v2 – u2
11. If T = 2w
, find the value of T when u = 0, v = 5 and
Solution w = 4.
(a) When a = 1, b = 5 and c = 3,
b2 – 4ac = 52 – 4(1)(3) Solution
= 13 When u = 0, v = 5 and w = 4,
v2 – u2
3 T =
(b) When a = 2, b = 7 and c = , 2w
4
3 52 – 02
b2 – 4ac = 72 – 4(2) = 2×4
4
= 43
25
=
8
8. Find the value of p2q – (2q)2 when 1
(a) p = 8 and q = 3, =3
9 8
(b) p = and q = 4.
2

Solution
12. If z = kt n – 1, find the value of z when k = –2, t = 5 and
(a) When p = 8 and q = 3, n = 4.
p2q – (2q)2 = 82(3) – (2 × 3)2
Solution
= 156
When k = –2, t = 5 and n = 4,
9 z = kt n – 1
(b) When p = and q = 4,
2
2 = (–2) × 54 – 1
9
p2q – (2q)2 = (4) – (2 × 4)2 = (–2) × 125
2
= –250
= 17

Level 3
9. Find the value of a2 + 3b2 – c2 when
13. The daily wage of a worker is given by the expression
(a) a = 1, b = 2 and c = 3,
$15t, where t is the number of working hours.
(b) a = 5, b = –4 and c = 6.
(a) Find the daily wage of the worker when t = 8.
Solution
(b) What do you think the number 15 in the expression
(a) When a = 1, b = 2 and c = 3, stands for?
a2 + 3b2 – c2 = 12 + 3 × 22 – 32
Solution
=4
(a) When t = 8,
(b) When a = 5, b = –4 and c = 6, daily wage of worker = $15t
a2 + 3b2 – c2 = 52 + 3 × (–4)2 – 62 = $(15 × 8)
= 37 = $120

x2 – 2xy
(b) The number 15 stands for the hourly wage in dollars.
10. Find the value of z3 – 4y2
when

(a) x = 9, y = 3 and z = 6,
(b) x = 8, y = –1 and z = –2.

Chapter 4  Introduction to Algebra


108
1 2 Solution
14. The price of a square frame of side x cm is $ x + 5x .
4 (a) When x = 2 and y = 3,
Find the price of a square frame of side
F = 66x + 48y
(a) 10 cm, (b) 20 cm.
= 66(2) + 48(3)
= 276
Solution
The admission fee is $276.
(a) When x = 10,
1 2 (b) 66 is the price of an adult ticket in dollars.
Price of the frame = $ x + 5x
4 48 is the price of a child ticket in dollars.
1
=$ (10)2 + 5(10)
4
17. The body mass index (BMI), I, of a person is given by
m
= $75 the formula I = 2 , where m is the mass in kilograms
h
(b) When x = 20, and h is the height in metres of the person.
Price of the frame = $
1 2
x + 5x (a) Judy is 1.63 m high and weighs 51 kg. Find her
4 BMI.
1 (b) A BMI between 20 and 25 indicates a normal
=$ (20)2 + 5(20)
4 weight. A BMI lower than 20 suggests the person
= $200 is underweight while a BMI above 25 suggests
overweight. Evaluate your own BMI and see
whether it is within the range of normal mass.
15. The formula to convert a temperature of H °F (Fahrenheit)
to S °C (Celsius) is given by Solution
5 m
S= (H – 32). (a) I =
9 h2
(a) The boiling point of water is 212 °F. Express this When m = 51 and h = 1.63,
in °C. 51
(b) The temperature in New York at a certain moment I =
1.632
was 77 °F . Express this temperature in °C. = 19.2 (correct to 3 s.f.)
Judy’s BMI is 19.2.
Solution
(a) When H = 212, (b) Answer depends on student’s height and mass.
5
S = (H – 32)
9 18. The three sides of a triangle are a cm, b cm and
5 c cm respectively. The perimeter of the triangle is P cm.
= (212 – 32)
9 (a) Write a formula connecting a, b, c and P.
= 100 (b) Using the formula in (a), find the perimeter of
The boiling point is 100 °C. a triangle whose sides are 15 cm, 13 cm and
8 cm.
(b) When H = 77,
5 Solution
S = (77 – 32)
9 (a) P = a + b + c
= 25
(b) P = 15 + 13 + 8
The temperature in New York at a certain moment
= 36 cm
was 25 °C.

19. The price $P for a birthday cake of radius r cm and
16. The admission fee, $F, to the theme park Universal
height h cm is given by the formula
Studios Singapore for a family of x adults and y children
1 2
is given by P= r  h.
25
F = 66x + 48y.
(a) Find the price for a cake of radius 10 cm and height
8 cm.
(a) Find the total admission fee to the park for a family
(b) If the height of the cake in (a) is increased to
of 2 adults and 3 children.
10 cm, what is the increase in price?
(b) What do you think the numbers 80 and 50 in the
formula stand for?

109
Solution (b) Combining the 20 ohms and 30 ohms resistors first,
(a) When r = 10 and h = 8, we have an equivalent circuit as shown.
1 2
P = r  h 12 ohms
25
1
= × 102 × 8
25 X 15 ohms Y
= 32
The price of the cake is $32.
Take a = 12 and b = 15,
(b) When r = 10 and h = 10, ab
R =
1 2 a+b
P = rh 12 × 15
25
=
1 12 + 15
= × 102 × 10
25 180
=
= 40 27
Increase = $(40 – 32) 20
=
= $8 3
The increase in price is $8. 2
=6
3
2
The equivalent resistance is 6 ohms.
20 . (a) When two resistors of resistance a ohms and b 3
ohms are connected using different wires to two
points, X and Y, in a circuit as shown in Fig. 1,
Exercise 4.3
the equivalent resistance R ohms is given by the
formula Level 1
ab 1. The price of a ballpoint pen is $5 more than that of a
R = .
a+b ruler. If the price of the ruler is $x, express the price of
Find the value of R when a = 20 and b = 30. the ballpoint pen in terms of x.

a ohms Solution
Price of the ballpoint pen
= $(x + 5)
X b ohms Y

Fig. 1
2. The area of a kitchen is 12 m2 less than the area of a
sitting room. If the area of the sitting room is s m2,

express the area of the kitchen in terms of s.
(b) Suppose 3 resistors of 20 ohms, 30 ohms and
15 ohms are connected to the points X and Y in Solution
the circuit as shown in Fig. 2. Using the formula Area of the kitchen
in (a), find their equivalent resistance. = (s – 12) m2
20 ohms
3. A father’s age is 9 times that of his son. If the age of the
30 ohms son is y years old, express the age of the father in terms
of y.
X Y
15 ohms
Solution
Age of the father = 9y years
Fig. 2
4. The length of car A is 45 cm shorter than the length of
Solution car B. Let x cm and y cm be the lengths of car A and car
(a) When a = 20 and b = 30, B respectively. Express x in terms of y.
ab
R =
a+b Solution
20 × 30 x = y – 45
=
20 + 30
= 12
The value of R is 12.

Chapter 4  Introduction to Algebra


110
5. The speed of a car is 5 times as much as that of a bicycle. (b) When y = 24,
Let c km/h and b km/h be the speeds of the car and the 24
x = –3
bicycle respectively. Express c in terms of b. 2
= 9
Solution There are 9 boys.
c = 5b
10. Nadia pays $50 for 3 movie tickets. Let $x be the price
1
6. The mass of a cherry is that of an apple. Let c g and of each ticket and $y be the change she gets.
10
a g be the masses of the cherry and the apple respectively. (a) Write a formula connecting x and y.
Write a formula connecting a and c. (b) If the prices of each ticket is $9, find the amount
of change.
Solution
1 Solution
c = ×a (a) Total price of the tickets = $ 3x
10
c =
a \  y = 50 – 3x
10
(b) When x = 9,
y = 50 – 3 × 9
Level 2 = 23
7. The total price for a plate of chicken rice and a drink is The amount of change is $23.
$5. Let $x be the price of the chicken rice.
(a) Express the price of the drink in terms of x. 11. The present age of Laura is x years old.
(b) If the price of the chicken rice is $3.50, find the (a) Find her age 5 years ago in terms of x.
price of the drink. (b) Laura’s mother is 4 times as old as Laura 5 years
ago. Express the present age of Laura’s mother in
Solution terms of x.
(a) Price of the drink = $ (5 – x)
(b) Price of the drink = $ (5 – 3.50) Solution
= $ 1.50 (a) Laura’s age 5 years ago = (x – 5) years
(b) Age of Laura’s mother 5 years ago
8. The time taken for Peter to finish a mathematics = 4(x – 5) years
assignment is 10 minutes more than twice that of Mary. Present age of Laura’s mother
Let t minutes be the time taken by Mary. = [4(x – 5) + 5] years
(a) Express the time taken by Peter in terms of t.
(b) If Mary’s time taken is 25 minutes, find the time
taken by Peter. 12. Paula, Queenie and Rosa buy a gift for their father, and
they share the payment. Paula’s share is three times as
Solution much as that of Queenie. Rosa’s share is half of Paula’s
(a) Time taken by Peter share. Let $q be the amount of Queenie’s share.
= 2 × t + 10 (a) Express Paula’s share in terms of q.
= (2t + 10) min (b) Express Rosa’s share in terms of q.
(c) If Queenie pays $60, find the price of the gift.
(b) Time taken by Peter
= (2 × 25 + 10) min Solution
= 60 min (a) Paula’s share = $3q
1
(b) Rosa’s share = $ (3q)
9. In a dancing club, the number of boys is 3 less than half 2
of the number of girls. Let x and y be the numbers of 3
=$ q
boys and girls in the club respectively. 2
(a) Express x in terms of y.
(c) If Queenie pays $60, then q = 60.
(b) If there are 24 girls, find the number of boys.
3
Price of the gift = $(3q + q + q)
2
Solution 3
1
= $(3 × 60 + 60 + × 60)
2
(a) x = ×y–3
2 = $330
y
x= –3
2

111
Level 3 Solution
(a) (i) Number of 2-point scores made = 3x + 5
13. There are some $2 notes, $10 notes and $50 notes in Mr
4
Singh’s wallet. The number of $50 notes is 1 more than (ii) Number of 3-point scores made = x
5
twice the number of $2 notes. The number of $10 notes
is 1 less than the number of $2 notes. Let x, y and z be (iii) Total score in the match
4
the numbers of $2, $10 and $50 notes respectively. = x + 2(3x + 5) + 3 × x
5
(a) Express y and z in terms of x.
12
(b) Express the total amount of money in terms of x. = x + 2(3x + 5) + x
5
(c) If there are 4 $2 notes, find the total amount of
money. (b) When x = 10,
Total score in the match
Solution 12
= 10 + 2 (3 × 10 + 5) + × 10
(a) y = x – 1 5
z = 2x + 1 = 104
(b) Total amount of money
= $ (2x + 10y + 50z) 16. A concert has two types of tickets. Adult tickets are sold
= $ [2x + 10(x – 1) + 50(2x + 1)] at $75 each and student tickets are sold at $40 each. The
total number of tickets sold is 1200. Define a variable
(c) When x = 4,
and express the total income by an algebraic expression.
Total amount of money
= $ [2(4) + 10(4 – 1) + 50(2 × 4 + 1)]
Solution
= $ 488
Let x be the number of adult tickets sold.
Then, number of student tickets sold
14. In making a square flowerbed, the cost for fencing is = 1200 – x
$20/m and the cost for flower planting is $50/m2. Let x Total income
m be the length of a side of the flowerbed. = $[75x + 40(1200 – x)]
(a) If $F is the cost of making the fence, find a formula
connecting F and x.
(b) If $P is the cost for planting the whole flowerbed, Revision Exercise 4
find a formula connecting P and x. 1. Simplify the following.
(c) If $T is the total cost of making the flowerbed, find (a) 5s × 3t + 1 × u
a formula connecting T and x. (b) m – 4n × 6m × m
(c) (a × 4 – b × b) ÷ 2c
Solution
(d) 3x – b ÷ c – 5 × y
(a) F = 20 × 4 × x
\  F = 80x Solution
(b) P = 50 × x 2 (a) 5s × 3t + 1 × u = 15st + u
\  P = 50x2 (b) m – 4n × 6m × m = m – 24m2n
(c) T = F + P 4a – b2
\  T = 80x + 50x2 (c) (a × 4 – b × b) ÷ 2c =
2c
b
(d) 3x – b ÷ c – 5 × y = 3x – – 5y
15. In an NBA basketball match, the number of 2-point scores c
that a team made is 5 more than 3 times of its 1-point
scores. The number of 3-point scores is four-fifth of the 2. Express the following word statements algebraically.
number of 1-point scores. Let x be the number of 1-point (a) Subtract c × c from d × 5.
scores that the team made. (b) Divide x cubed by y squared.
(a) Express, in terms of x, (c) Divide the product of 6a and 4 by 8b.
(i) the number of 2-point scores made,
(ii) the number of 3-point scores made, Solution
(iii) the total score in the match. (a) Expression
=d×5–c×c
(b) If the team had 10 1-point scores in the match, find = 5d – c2
its total score in the match. x3
= x3 ÷ y2 =
(b) Expression
y2

Chapter 4  Introduction to Algebra


112
(c) Expression
= (6a × 4) ÷ 8b 7. The capacity of a car is 5 passengers and that of a van
24a is 8 passengers. Find the total capacity for m cars and n
= vans.
8b
3a
= Solution
b
Total capacity = (5 × m + 8 × n)
= (5m + 8n) passengers
3. Find the values of the following expressions when
n = 10.
8. A mathematics book has 30 pages less than twice the
Solution number of pages of a history book. Let m and h be the
(a) When n = 10, numbers of pages of the mathematics book and the history
3n + 8 book respectively. Write a formula connecting m and h.
= 3(10) + 8
= 38 Solution
m = 2h – 30
(b) When n = 10,
9(1 – 2n)
= 9(1 – 2(10)) 9. Ali is p years old now.
= –180 (a) Find his age 7 years ago.
(b) Find his age in t years’ time.
(c) Ali’s father is 3 years older than 4 times Ali’s age.
4. Find the value of the expression (2a + 3b) 2 when Express his father’s age in terms of p.
a = 1 and b = –2. (d) If p = 8, find his father’s age.
Solution Solution
When a = 1 and b = –2, (a) Ali’s age 7 years ago = (p – 7) years
(2a + 3b)2 = [2(1) + 3(–2)]2
= [–4]2 (b) Ali’s age in t years’ time = (p + t) years
= 16 (c) His father’s age = (4 × p + 3)
= (4p + 3) years
1
5. Given the formula E = m(v2 – u2), find the value of E (d) When p = 8,
2
when m = 5, v = 11 and u = 7. his father’s age = (4 × 8 + 3)
= 35 years
Solution
When m = 5, v = 11 and u = 7,
10. There are x boys and y girls in a class. Half of the boys
1
E = m(v2 – u2) 1
2 and of the girls join a learning camp.
3
1
= × 5 × (112 – 72) (a) Express, in terms of x and y,
2
(i) the total number of students in the class,
= 180 (ii) the total number of students joining the camp.
(b) When x = 18 and y = 24, find the number of students
a – 3b2
6. Given the formula y = , find the value of y when joining the camp.
(a – 3b)2
a = 10 and b = 2.
Solution
(a) (i) Total number of students in class
Solution
=x+y
When
a = 10 and b = 2,
a – 3b2 (ii) Total number of students joining camp
y = 1 1
(a – 3b)2 = x + y
2 3
10 – 3 × 22
= (b) Number of students joining camp
(10 – 3 × 2)2
1 1
–2 = (18) + (24)
= 2 3
16 =9+8
= –
1 = 17
8

113
11. John’s savings after n months is $(2500 + 300n).
(a) Find the amount of savings he has after
(i) 5 months, (ii) 1 year.
(b) After n months, John uses all his savings to buy
gold coins costing $g each.
(i) Express the number of gold coins he buys in
terms of g and n.
(ii) When g = 100 and n = 6, find the number of
gold coins John buys.

Solution
(a) (i) When n = 5,
amount of savings John has
= $(2500 + 300 × 5)
= $4000
(ii) 1 year = 12 months
When n = 12,
amount of savings John has
= $(2500 + 300 × 12)
= $6100
(b) (i) Number of gold coins he buys
2500 + 300n
=
g

(ii) When g = 100 and n = 6,


number of gold coins he buys
2500 + 300 × 6
=
100
= 43

12. The density of zinc is 135 kg/m­3 more than 7 times the
density of water. The density of copper is 70 kg/m3 less
than 9 times the density of water. Let x kg/m3 ­be the
density of water.
(a) Express the density of zinc in terms of x.
(b) Express the density of copper in terms of x.
(c) A piece of brass, made up of zinc and copper,
contains 0.02 m3 ­of zinc and 0.03 m3 ­of copper.
Let m kg be the mass of the piece.
(i) Find a formula connecting m and x.
(ii) If x = 1000, find the mass of the piece.

Solution
(a) Density of zinc = (7x + 135) kg/m3
(b) Density of copper = (9x – 70) kg/m3
(c) (i) m = 0.02(7x + 135) + 0.03(9x – 70)
(ii) When x = 1000,
m = 0.02(7 × 1000 + 135) + 0.03(9 × 1000 – 70)
= 410.6
The mass of the piece is 410.6 kg.

Chapter 4  Introduction to Algebra


114
Review Exercise 1 (b)
1
–2 – 4
3
÷ (–3)3 + 7
1
3 4 6
1. (a) Express 126 and 132 as products of prime factors.
(b) Find the HCF and LCM of 126 and 132. 7 19 43
= – – ÷ –27 +
3 4 6
Solution
–28 – 57 –162 + 43
(a) 126 = 2 × 32 × 7 =
12
÷
6
132 = 22 × 3 × 11
–85 –119
(b) HCF = 2 × 3 = ÷
12 9
=6
LCM = 22 × 32 × 7 × 11 85 –119
= ×
= 2772 12 9

5
2. Let a = 4116 and b = 2 × 34 × 72 × 11. =
14
(a) Express a as a product of its prime factors.
(b) Find the HCF and LCM of a and b.
5. The temperature of a room is m °C and the temperature of
Solution a freezer is –n °C, where m and n are positive numbers.
(a) The temperature of an oven is 150 °C higher than three
4116 times the temperature of the room. Express, in terms of
m and n,
6 686 (a) the difference between the two temperatures of the
room and the freezer,
2 3 14 49 (b) the average of the two temperatures of the room
and the freezer,
2 7 7 7 (c) the temperature of the oven.
2 3
\  a = 4116 = 2 × 3 × 7 Solution
(b) b = 2 × 34 × 72 × 11 (a) The difference = m – (–n)
HCF of a and b = 2 × 3 × 72 = (m + n)°C
= 294
1
LCM of a and b = 22 × 34 × 73 × 11 (b) The average = × [m + (–n)]
2
= 1222452 m–n
= °C
2
1
3. Given the three numbers –3, 4 and 5 ,
(c) Temperature of the oven
2
(a) represent the numbers on a number line, = (3m + 150)°C
(b) arrange the numbers in ascending order.
4
6. (a) Express as
Solution 7
1
(a) The numbers –3, 4 and 5 are shown on the (i) a recurring decimal,
2
following number line. (ii) a decimal correct to 3 significant figures.
1
–3 4 5–2 3
17 – 1.236
(b) Using a calculator, evaluate , giving your
–3 –2 –1 0 1 2 3 4 5 6 1.852
answer correct to 4 significant figures.
1
(b) The numbers in ascending order are –3, 4, 5 .
2 Solution
4
(a) (i) = 0.571 428
7
4. Evaluate each of the following.
4
(a) (–16) + (–3) 3 [17 – (–5)2] (ii) = 0.571  (correct to 3 s.f.)
7
1 3 1
(b) –2 – 4 ÷ (–3)3 + 7 3
17 – 1.236
3 4 6 (b) = 0.003 901  (correct to 4 s.f.)
1.852
Solution
(a) (–16) + (–3) × [17 – (–5)2]
= (–16) + (–3) × [17 – 25]
= (–16) + (–3) × (–8)
= (–16) + 24
= 8

115
7. In a supermarket, a bottle of shampoo costs $7.95, a can Solution
of pineapple costs $3.20 and a piece of mutton costs (a) When a = 456, b = 2 and t = 3,
$13.40. P = abt
(a) Estimate the total price of these three items by = 456 × 23
rounding off each price correct to the nearest dollar. = 3648
(b) If Mrs Foo has $24.50 in her pocket, can the
(b) P = 3600  (correct to the nearest hundred)
estimated total price in (a) guarantee that she has
enough money to buy these three items. Explain
11. (a) Express 315 as a product of its prime factors.
your answer.
(b) John has 315 pieces of square tiles, each of side
30 cm long.
Solution
(i) Find the minimum number of such tiles he
(a) Total price
should buy so that he can lay a large square
= $ (7.95 + 3.20 + 13.40)
with all the tiles.
≈ $ (8 + 3 + 13) (Each number correct to the nearest
(ii) Find the area of the large square thus formed.
integer)
(c) There are 315 apples and 168 oranges. Each kind
= $24
of fruit is divided into small packs of equal number
(b) The estimated price in (a) CANNOT guarantee that of fruit. Find the greatest possible number of fruit
Mrs Foo has enough money to buy the 3 items. in a pack.
This is because the rounding amount may be more
than $0.5. Solution
(a) 315 = 32 × 5 × 7
8. The price of an apartment is $408 995. (b) (i) 315 = 17.7  (correct to 3 s.f.)
(a) Find the price of the apartment correct to the nearest The perfect square just greater than 315 is
hundred. 182, i.e. 324.
(b) Round off the price correct to 2 significant figures. \  the minimum number of tiles
= 324 – 315
Solution = 9
(a) The price = $408 995
(ii) Area of the large square
= $409 000
= 30 × 30 × 324
(b) The price = $408 995 = 291600 cm2
= $410 000  (correct to 2 s.f.)
(c) 168 = 23 × 3 × 7
HCF of 315 and 168 = 3 × 7
9. (a) Express the product of x and the sum of 2y and 3z = 21
in terms of x, y and z. Hence, the greatest number of fruit in a pack is 21.
(b) If x = 5, y = – 4 and z = 2, find the value of the
product.
12. There is a group of soldiers. When they are lined in rows
of 6, 8 or 15, there are 5 soldiers left.
Solution
(a) Find the minimum number of soldiers in the
(a) Product = x(2y + 3z)
group.
(b) When x = 5, y = –4 and z = 2, (b) If the group can be exactly divided into small teams
product = 5[2(–4) + 3(2)] of 13 soldiers each, find the minimum number of
= –10 soldiers in the group.

Solution
10. The population P of bacteria in a colony at time t hours
(a) 6 = 2 × 3
is given by the formula P = abt, where a and b are
8 = 23
constants.
15 = 3 × 5
(a) Find the value of P when a = 456, b = 2 and
LCM of 6, 8 and 15 = 23 × 3 × 5
t = 3.
= 120
(b) Round off the answer in (a) correct to the nearest
hundred. The minimum number of soldiers
= 120 + 5
= 125

Review Exercise 1
116
(b) The possible number of soldiers is 120 × n + 5, 14. There was a traffic congestion of 100 cars in the
where n is a positive integer. queue at Pan Island Expressway (PIE) and these
When n = 1, 120 × 1 + 5 = 125. 100 cars were at rest. Assume that each car was
When n = 2, 120 × 2 + 5 = 245. x m long and the gap between any two adjacent cars was
When n = 3, 120 × 3 + 5 = 365. y m. Let z m be the distance from the front of the first
When n = 4, 120 × 4 + 5 = 485. car to the end of the last car.
When n = 5, 120 × 5 + 5 = 605. (a) Express z in terms of x and y.
When n = 6, 120 × 6 + 5 = 725. (b) If x = 4.5 and y = 0.6, find the value of z.
When n = 7, 120 × 7 + 5 = 845. 1 4
(c) If x = 3 and y = , find the value of z.
2 5
The numbers 125, 245, 365, 485, 605 and 725 are
(d) Which estimate of z, (b) or (c), was closer to the
not divisible by 13, while 845 is divisible by 13.
reality? Explain briefly.
\  the minimum number of soldiers is 845.
Solution
(a) z = 100x + 99y
13. In September, the local time of New York and that of
Singapore was GMT– 4 and GMT+8 respectively. (b) z = 4.5 × 100 + 0.6 × 99
(a) When it is 13 00 on 2 September in Singapore, = 509.4
what is the local time in New York? 1 4
(b) An aeroplane takes 19 hours to fly between New (b) z = 3 × 100 + × 99
2 5
York and Singapore. 1
(i) If it departs from New York at 23 00 on = 429
5
3 September, find its arrival time and date in
Singapore. (c) The length of a car is usually between 4 m and
(ii) If it departs from Singapore at 10 55 on 5 m. Hence, the estimate in (b) is closer to the
27 September, find its arrival time and date reality.
in New York.
15. Gymnasium A has a monthly fee of $20 and the charge
Solution per visit is $5. Gymnasium B has no monthly fee and the
(a) Difference in time zones = 8 – (–4) charge per visit is $8. Suppose that you go to a gymnasium
= 12 for exercises n times in a month. Let $C1 and $C2 be the
total monthly expenditure if you visit gymnasiums A and
\ the local time in New York is 12 hours behind B respectively.
the local time in Singapore. (a) Express C1 in terms of n.
When it is 13 00 in 2 September in Singapore, the (b) Express C2 in terms of n.
local time in New York (c) Find the values of C1 and C2 when n = 6.
= 13 00 – 12 00 (d) Find the possible values of n such that C2 , C1.
= 01 00 on 2 September
Solution
(b) (i) On arrival in Singapore, (a) C1 = 20 + 5n
local time in New York
= 23 00 on 3 September + 19 (b) C2 = 8n
= 18 00 on 4 September (c) When n = 6,
local time in Singapore C1 = 20 + 5 × 6
= 18 00 on 4 September + 12 = 50
= 06 00 on 5 September C2 = 8 × 6
= 48
(ii) On arrival in New York,
local time in Singapore
= 10 55 on 27 September + 19
= 05 55 on 28 September
local time in New York
= 05 55 on 28 September – 12
= 17 55 on 27 September

117
(d)
n C1 C2
0 20 0
1 25 8
2 30 16
3 35 24
4 40 32
5 45 40
6 50 48
7 55 56

The above table shows that C2 < C1 when n = 0,


1, 2, 3, 4, 5 and 6.

Review Exercise 1
118
5 Algebraic Manipulation

Class Activity 1
Objective: To apply the process of collecting like terms using the AlgeDiscTM application in AlgeToolsTM.

In the AlgeDiscTM, apart from number discs, we have the x-discs and y-discs as follows.

1 –1 x –x y –y

front back front back front back

Each disc has two sides. We can flip a disc to show either the front or the back. Similar to 1 and –1, x and –x as well as y and
–y form zero pairs. That is 1 + (–1) = 0, x + (–x) = 0 and y + (–y) = 0. Examples of zero pairs are:

(a) 2x + (–2x) = 2x – 2x = 0 (b) (–3y) + 3y = –3y + 3y = 0

x x –y –y –y

–x –x y y y

zero pairs zero pairs

Some algebraic expressions can be represented by a set of algebra discs. For example,

(a) 2x + 3: x x 1 1 1 This set of algebra discs can also



represent 3 + 2x.
(b) y – 4: y –1 –1 –1 –1 This set of algebra discs can also
represent – 4 + y.
(c) –x + 3y – 2: –x y y y –1 –1 This set of algebra discs can also
represent –2 + 3y – x, –2 – x + 3y
and 3y – x – 2.
Terms involving x and y can be simplified by collecting like terms as illustrated below.
zero pairs
x x x x

y y y –x –x –x –x –x –x –x –x
➜ ➜
–x –x –x –x –x y y y y y y

y y y

2x + 3y – 5x + y Collect the like terms –3x + 4y


2x + 3y – 5x + y

= 2x – 5x + 3y + y

–3x 4y
\ 2x + 3y – 5x + y = 2x – 5x + 3y + y
= –3x + 4y

119
Tasks
1. Represent the following algebraic expressions using the algebra discs in the AlgeDiscTM application.

(a) 3x + 2:
x x x 1 1


(b) x + 4y:
x y y y y


(c) –2x – 3y + 4:
–x –x –y –y –y 1 1 1 1

2. With the aid of the AlgeDiscTM application, simplify each of the following:

(a) 2x + 5x

x x

x x x x x

\ 2x + 5x = 7x

(b) 2x – 5x
zero pairs

x x

–x –x –x –x –x

\ 2x – 5x = –3x

(c) –2x + 5x
zero pairs

–x –x

x x x x x

\  –2x + 5x = 3x

(d) –2x – 5x

–x –x

–x –x –x –x –x

\  –2x – 5x = –7x

Chapter 5  Algebraic Manipulation


120
(e) 3y + 1 + 4y + 3

y y y y y y

1 y y y y

y y y y 1

1 1 1 1 1 1

3y + 1 + 4y + 3 Collect the like terms
3y + 4y + 1 + 3

\ 3y + 1 + 4y + 3 = 3y + 4y + 1 + 3
= 7y + 4

(f) 3y + 1 – 4y – 3
zero pairs
y y y y y y

1 –y –y –y –y
➜ ➜
–y –y –y –y 1 –y

–1 –1 –1 –1 –1 –1 –1 –1

3y + 1 – 4y – 3 Collect the like terms –y – 2
3y – 4y + 1 – 3

\ 3y + 1 – 4y – 3 = 3y – 4y + 1 – 3
= –y – 2

(g) –3y – 1 + 4y + 3
zero pairs
–y –y –y –y –y –y

–1 y y y y
➜ ➜
y y y y –1 y

1 1 1 1 1 1 1 1

–3y – 1 + 4y + 3 Collect the like terms y+2
–3y + 4y – 1 + 3

\  –3y – 1 + 4y + 3 = –3y + 4y – 1 + 3
=y+2

121
(h) –3y – 1 – 4y – 3

–y –y –y –y –y –y

–1 –y –y –y –y

–y –y –y –y –1

–1 –1 –1 –1 –1 –1

–3y – 1 – 4y – 3 Collect the like terms
–3y – 4y – 1 – 3

\  –3y – 1 – 4y – 3 = –3y – 4y – 1 – 3
= –7y – 4

(i) 4x – 5y + x + 2y – 1

y y y y

–y –y –y –y –y x x x x x

x ➜ –y –y –y –y –y

y y y y
zero pairs
–1 –1

4x – 5y + x + 2y – 1 Collect the like terms
5x – 5y + 2y – 1

x x x x x

➜ –y –y –y

–1

5x – 3y – 1

\ 4x – 5y + x + 2y – 1 = 5x – 5y + 2y – 1
= 5x – 3y – 1

Chapter 5  Algebraic Manipulation


122
(j) –3x + 2y – 4 – x + y + 1
–x –x –x

y y

–1 –1 –1 –1 –x –x –x –x

–x y y y –x –x –x –x
➜ ➜
y –1 –1 –1 –1 y y y

1 1 –1 –1 –1

zero pairs
–3x + 2y – 4 – x + y + 1 Collect the like terms –4x + 3y – 3
–4x + 3y – 4 + 1

\  –3x + 2y – 4 – x + y + 1 = –4x + 3y – 4 + 1
= –4x + 3y – 3

Questions
1. Explain how you would simplify ax + bx, where a and b are given integers. You may substitute a and b with different
integers to help you illustrate your explanation.

ax + bx = (a + b)x

2. Can you write 3x + 4y as 7xy? Explain your answer. You may substitute x and y with some numbers to help you explain.

3x and 4y are unlike terms. Therefore, 3x + 4y cannot be simplified.

123
Class Activity 2
Objective: To perform addition and subtraction of linear expressions using the AlgeDiscTM application in AlgeToolsTM.

Tasks
1. To obtain the negative of an expression, we flip the discs that represent the expression.
For example, simplify –(3x – 2).

– x x x –1 –1 ➜ –x –x –x 1 1

It means flipping over all the discs inside the box.

\  –(3x – 2) = –3x + 2 
It means changing signs of all the terms inside the bracket.
Thus, 3x becomes –3x and –2 becomes 2.  

Simplify the following expressions using the AlgeDiscTM application.

(a) –(2x + 4)

– x x 1 1 1 1 ➜ –x –x –1 –1 –1 –1

\  –(2x + 4) = –2x – 4

(b) –(y – 3)

– y –1 –1 –1 ➜ –y –x 1 1 1

\  –(y – 3) = –y + 3

(c) –(–3x + 2y)

– –x –x –x y y ➜ x x x –y –y

\  –(–3x + 2y) = 3x – 2y

(d) –(x – 4y + 2)

– x –y –y –y –y 1 1 ➜ –x y y y y –1 –1

\  –(x – 4y + 2) = –x + 4y – 2

Chapter 5  Algebraic Manipulation


124
2. To add two expressions, we remove the brackets and collect the like terms.
For example, simplify (2x – 3y) + (–3x + y).

zero pairs
x x x x

–y –y –y –x –x –x –x
➜ ➜
–x –x –x –y –y –y –y –y

y y

(2x – 3y) + (–3x + y) Collect the like terms –x – 2y

(2x – 3y) + (–3x + y)


= 2x – 3y – 3x + y    The signs of all the terms inside the bracket remain unchanged.
= 2x – 3x – 3y + y
= –x – 2y

Simplify the following expressions using the AlgeDiscTM application.



(a) (3x + 1) + (–x + 2)
zero pairs
x x x

x x x x –x x x
➜ ➜
–x 1 1 1 1 1 1 1 1

(3x + 1) + (–x + 2y) Collect the like terms 2x + 3

\ (3x + 1) + (–x + 2y) = 3x + 1 – x + 2


= 3x – x + 1 + 2
= 2x + 3

(b) (–2x + y) + (–3x – y)


zero pairs
y

–x –x y –y

–x –x –x –y ➜ –x –x –x –x –x ➜ –x –x –x –x –x

(–2x + y) + (–3x – y) Collect the like terms –5x

\ (–2x + y) + (–3x – y) = –2x + y – 3x – y


= –2x – 3x + y – y
= –5x

125
(c) (4y – 3) + (y – 1)

y y y y –1 –1 –1 y y y y y

y –1 –1 –1 –1 –1

(4y – 3) + (y – 1) Collect the like terms

\ (4y – 3) + (y – 1) = 4y – 3 + y – 1
= 4y + y – 3 – 1
= 5y – 4

(d) (–3x – y + 1) + (–2x + 3y + 4)


zero pairs
y y y

–y

–x –x –x –y 1 –x –x –x –x –x

–x –x y y y 1 1 1 1 1 1 1 1 1

(–3x – y + 1) + (–2x + 3y + 4) Collect the like terms

y y

➜ –x –x –x –x –x

1 1 1 1 1

–5x + 2y + 5

\ (–3x – y + 1) + (–2x + 3y + 4) = –3x – y + 1 – 2x + 3y + 4


= –5x + 2y + 5

3. Subtracting an expression is the same as adding the negative of that expression.


For example, simplify (2x – 3y) – (–3x + y).

Simplify the following expressions using the AlgeDiscTM application.

x x
x x x x
x x x x x x x x
–y –y –y –y –y –y ➜ ➜
–y –y –y –y –y –y –y
– –x
x –x –x y ➜ x x x –y
–y
(2x – 3y) – (–3x + y) Remove brackets Collect the like terms 5x – 4y

(2x – 3y) – (–3x + y)
= 2x – 3y + 3x – y    Observe that all the terms inside the bracket change signs after the bracket is removed.
= 2x + 3x –3y – y
= 5x – 4y

Chapter 5  Algebraic Manipulation


126
(a) 3x – (–5x)

x x x x x x

– –x
x –x –x –x –x x x x x x

3x – (–5x) Remove brackets

\ 3x – (–5x) = 3x + 5x
= 8x

(b) (2x – 1) – (3x + 2)

x x –1 x x –1

– x x x 1 1 –x –x –x –1 –1

(2x – 1) – (3x + 2) Remove the brackets

zero pairs
x x

–x –x –x –x
➜ ➜
–1 –1 –1 –1 –1 –1

Collect the like terms –x – 3

\ (2x – 1) – (3x + 2) = 2x + 1 – 3x – 2
= –x – 3

(c) (–2x + 3y) – (x – 2y)

–x –x y y y –x –x y y y

– –x
x –y –y x y y

(–2x + 3y) – (x – 2y) Remove the brackets

zero pairs
–x –x

x –x
➜ ➜
y y y y y x y y y y

Collect the like terms –x – 3

\ (–2x + 3y) – (x – 2y) = –2x + 3y – x + 2y


= –3x + 5y

127
(d) (–3x – y + 4) – (–2x + y – 1)

–x –x –x –y 1 1 1 1 –x –x –x –y 1 1 1 1

– –x
x –x y –1 x x –y 1

(–3x – y + 4) – (–2x + y – 1) Remove the brackets

zero pairs
–x –x –x

x x –x

–y –y –y –y

1 1 1 1 1 1 1 1 1 1

Collect the like terms –x – 2y + 5

\ (–3x – y + 4) – (–2x + y – 1) = –3x – y + 4 + 2x – y + 1


= –x – 2y + 5

Class Activity 3
Objective: To perform multiplication of linear expressions and simplify them using the AlgeDiscTM application in AlgeToolsTM.

Tasks
1. Given an algebraic expression, we can group its terms and then multiply it by an integer.
For example, expand –3(4x).

– x x x x –x –x –x –x

– x x x x ➜ –x –x –x –x

– x x x x –x –x –x –x
–3(4x) –12x

–3(4x) = –(3 groups of 4x) = –(3 × 4x) = –(12x) = –12x

The ‘–’ sign means flip over all the algebra discs or change the sign
of the term in the bracket.

\  –3(4x) = –12x

Chapter 5  Algebraic Manipulation


128
For example, expand –2(–5x).

– –x –x –x –x –x x x x x x

– –x –x –x –x –x x x x x x

– 2(–5x) 10x

It means 2 groups of –5x which give a total of –10x.


The ‘–’ sign means flipping over the algebra discs or chang-
ing the signs of the term –10x.
\  –2(–5x) = 10x

Expand the following using the AlgeDiscTM application.

(a) 3(4x)

x x x x x x x x

x x x x ➜ x x x x

x x x x x x x x

3(4x) 12x

\ 3(4x) = 12x

(b) 3(–4x)

–x –x –x –x –x –x –x –x

–x –x –x –x ➜ –x –x –x –x

–x –x –x –x –x –x –x –x
3(–4x) –12x

\ 3(–4x) = –12x

(c) –3(–4x)

– –x –x –x –x x x x x

– –x –x –x –x ➜ x x x x

– –x –x –x –x x x x x

–3(4x) –12x

\  –3(–4x) = 12x

129
(d) 2(5x)

x x x x x x x x x x

x x x x x x x x x x

2(5x) 10x

\ 2(5x) = 10x

(e) 2(–5x)

–x –x –x –x –x –x –x –x –x –x

–x –x –x –x –x –x –x –x –x –x
2(–5x) –10x

\ 2(–5x) = –10x

(f) –2(–5x)

– –x –x –x –x –x x x x x x

– –x –x –x –x –x x x x x x

–2(–5x) 10x

\  –2(–5x) = 10x

2. The expansion of a linear expression can be done in a similar way.


For example, expand 2(3x – 1).

x x x –1 x x x –1
➜ 2(3x – 1) = 6x – 2
x x x –1 x x x –1
2(3x – 1) 6x – 2

The means 2 groups of 3x – 1.


\ 2(3x –1) = 6x – 2

Expand the following using the AlgeDiscTM application.

(a) –2(3x – 1)

– x x x –1 –x –x –x 1

– x x x –1 –x –x –x 1

–2(3x – 1) –6x + 2

\  –2(3x –1) = –6x + 2

Chapter 5  Algebraic Manipulation


130
(b) –2(–3x – 1)

– –x –x –x –1 x x x 1

– –x –x –x –1 x x x 1

–2(–3x – 1) 6x – 2

\  –2(–3x –1) = 6x + 2

(c) 3(x + 2y)

x y y x y y

x y y ➜ x y y

x y y x y y

3(x + 2y) 3x + 6y

\ 3(x + 2y) = 3x – 6y

(d) –3(x + 2y)

– x y y –x –y –y

– x y y ➜ –x –y –y

– x y y –x –y –y
–3(x + 2y) –3x – 6y

\  –3(x + 2y) = –3x – 6y

3. In general, an expression can be simplified by first removing the brackets and then collecting the like terms.
For example, simplify 2(3x – 2) – 3(–x – 1).

x x x –1 –1
x x x –1 –1
2(3x – 2) = 6x – 4
x x x –1 –1
x x x –1 –1

– –x –1 ➜ x 1
–3(–x – 1) = 3x + 3
– –x –1 x 1

– –x –1 x 1

2(3x – 2) – 3(–x – 1) Remove brackets


131
x x x x x x x x x x x x

x x x x x x
➜ ➜
–1 –1 –1 –1 –1

1 1 1
zero pairs
Collect the like terms 9x –1

2(3x – 2) – 3(–x – 1)
= 6x – 4 + 3x + 3    
We change the signs of all terms in the bracket.
= 6x + 3x – 4 + 3
= 9x –1

For example, simplify (5y + 2) + 3(–2y + 1).

y y y y y y y y y y

1 1 1 1

–y –y 1 ➜ –y –y 1

3(–2y + 1) = –6y + 3
–y –y 1 –y –y 1

–y –y 1 –y –y 1

(5y + 2) + 3(–2y + 1) Remove brackets


zero pairs

y y y y y

–y –y –y –y –y –y –y
➜ ➜
1 1 1 1 1 1 1

1 1 1

Collect the like terms –y + 5


(5y +2) + 3(–2y + 1)   The signs of all terms in the bracket remain unchanged after the bracket is removed.
= 5y + 2 – 6y + 3
= 5y – 6y + 2 + 3
= –y + 5

Chapter 5  Algebraic Manipulation


132
For example, simplify –2(2x + y) – 4(–x + y).

– x x y –x –x –y –2(2x + y) = – 4x – 2y
Since there is a ‘–’ sign in front of the
bracket, all terms inside the bracket will
– x x y –x –x –y change sign after the bracket is removed.

– –x y x –y

– –x y x –y – 4(–x + y) = 4x – 4y
Since there is a ‘–’ sign in front of the
bracket, all terms inside the bracket will
– –x y x –y change sign after the bracket is removed.

– –x y x –y

–2(2x + y) – 4(–x + y) Remove brackets



zero pairs
–x –x –x –x

x x x x –y –y
➜ ➜
–y –y –y –y
–y –y

–y –y –y –y
Collect the like terms –6y

–2(2x + y) – 4(–x + y) = –4x – 2y + 4x – 4y  The signs of the terms in the brackets are all changed.
= –4x + 4x – 2y – 4y
= –6y

Simplify the following using the AlgeDiscTM application.

(a) 3(–2x + 1) + 2(4x – 1)

–x –x 1 –x –x 1

–x –x 1 –x –x 1

–x –x 1 ➜ –x –x 1

x x x x –1 x x x x –1

x x x x –1 x x x x –1

3(–2x + 1) + 2(4x – 1) Remove brackets

133
zero pairs

–x –x –x –x –x –x

x x x x x x x x


1 1 1 ➜ x x

–1 –1 1

Collect the like terms 2x + 1

\ 3(–2x + 1) + 2(4x – 1) = 2x + 1

(b) 2(3x – 2) – 3(–x – 2)

x x x –1 –1 x x x –1 –1

x x x –1 –1 x x x –1 –1

– –x –1 –1 ➜ x 1 1

– –x –1 –1 x 1 1

– –x –1 –1 x 1 1

2(3x – 2) – 3(–x – 1) Remove brackets



zero pairs
x x x x x x x x x

1 1 1 1 1 1 ➜

–1 –1 –1 –1
Collect the like terms

➜ x x x x x x x x x

1 1
9x + 2

\ 2(3x – 2) – 3(–x – 2) = 9x + 2

Chapter 5  Algebraic Manipulation


134
(c) –3(x – 2y) – 2(–2x + y)

– x –y –y –x y y –x –x –x

– x –y –y –x y y x x x x

– x –y –y –x y y y y y y y y
➜ ➜

– –x –x y x x –y –y –y

– –x –x y x x –y

–3(x – 2y) – 2(–2x + y) Remove brackets Group like terms

x

y y y y

x + 4y

\  –3(x – 2y) – 2(–2x + y) = x + 4y

4. Two algebraic expressions are said to be equivalent if they can be simplified to the same expression.
For example, since –3(–4x + 2) = 12x – 6 and 3(4x – 2) = 12x – 6,
\  –3(–4x + 2) and 3(4x – 2) are equivalent expressions.

Work in groups to evaluate and select pairs of equivalent expressions from the following using the AlgeDiscTM application
(a) 4(–5x) (b) –3(2x – 1)
(c) 6(x – 1) (d) 2(5x + 1) – 5(–2x + 1)
(e) –4(–5x) + 9 (f) 3(2x + 1)
(g) –6(–x + 1) (h) 3(–2x + 1)
(i) –4(5x) ( j) 3(4x – 5) – 4(–2x – 6)
(k) 2(3x + 5) – 7 (l) –10(–2x + 1) + 7

(a) and (i) are equal to –20x.


(b) and (h) are equal to –6x + 3.
(c) and (g) are equal to 6x – 6.
(d) and (l) are equal to 20x – 3.
(f) and (k) are equal to 6x + 3.
(e) and (j) are equal to 20x + 9.

5. Expand a(bx), where a and b are numbers. You may substitute a and b with some numbers to help you explain.

a(bx) = abx

6. Expand a(x + y), where a is a number. You may substitute a with some numbers to help you explain.

a(x + y) = ax + ay

135
Try It! 4. The total value of 4 stacks of coins are
$(7a + 4b + 10a + 5b).
Section 5.1
(a) Simplify the expression 7a + 4b + 10a + 5b.
1. Given the expression –9mn + n – 6, state (b) Find the total value if a = 1 and b = 2.
(a) the number of terms in the expression,
(b) the coefficient of n, Solution
(c) the coefficient of mn, (a) 7a + 4b + 10a + 5b
(d) the constant term. = 7a + 10a + 4b + 5b
= 17a + 9b
Solution
(a) The terms are –9mn, n and –6. (b) When a = 1 and b = 2,
Number of terms = 3 17a + 9b
= 17 × 1 + 9 × 2
(b) Coefficient of n = 1 = 35
(c) Coefficient of mn = –9 The total value is $35.
(d) Constant term = –6
Section 5.2
2. Simplify the following. 5. Simplify (4a + b) + (3a – 6b).
(a) 5t + 6t – 7t
5
(b) – 4z – 3z + z Solution
2
(4a + b) + (3a – 6b) = 4a + b + 3a – 6b
Solution = 4a + 3a + b – 6b
(a) 5t + 6t – 7t = (5 + 6)t – 7t = 7a – 5b
= 11t – 7t
= (11 – 7)t 6. Find the sum of 5p – 4q + 7 and –3p – q + 2.
= 4t
5 Solution
(b) – 4z – 3z + 5z = (– 4 – 3)z + z
2 (5p – 4q + 7) + (–3p – q + 2)
5 = 5p – 4q + 7 – 3p – q + 2
= –7z + z = 5p – 3p – 4q – q + 7 + 2
2
5
= 2p – 5q + 9
= –7 + z
2
9 7. Simplify (7y – 2) – (4y – 9).
=– z
2
Solution
(7y – 2) – (4y – 9) = 7y – 2 – 4y + 9
3. Simplify the following. = 7y – 4y – 2 + 9
(a) 5c – 4d – 3c – d = 3y + 7
3 1
(b) 2t – 7x + + t – 2x –
2 5
8. A lift goes up (7x + 3) m, then goes down (4x – 6) m,
Solution and finally goes up (2x – 1) m. How high is the lift from
(a) 5c – 4d – 3c – d = 5c – 3c – 4d – d its starting point?
= (5 – 3)c + (– 4 – 1)d
= 2c – 5d Solution
Let the upward movement be positive.
3 1 The required height
(b) 2t – 7x + + t – 2x –
2 5 = (7x + 3) – (4x – 6) + (2x – 1)
3 1 = 7x + 3 – 4x + 6 + 2x – 1
= 2t + t – 7x – 2x + –
2 5 = 7x – 4x + 2x + 3 + 6 + 1
= (2 + 1)t – (7 + 2)x +
15

2 = (5x + 8) m
10 10
13
= 3t – 9x +
10

Chapter 5  Algebraic Manipulation


136
Section 5.3 Solution
9. Expand the following. 2x + 1 x+4 3(2x + 1) + 2(x + 4)
(a) + =
2 3 6
(a) 5(2x + 7y)
6x + 3 + 2x + 8
(b) –3(–4a + 8b) + 10(2a + b) =
6
8x + 11
Solution =
6
(a) 5(2x + 7y) = 5(2x) + 5(7y)
= 10x + 35y x–1 6 – 2x 3x + 1
(b) – +
(b) –3(– 4a + 8b) + 10(2a + b) 7 5 2
= (–3)(– 4a) – 3(8b) + 10(2a) + 10b 10(x – 1) – 14(6 – 2x) + 35(3x + 1)
=
= 12a – 24b + 10b 70
= 12a – 14b 10x – 10 – 84 + 28x + 105x + 35
=
70
143x – 59
10. Expand the following. =
70
(a) c(4x + 6y + 9z)
(b) (3m – 6n + p)(– 4d)
Section 5.4
Solution 14. Factorise 21a + 18b.
(a) c(4x + 6y + 9z) = c(4x) + c(6y) + c(9z)
= 4cx + 6cy + 9cz Solution
(b) (3m – 6n + p)(– 4d) 21a + 18b = 3(7a) + 3(6b)
= 3m(– 4d) – 6n(– 4d) + p(– 4d) = 3(7a + 6b)
= –12dm + 24dn – 4dp
15. Factorise 14by – 35bz.
11. Simplify 7y – 3[4 – 5(1 – y)].
Solution
Solution 14by – 35bz = 7b(2y) – 7b(5z)
7y – 3[4 – 5(1 – y)] = 7y – 3[4 – 5 + 5y] = 7b(2y – 5z)
= 7y – 3(–1 + 5y)
= 7y + 3 – 15y 16. Factorise 28ct – 20cz – 4c.
= 7y – 15y + 3
= 3 – 8y Solution
28ct – 20cz – 4c
12. Express each of the following as two separate terms. = (4c)(7t) – (4c)(5z) – 4c
3x – 2 –5(–2x + 7) = 4c(7t – 5z – 1)
(a) (b)
4 6

Solution
3x –2x 1
(a) = × (3x – 2)
4 4
1 1
= × 3x + × (–2)
4 4

–5(–2x + 7) –5
(b) 6
= 6 × (–2x + 7)
–5 5
= 6 × (–2x) – ×7
6
5 35
= 3
x–
6

13. Simplify the following.
2x + 1 x+4
(a) +
2 3
x–1 6 – 2x 3x + 1
(b) – +
7 5 2

137
Exercise 5.1 (e)
2
p
1
– q+ p– q
1 1
3 4 6 2
Level 1
5 1
1. State the number of terms and the constant term of each (f) 7t + 4av – t + av
3 2
of the following expressions.
(a) 2a – 3b – 1 (b) 7x + 6 – 4y + 5z Solution
(a) 3n + 10 – 4n – 11 = 3n – 4n + 10 – 11
Solution = –n – 1
(a) The terms are 2a, – 3b and –1. (d) –6 + 3k – 4k + 7 = –6 + 7 + 3k – 4k
Number of terms = 3 =1–k
Constant term = –1 (c) 3m + 4n – 2n + 5m
(b) The terms are 7x, 6, –4y and 5z. = 3m + 5m + 4n – 2n
Number of terms = 4 = 8m + 2n
Constant term = 6 (d) –7x – 3y – 2x + 3y
= –7x – 2x – 3y + 3y
2. Write down the coefficients of x and y in each of the = –9x
following. 2 1 1 1
(a) 3x – 4y + 6 (b) x2 – x + y + 8 (e) p – q+ p– q
3 4 6 2
2 1 1 1
Solution = p+ p– q– q
3 6 4 2
(a) Coefficient of x = 3 5 3
coefficient of y = –4 = p– q
6 4
(b) coefficient of x = –1 5 1
coefficient of y = 1 (f) 7t + 4av – t + av
3 2
5 1
= 7t – t + 4av + av
3. Simplify the following. 3 2
16 9
(a) 7a + 2a (b) 5b – 8b = t + av
3 2
(c) – 4x + 6x (d) –2y – 3y
(e) c + c + c (f) d + 2d – 9d
(g) –3p + p + 4p (h) –2q – 5q – q 5. (a) Simplify the expression – 4 – 2x + 5 + x.
(i) 4y – 9y + 5y ( j) – 4m – 2m + 5m – m (b) Find the value of the expression when x = 2.

Solution Solution
(a) 7a + 2a = (7 + 2)a = 9a (a) – 4 – 2x + 5 + x = – 4 + 5 – 2x + x
(b) 5b – 8b = (5 – 8)b = –3b =1–x
(c) – 4x + 6x = (–4 + 6)x (b) When x = 2,
= 2x 1 – x
(d) –2y – 3y = (–2 – 3)y = 1 – 2
= –5y = –1
(e) c + c + c = (1 + 1 + 1)c
= 3c 6. (a) Simplify the expression 7a – 2b + 5b – a – 3.
(f) d + 2d – 9d = (1 + 2 – 9)d (b) Find the value of the expression when a = –1 and
= –6d b = 2.
(g) –3p + p + 4p = (–3 + 1 + 4)p = 2p
(h) –2q – 5q – q = (–2 – 5 – 1)q = –8q Solution
(i) 4y – 9y + 5y = (4 –9 + 5)y = 0 (a) 7a – 2b + 5b – a – 3 = 7a – a – 2b + 5b – 3
(j ) – 4m – 2m + 5m – m = –6m + 5m – m = 6a + 3b – 3
= –m – m (b) When a = –1 and b = 2,
= –2m 6a + 3b – 3 = 6(–1) + 3(2) – 3
= – 6 + 6 – 3
= –3
Level 2
4. Simplify the following. 7. (a) Simplify the expression
(a) 3n + 10 – 4n – 11 3 2 1 1
x – –y+ – x.
(b) –6 + 3k – 4k + 7 4 5 3 8
(c) 3m + 4n – 2n + 5m (b) Find the value of the expression when x = –2,
(d) –7x – 3y – 2x + 3y and y = –6.

Chapter 5  Algebraic Manipulation


138
Solution (b) When x = 2 and y = 3,
(a)
3
x –
2
–y+
1

1
x height of the stack = 6(2) + 4(3)
4 5 3 8 = 24 cm
3 1 2 1
= x– x– + –y
4 8 5 3


5
= x–
1
–y 11. The lengths of the sides of a triangle are 2x cm, 4y cm
8 15 and 3x cm. Express the perimeter of the triangle in terms
(b) When x = –2 and y = –6, of x and y.
5 1 5 1
x – – y = (–2) – (–2 ) – (–6) 2x 3x
8 15 8 15
5 2
=– + +6
4 5 4y
–25 + 8 + 120
=
20 Solution

103 Perimeter of the triangle
= = (2x + 4y + 3x)
20
3 = (5x + 4y) cm
=5
20
12. A rectangle is 3p cm long and 2p cm wide.
8. The total number of atoms in the compound (a) Express the perimeter of the rectangle in terms of
CnH2n + 2 is n + 2n + 2. p.
(a) Simplify n + 2n + 2. (b) When p = 12, find the perimeter of the rectangle.
(b) Find the total number of atoms if n = 10. 3p

Solution
(a) n + 2n + 2 = 3n + 2 2p
(b) When n = 10,
total number of atoms = 3(10) + 2
= 32 Solution
(a) Perimeter of the rectangle = 2(3p + 2p)
9. The total price of a section A ticket, a section B ticket = 10p cm
and a section C ticket of a concert is given by (b) When p = 12,
$(p + 50 + p + 2p – 30). perimeter of the rectangle = 10(12)
(a) Simplify p + 50 + p + 2p – 30. = 120 cm
(b) Find the total price if p = 90.
13. A lady works 2t hours each day from Monday to Friday.
Solution She works (2t – y) hours on Saturdays. She does not
(a) p + 50 + p + 2p – 30 work on Sundays.
= p + p + 2p + 50 – 30 (a) Express her working hours in a week in terms of
= 4p + 20 t and y.
(b) When p = 90, (b) How many hours does she work each week when
total price = $(4 × 90 + 20) 1
t = 4 and y = 3?
2
= $380
Solution
(a) Working hours in a week
Level 3 = [2t × 5 + (2t – y)]
10. The height of a stack of 10 books is = (10t + 2t – y)
(2x + 3y + 4x + y) cm. = (12t – y) hours
(a) Simplify the expression 2x + 3y + 4x + y. 1
(b) Find the height of the stack if x = 2 and y = 3. (b) When t = 4 and y = 3,
2
1
working hours in a week = 12 4 –3
Solution 2
(a) 2x + 3y + 4x + y = 51 hours
= 2x + 4x + 3y + y 1
She works 51 hours each week when t = 4 and
= 6x + 4y 2
y = 3.

139
14. Write an algebraic expression that has 3 terms involving Solution
the variables p and q. (a) (6p + 7) – (3p + q)
= 6p + 7 – 3p – q
Solution = 3p – q + 7
Some possible expressions are:
(b) –4 – (6x – 3)
2p + q – 6,
= –4 – 6x + 3
p2 + 2pq – 3q2.
= –6x – 1
(c) (–2s – t) – (3s + t)
15. Create an application problem whose answer can be
= –2s – t – 3s – t
simplified to 7x.
= –5s – 2t
Solution (d) (–4x + 5y) – (3x – 6y)
Find the number of days in x weeks. = –4x + 5y – 3x + 6y
There are 7 days in a week. = –7x + 11y
There are 7x days in x weeks.

Level 2
Exercise 5.2 4. Simplify the following.
Level 1 (a) (2h – 3k + 6) + (8h – 5k – 2)
1. Simplify the following. 1 3
(b) –m – 8n + + –7m + 6n +
(a) – (2x + 1) (b) –(–3x + 6) 2 2
(c) –(4y – 9) (d) –(–5x + 8y – 7) (c) (7x + 2y) + (4x – 6) – (–3 + 2y)
1 3 1 1
Solution (d) – t– s – s – t
4 2 3 2
(a) –(2x + 1) = –2x – 1
(b) –(–3x + 6) = 3x – 6 Solution
(a) (2h – 3k + 6) + (8h – 5k – 2)
(c) –(4y – 9) = –4y + 9 = 2h – 3k + 6 + 8h – 5k – 2
(d) –(–5x + 8y – 7) = 5x – 8y + 7 = 10h – 8k + 4
1 3
(b) –m – 8n + + –7m + 6n +
2. Simplify the following. 2 2
(a) (2a + 3) + (a – 4) 1 3
= –m – 8n + – 7m + 6n +
(b) (–2b – 5) + (3b – 1) 2 2
(c) (–4c + 2d) + (–3c + d) = –8m – 2n + 2
(d) (8m – 7n) + (–5m – 2n)
(c) (7x + 2y) + (4x – 6) – (–3 + 2y)
= 7x + 2y + 4x – 6 + 3 – 2y
Solution
= 11x – 3
(a) (2a + 3) + (a – 4)
= 2a + 3 + a – 4 1 3 1 1
(d) – t– s – s – t
= 3a – 1 4 2 3 2

(b) (–2b – 5) + (3b – 1) 1 3 1 1


=– t– s– s+ t
4 2 3 2
= –2b – 5 + 3b – 1
= b – 6
1
=– t+ t– s– s
1 3 1
4 2 2 3
(c) (–4c + 2d) + (–3c + d)
1 11
= –4c + 2d – 3c + d =– t– s
4 6
= –7c + 3d
(d) (8m – 7n) + (–5m – 2n)
5. Add 7x – 2y – 4z to –2x + 3y – 5z.
= 8m – 7n – 5m – 2n
= 3m – 9n
Solution
(–2x + 3y – 5z) + (7x – 2y – 4z)
3. Simplify the following.
= –2x + 3y – 5z + 7x – 2y – 4z
(a) (6p + 7) – (3p + q)
= 5x + y – 9z
(b) –4 – (6x – 3)
(c) (–2s – t) – (3s + t)
(d) (– 4x + 5y) – (3x – 6y)

Chapter 5  Algebraic Manipulation


140
6. Find the sum of 5a – 3b, 7b – 3c and 9c – a. Solution
(a) Length of the third side
Solution = (7x – 3y + 6) – (2x + y – 1) – (x – 2y + 10)
(5a – 3b) + (7b – 3c) + (9c – a) = 7x – 3y + 6 – 2x – y + 1 – x + 2y – 10
= 5a – 3b + 7b – 3c + 9c – a = 7x – 2x – x – 3y – y + 2y + 6 + 1 – 10
= 4a + 4b + 6c = (4x – 2y – 3) cm

(b) When x = 5 and y = –1,
7. Subtract a – 4b – 3c from a + 2b – 6c. length of the third side
= 4 × 5 – 2(–1) – 3
Solution = 19 cm
(a + 2b – 6c) – (a – 4b – 3c)
= a + 2b – 6c – a + 4b + 3c
12. Peter’s mother is 4 times as old as Peter. Peter’s sister
= 6b – 3c
is 3 years younger than Peter. Let n years old be the age
of Peter. Express, in terms of n,
8. Subtract t – 3v from the sum of 7t – 2u – 3v and (a) the age of Peter’s mother,
3t + 5u – 8v. (b) the age of Peter’s sister,
(c) the sum of the ages of Peter, his mother and his
Solution sister.
(7t – 2u – 3v + 3t + 5u – 8v) – (t – 3v)
= 7t – 2u – 3v + 3t + 5u – 8v – t + 3v Solution
= 9t + 3u – 8v (a) Age of Peter’s mother
= 4n years
(b) Age of Peter’s sister
Level 3
= (n – 3) years
9. There are 3 consecutive integers. If the smallest one is
n, find the sum of the 3 integers. (c) Sum of the ages
= n + 4n + (n – 3)
Solution = n + 4n + n – 3
The 3 integers are n, n + 1 and n + 2. = (6n – 3) years
Their sum = n + (n + 1) + (n + 2)
= n + n + 1 + n + 2 13. In an experiment, the original temperature of a
= 3n + 3 beaker of water is (t + 20) °C. It is heated up by
(3t + 15) °C and then cools down by (2t – 10) °C.
10. The masses of 3 boxes of chocolates are (a) Find the final temperature in terms of t.
(3p + 4q + 2) g, (4p + 6q + 5) g and (p + 7q + 9) g. (b) If t = 11, find the final temperature.
(a) Find their total mass in terms of p and q. (c) Is it possible that t = 25? Explain briefly.
(b) If p = 10 and q = 20, find the total mass.
Solution
Solution (a) Final temperature
(a) Total mass = (t + 20) + (3t + 15) – (2t – 10)
= (3p + 4q + 2) + (4p + 6q + 5) + (p + 7q + 9) = t + 20 + 3t + 15 – 2t + 10
=3p + 4q + 2 + 4p + 6q + 5 + p + 7q + 9 = (2t + 45) °C
= (8p + 17q + 16) g (b) When t = 11,
final temperature
(b) When p = 10 and q = 20, total mass = 2 × 11 + 45
= 8(10) + 17(20) + 16 = 67 °C
= 436 g (c) When t = 25,
initial temperature = t + 20
11. The perimeter of a triangle is (7x – 3y + 6) cm. If the = 25 + 20
lengths of two sides of the triangle are (2x + y – 1) cm = 45 °C
and (x – 2y + 10) cm. rise in temperature = 3t + 15
(a) Find the length of the third side in terms of x and y. = 3(25) + 15
(b) If x = 5 and y = –1, find the length of the third side. = 90 °C
45 °C + 90 °C = 135 °C,
which exceeds the boiling point, 100 °C, of pure
water.
Hence, it is NOT possible that t = 25.

141
14. The perimeter of a rectangle is (6x + 5y) cm. Suggest (d) 9m = 9n + 72
two possible dimensions of the rectangle. or m = n + 8
(e) Some other interesting properties include:
Solution (i) The sum of the 9 numbers is a multiple of 9.
Perimeter = (6x + 5y) cm (ii) The sum of the 3 numbers on each diagonal
Sum of the length and the breadth is equal.
1
= (6x + 5y)
2

= 3x + y cm
5 Exercise 5.3
2
Level 1
Two possible dimensions of the rectangle are 1. Expand the following.
5 5 (a) 3(2 + a) (b) 4(7b + 5c)
3x cm by y cm and 2x cm by 3x + y cm.
2 2 (c) (2d – 6e)(5) (d) (–3g – 4h)(2)
(e) – 4(7 – 5n) (f ) –5(–3p + 9q)
15. (g) 6(2r – 3s + 4t)
Sun Mon Tue Wed Thu Fri Sat
(h) a(–5x + 3y – 8z)
1 2 3 4 5 2
6 7 8 9 10 11 12 (i) (6a – 18b – 24c)
3
13 14 15 16 17 18 19 5
( j) (4a – 8b + 12c) –
2
20 21 22 23 24 25 26
27 28 29 30 31 Solution
(a) 3(2 + a) = 3(2) + 3(a)
Suppose 9 dates in a certain month are enclosed by a = 6 + 3a
rectangle as shown. (b) 4(7b + 5c) = 4(7b) + 4(5c)
(a) Explain a quick way to calculate the sum of the 9 = 28b + 20c
numbers. (c) (2d – 6e)(5) = 10d – 30e
(b) Let n be the number at the top left hand corner of (d) (–3g – 4h)(2) = –6g – 8h
the rectangle. Express the sum of the 9 numbers (e) (– 4)(7 – 5n) = –28 + 20n = 20n – 28
in terms of n. (f ) (–5)(–3p + 9q) = 15p – 45q
(c) Let m be the middle number in the rectangle. (g) 6(2r – 3s + 4t) = 12r – 18s + 24t
Express the sum of the 9 numbers in terms of m. (h) a(–5x + 3y – 8z) = –5ax + 3ay – 8az
(d) Write down an equation relating m and n. 2
(e) Describe some other interesting properties about (i) (6a – 18b – 24c) = 4a – 12b – 16c
3
the numbers within the rectangle. 5
( j) (4a – 8b + 12c) – = –10a + 20b – 30c
2
Solution
(a) Sum of the 9 numbers
= (7 + 8 + 9) + (14 + 15 + 16) + (21 + 22 + 23) 2. Simplify the following.
= 3 × 8 + 3 × 15 + 3 × 22 (a) 4(x + 7) + 3(x + 5)
= 24 + 45 + 66 (b) 3(2a + 4b) + 9(3a – 2b)
= 135 (c) –4(2x + 1) + 3(–x – 2)
(d) 2(3u – 5) – 3(2u + 1)
OR
(e) –5(3x – y) – 7(–2x + 3y)
Sum of the 9 numbers (f) b(–5v – 4) – 2b(v + 4)
= 9 × 15 (g) 3(2a – 3b – 5c) – 5(a – 3c)
= 135 (h ) (x – 3y)(–2) + (x – y + 3)(6)
15 is the mean of the 9 numbers.
Solution
(b) Sum of the 9 numbers
(a) 4(x + 7) + 3(x + 5)
= n + (n + 1) + (n + 2) + (n + 7) + (n + 8)
= 4x + 28 + 3x + 15
  + (n + 9) + (n + 14) + (n + 15) + (n + 16)
= 7x + 43
= 9n + 72
(b) 3(2a + 4b) + 9(3a – 2b)
(c) Sum of the 9 numbers = 6a + 12b + 27a – 18b
= (m – 8) + (m – 7) + (m – 6) + (m – 1) + m = 33a – 6b
  + (m + 1) + (m + 6) + (m + 7) + (m + 8)
(c) –4(2x + 1) + 3(–x – 2)
= 9m
= –8x – 4 – 3x – 6
= –11x – 10

Chapter 5  Algebraic Manipulation


142
(d) 2(3u – 5) – 3(2u + 1) (d) 5[a – (b – a)] + 7(–a + 2b)
= 6u – 10 – 6u – 3 = 5(a – b + a) – 7a + 14b
= –13 = 10a – 5b – 7a + 14b
(e) –5(3x – y) – 7(–2x + 3y) = 3a + 9b
= –15x + 5y + 14x – 21y (e) 9(y – z) – [5y – z – 3(2y – 4z)]
= –x – 16y = 9y – 9z – (5y – z – 6y + 12z)
(f) b(–5v – 4) – 2b(v + 4) = 9y – 9z + y – 11z
= –5bv – 4b – 2bv – 8b = 10y – 20z
= –7bv – 12b
(f ) 4[2p + 3q – (p + q)]
(g) 3(2a – 3b – 5c) – 5(a – 3c) = 4(2p + 3q – p – q)
= 6a – 9b – 15c – 5a + 15c = 4(p + 2q)
= a – 9b = 4p + 8q
(h) (x – 3y)(–2) + (x – y + 3)(6) x x–8
= –2x + 6y + 6x – 6y + 18 (g) 2
+
3
= 4x + 18 3x + 2(x – 8)
=
6
3x + 2x – 16
Level 2 =
6
3. Simplify the following. 5x – 16
=
(a) 4a – [5a – (3 + 2a)] 6
(b) 7t – [5s + 8(s + 2t)] 2x + 1 x–3
(c) 4m + n + [5m – 6(m – n)] (h) –
4 5
(d) 5[a – (b – a)] + 7(–a + 2b)
(e) 9(y – z) – [5y – z – 3(2y – 4z)] 5(2x + 1) – 4(x – 3)
=
(f ) 4[2p + 3q – (p + q)] 20

x x–8 10x + 5 – 4x + 12
(g) 2 + =
3 20
6x + 17
2x + 1 x–3
(h) – =
4 5 20

3t t+8
(i) – + 3t t+8
7 2 (i) – +
7 2
5t – 2 2(t + 1)
( j) – –2(3t) + 7(t + 8)
5 3 =
14
5(x – 1) 3(2x + 1)
(k) – – –6t + 7t + 56
6 4
=
x+2 4(3x – 1) 14
(l) 1 – +
2 9 t + 56
=
y+1 y+2 5y – 1 14
(m) + –
3 2 6
( j)
5t – 2

2(t + 1)
y 2(3y – 1) 4y + 7
(n) – 5 + – 5 3
4 3
3(5t – 2) – 10(t + 1)
=
Solution 15
(a) 4a – [5a – (3 + 2a)] 15t – 6 – 10t – 10
=
= 4a – (5a – 3 – 2a) 15
= 4a – (3a – 3) 5t – 16
= 4a – 3a + 3 =
15
= a + 3
5(x – 1) 3(2x + 1)
(b) 7t – [5s + 8(s + 2t)] (k) – –
6 4
= 7t – (5s + 8s + 16t) –10(x – 1) – 9(2x + 1)
= 7t – 13s – 16t =
12
= –13s – 9t
–10x + 10 – 18x – 9
=
(c) 4m + n + [5m – 6(m – n)] 12
= 4m + n + (5m – 6m + 6n) –28x + 1
= 4m + n – m + 6n =
12
= 3m + 7n

143
(l) 1 –
x+2
+
4(3x – 1) 5. There are (2a + b) books in a pile. The thickness of each
2 9 book is 2 cm. Find the height of the pile of books in
18 – 9(x + 2) + 8(3x – 1) terms of a and b, expressing the answer in the simplest
= form.
18
18 – 9x – 18 + 24x – 8
= Solution
18
Height of the pile of books = 2(2a + b)
15x – 8 = (4a + 2b) cm
=
18

(m)
y+1
+
y+2

5y – 1 6. A grocer bought n eggs at $x each. He marked up the
3 2 6 price of each egg by $y and sold all of them. Find the
2(y + 1) + 3(y + 2) – (5y – 1) revenue in terms of n, x and y, expressing the answer in
= the simplest form.
6
2y + 2 + 3y + 6 – 5y + 1
= Solution
6
Sales amount = $(x + y)n
9 = $(nx + ny)
=
6
3
= 7. If the oranges in a box are arranged in 6 rows and each
2
row consists of (2m – 3) oranges, there are 5 oranges
(n) – +
y 2(3y – 1)

4y + 7 left.
5 4 3 (a) Express, in expanded form, the number of oranges
–12y + 30(3y – 1) – 20(4y + 7) in the box in terms of m.
=
60 (b) Find the number of oranges in the box when
–12y + 90y – 30 – 80y – 140 m = 7.
=
60
Solution
–2y – 170 (a) Number of oranges = 6(2m – 3) + 5
=
60
= 12m – 18 + 5
= 12m – 13
(b) When m = 7,
Level 3
12m –13 = 12(7) – 13
4. Let n be a number. Find the resulting expression after = 84 – 13
going through each step in order. = 71
(a) Add 10 to the number. The number of oranges in the box is 71 when
(b) Multiply the sum by 2. m = 7.
(c) Subtract 4 from the result.
(d) Divide the result in (c) by 2.
(e) Subtract the number n from the result in (d). 8. In making the frame of a rectangular box, a carpenter
(f ) Simplify the expression in (e). needs 4 pieces of wood of length (2x + 3y) cm long and
Note: This is the mathematics behind the number trick 8 pieces of length (2x + y) cm long.
at the beginning of the chapter. (a) Express the total length of wood required in terms
of x and y.
Solution (b) If x = 30 and y = 10, find the total length of wood
(a) n + 10 required.
(b) 2(n + 10) Solution
(c) 2(n + 10) – 4 (a) 4(2x + 3y) + 8(2x + y) = 24x + 20y
(d) [2(n + 10) – 4] ÷ 2 (b) When x = 30 and y = 10,
24x + 20y = 24(30) + 20(10)
(e) [2(n + 10) – 4] ÷ 2 – n = 920
(f ) Expression
= [2(n + 10) – 4] ÷ 2 – n The total length of wood required is 920 cm.
= (2n + 20 – 4) ÷ 2 – n
= (2n + 16) ÷ 2 – n 9. A condominium has 28 floor levels. In each of the lower
=n+8–n 20 floor levels, there are x 3-bedroom and y 2-bedroom
=8 apartments. In each of the upper 8 floor levels, there are
(x – 1) 4-bedroom apartments.

Chapter 5  Algebraic Manipulation


144
(a) Express the total number of bedrooms in the (a) Complete the following table.
condominium in terms of x and y.
(b) If x = 5 and y = 3, find the total number of bedrooms n Total number of matchsticks
in the condominium. 1
2
Solution
(a) 20(3x + y) + 8[4(x – 1)] = 60x + 40y + 32x – 32 3
= 92x + 40y – 32 4
(b) When x = 5 and y = 3, 5
total number of bedrooms = 92(5) + 40(3) – 32
= 548 (b) Find the total number of matchsticks used to form
n squares in terms of n.
(c) If each matchstick is 3 cm long, find the perimeter
10. Design a number trick such that the final result is always 7. of the figure formed by n squares in terms of n.

Solution
Solution
The steps for the number trick can be: (a)
(a) Think of a number. n Total number of matchsticks
1 4
(b) Add 9 to the number.
2 7
(c) Multiply the sum by 3.
3 10
(d) Subtract 6 from the result in (c). 4 13
(e) Divide the difference by 3. 5 16
(f ) Subtract your original number from the result in
(e). (b) Total number of matchsticks used to form
n squares = 3n + 1
(g) You will get the number 7. (c) The perimeter of the figure is formed by 2n + 2
If the original number is n, the resulting expression matchsticks.
= [(n + 9) × 3 – 6] ÷ 3 – n Perimeter = 3(2n + 2)
= (3n + 27 – 6) ÷ 3 – n = (6n + 6) cm
= (3n + 21) ÷ 3 – n
=n+7–n
=7 Exercise 5.4
Level 1
11. Some short-cuts in Arithmetic make use of the distributive 1. Factorise the following.
law. For example, (a) 6a + 3b (b) 10c – 12d
37 × 99 = 37 × (100 – 1) (c) 28e + 21 (d) 6f – 6
= 3700 – 37 (e) 18ax – 15ay (f ) –5mx – 10my
= 3663.
(g) 15ab – 3a (h) 12s + 48bs
Devise another two such short-cuts. (i) 51cu + 17cv ( j) –56ay – 8a

Solution Solution
The following shows two short-cuts in Arithmetic that (a) 6a + 3b = 3(2a) + 3b
make use of the distributive law. = 3(2a + b)
(i) 56 × 101 = 56 × (100 + 1) (b) 10c – 12d = 2(5c) – 2(6d)
= 5600 + 56 = 2(5c – 6d)
= 5656 (c) 28e + 21 = 7(4e) + 7(3)
(ii) 387 × 999 = 387 × (1000 – 1) = 7(4e + 3)
= 387 000 – 387 (d) 6f – 6 = 6(f – 1)
= 386 613 (e) 18ax – 15ay = 3a(6x) – 3a(5y)
= 3a(6x – 5y)
(f ) –5mx – 10my = –5m(x) + (–5m)(2y)
12. Some matchsticks are used to form a pattern of n squares = –5m(x + 2y)
as shown below. (g) 15ab – 3a = 3a(5b – 1)
(h) 12s + 48bs = 12s(1 + 4b)
(i) 51cu + 17cv = 17c(3u + v)
( j) –56ay – 8a = –8a(7y + 1)

145
Level 2 4. Compute the following without using calculators.
2. Factorise the following. (a) 389 × 57 + 389 × 43
(a) 7a + 7b + 7c (b) 86 × 471 – 76 × 471
(b) 12a – 8b + 20c
(c) 5ax – 15bx – 30x Solution
(d) –18 – 24ay – 6y (a) 389 × 57 + 389 × 43 = 389 × (57 + 43)
(e) mx + my + 3mz = 389 × 100
(f ) 7bt – 21b – 35bt = 38 900
(g) 3a(x + y) – 4b(x + y) (b) 86 × 471 – 76 × 471 = (86 – 76) × 471
(h) a(5m – 4) + b(5m – 4) = 10 × 471
(i) 8c(x – 2y) + 20c(x – 2y) = 4710
(j) 10(a + 2b)x – 25(a + 2b)y

Solution 5. There are 16ax + 20ay pieces of square tiles of unit


(a) 7a + 7b + 7c = 7(a + b + c) length, where a, x and y are positive integers. State two
possible dimensions of rectangles that can be formed by
(b) 12a – 8b + 20c = 4(3a – 2b + 5c) paving all the tiles.
(c) 5ax – 15bx – 30x = 5x(a – 3b – 6)
Solution
(d) –18 – 24ay – 6y = –6(3 + 4ay + y) 16ax + 20ay = (4a)(4x) + (4a)(5y)
(e) mx + my + 3mz = m(x + y + 3z) = 4a(4x + 5y)
The possible dimensions are
(f ) 7bt – 21b – 35bt = 7b(t – 3 – 5t) 1 × 4a(4x + 5y), 2 × 2a(4x + 5y),
= –7b(3 + 4t) 4 × a(4x + 5y), a × 4(4x + 5y),
(g) 3a(x + y) – 4b(x + y) = (3a – 4b)(x + y) 2a × 2(4x + 5y), 4a × (4x + 5y).
(h) a(5m – 4) + b(5m – 4) = (5m – 4)(a + b)
6. Write 3 algebraic expressions, in expanded form,
(i) 8c(x – 2y) + 20c(x – 2y) = (8c + 20c)(x – 2y)
involving a, x and y such that (x + y) is one of their
= 28c(x – 2y)
factors.
(j) 10(a + 2b)x – 25(a + 2b)y
= [5(a + 2b)](2x) – [5(a + 2b)](5y) Solution
= 5(a + 2b)(2x – 5y) The following are some algebraic expressions, in
expanded form, involving a, x and y such that (x + y) is
a factor:
Level 3 ax + ay, ax2 + axy, –2a2x – 2a2y
3. Simplify each of the following give the answer in
factorised form.
(a) 4(2x + 3) – 5(4x – 6) Revision Exercise 5
(b) 3(7x – 2y) – 9(–x + y) 1. Simplify the following.
(c) 2(4ax – 3y + 1) – 6y(2a – 1) + 2(2a – 1) (a) 2a + 3b – 4a + 5b
(b) (2p – 7q – 6r) + (3p – 4q – r)
Solution (c) (2x – 3y + 4) + (–3x + 6y – 1)
(a) 4(2x + 3) – 5(4x – 6) (d) (–3m – 8n + 2p) – (– 4m + 7n – 3p)
= 8x + 12 – 20x + 30
= –12x + 42 Solution
= 6(–2x) + 6(7) (a) 2a + 3b – 4a + 5b = –2a + 8b
= 6(–2x + 7)
(b) (2p – 7q – 6r) + (3p – 4q – r)
(b) 3(7x – 2y) – 9(–x + y) = 2p – 7q – 6r + 3p – 4q – r
= 21x – 6y + 9x – 9y = 5p – 11q – 7r
= 30x – 15y
(c) (2x – 3y + 4) + (–3x + 6y – 1)
= 15(2x) – 15(y)
= 2x – 3y + 4 – 3x + 6y – 1
= 15(2x – y)
= –x + 3y + 3
(c) 2(4ax – 3y + 1) – 6y(2a – 1) + 2(2a – 1)
(d) (–3m – 8n + 2p) – (– 4m + 7n – 3p)
= 8ax – 6y + 2 – 12ay + 6y + 4a – 2
= –3m – 8n + 2p + 4m – 7n + 3p
= 8ax – 12ay + 4a
= m – 15n + 5p
= (4a)(2x) – (4a)(3y) + 4a
= 4a(2x – 3y + 1)

Chapter 5  Algebraic Manipulation


146
2. Simplify the following. (d) 1 –
3(x – 2)
+
4(1 – 2x)
(a) 4(2m – 1) + 3(4m + 1) 6 5
(b) 6(–2n – 3) – 5(2n + 6) x + 2 4(1 – 2x)
= 1– +
(c) –3(4x + y) + 2(5x – 8y) 2 5
(d) –7(2x – y + 9) – 4(–3x + y – 5) 10 – 5(x + 2) + 8(1 – 2x)
=
10
10 – 5x – 10 + 8 – 16x
Solution =
10
(a) 4(2m – 1) + 3(4m + 1)
–21x + 8
= 8m – 4 + 12m + 3 =
10
= 20m – 1
(b) 6(–2n – 3) – 5(2n + 6) 4. Factorise the following.
= –12n – 18 – 10n – 30 (a) 14a – 21
= –22n – 48 (b) 6m – 8n
(c) –9bx – 15by
(c) –3(4x + y) + 2(5x – 8y)
(d) 15ax – 20ay + 10az
= –12x – 3y + 10x – 16y
= –2x – 19y
Solution
(d) –7(2x – y + 9) – 4(–3x + y – 5) (a) 14a + 21 = 7(2a) + 7(3)
= –14x + 7y – 63 + 12x – 4y + 20 = 7(2a + 3)
= –2x + 3y – 43
(b) 6m – 8n = 2(3m) – 2(4n)
= 2(3m – 4n)
3. Simplify the following.
(c) –9bx – 15by = (–3b)(3x) + (–3b)(5y)
3x 2(x – 1)
(a) + = –3b(3x + 5y)
4 5

(b)
4(2x – 1)

x–3 (d) 15ax – 20ay + 10az
7 2 = (5a)(3x) – (5a)(4y) + (5a)(2z)
3(x – 1) 5(x – 3) = 5a(3x – 4y + 2z)
(c) – +
2 3
3(x – 2) 4(1 – 2x)
(d) 1 – + 5. (a) Simplify 6(x + 2y) – 7(4x – 3y).
6 5
(b) Factorise the result in (a).
Solution (c) When x = –1 and y = 5, find the value of the
3x 2(x – 1) expression in (a).
(a) +
4 5
15x + 8(x – 1) Solution
= (a) 6(x + 2y) – 7(4x – 3y) = 6x + 12y – 28x + 21y
20
15x + 8x – 8 = –22x + 33y
=
20
(b) –22x + 33y = –11(2x – 3y)
23x – 8
= (c) When x = –1 and y = 5,
20
–22x + 33y = –22(–1) + 33(5)
4(2x – 1) x–3 = 187
(b) –
7 2
8(2x – 1) – 7(x – 3)
= 6. The three sides of a triangle are (7x + 3y) cm,
14
16x – 8 – 7x + 21 (5x + y – 4) cm and (3x – 5y – 8) cm long.
=
14
9x + 13
=
14
7x + 3y 3x – 5y – 8
3(x – 1) 5(x – 3)
(c) – +
2 3
–9(x – 1) + 10(x + 3)
=
6 5x + y – 4
–9x + 9 + 10x + 30
= (a) Express the perimeter of the triangle in terms of x
6
x + 39
and y.
= (b) If x = 3 and y = –2, find the perimeter of the triangle.
6
(c) Explain why we cannot have x = 5 and y = 1 for
this triangle.

147
Solution 9. An examination consists of 3 papers. The minimum
(a) Perimeter of the triangle total score required to pass the examination is
= (7x + 3y) + (5x + y – 4) + (3x – 5y – 8) (8x + 4y) marks. Muthu scores (2x – y + 10) marks and
= 7x + 3y + 5x + y – 4 + 3x – 5y – 8 (2x + 3y – 6) marks in the first two papers.
= 7x + 5x + 3x + 3y + y – 5y – 4 – 8 (a) Find Muthu’s total score in the first two papers.
= (15x – y –12) cm (b) How many marks does Muthu score in the third
(b) When x = 3 and y = –2, paper if he just passes the examination?
perimeter of the triangle (c) Factorise the result in (b).
= 15 × 3 – (–2) –12
= 35 cm Solution
(a) Muthu’s total score in the first two papers
(c) When x = 5 and y = 1,
= [(2x – y + 10) + (2x + 3y – 6)]
7x + 3y = 7 × 5 + 3 × 1 = 38,
= (2x – y + 10 + 2x + 3y – 6)
5x + y – 4 = 5 × 5 + 1 – 4 = 22,
= (4x + 2y + 4) marks
3x – 5y – 8 = 3 × 5 – 5 × 1 – 8 = 2.
(b) The required score
As 22 + 2 < 38, the lengths 38 cm, 22 cm and 2
= [(8x + 4y) – (4x + 2y + 4)]
cm cannot form a triangle.
= [8x + 4y – 4x – 2y – 4)
= (4x + 2y – 4) marks
7. In a chemical reaction, two compounds A and B reacted
to produce two products C and D. (c) 4x + 2y – 4 = 2(2x + y – 2)
A+B C + D.
By the law of conservation of mass, the total mass of A 10. In the figure, n identical tables are joined end-to-end
and B is equal to the total mass of C and D. The masses to form a long table. A single table can have 2 seats on
of A, B and D are (5x – y) g, (7x + 3y) g and (8x + 6y) each side and one seat at each end.
g respectively.
(a) Express the mass of C in terms of x and y.
(b) Factorise the result in (a).
(c) Find the mass of C if x = 7 and y = –3.

Solution (a) Copy and complete the following table.


(a) Mass of C
= (5x – y) + (7x + 3y) – (8x + 6y) n Total number of seats
= 5x – y + 7x + 3y – 8x – 6y 1
= 5x + 7x – 8x – y + 3y – 6y 2
= (4x – 4y) g
3
(b) Mass of C
4
= 4(x – y) g
5
(c) When x = 7 and y = –3,
mass of C
(b) If n tables are joined to form a long table, express
= 4[7 – (–3)]
the total number of seats in terms of n.
= 40 g
Solution
8. The numbers of marbles in 2 bags are 3ax and 6bx. (a)
(a) Find the total number of marbles in all the bags.
n Total number of seats
(b) Factorise the result in (a).
(c) All the marbles are arranged in rows and columns 1 6
to form a rectangle. If one side of the rectangle has 2 10
(a + 2b) marbles, find the number of marbles on 3 14
the other side.
4 18
Solution 5 22
(a) Total number of marbles
= 3ax + 6bx (b) Total number of seats for n tables
(b) 3ax + 6bx = (3x)(a) + (3x)(2b) = 6 + 4(n – 1)
= 3x(a + 2b) = 6 + 4n – 4
(c) As 3x(a + 2b) = 3x × (a + 2b), number of marbles = 4n + 2
on the other side = 3x.

Chapter 5  Algebraic Manipulation


148
6 Simple Equations in One
Variable

Class Activity 1
Objective: To use AlgeDiscTM application in AlgeToolsTM to help us understand the concept of linear equations.

Task
Solve the equation 2x + 7 = –3 using AlgeDiscTM application (Linear Equation, Activity 2) in AlgeToolsTM.

Balance Equation

1. Place the corresponding discs on the LHS and RHS of the balance. Click the ‘Form
Equation’ button. Note that the number discs 7 and –3 are introduced.

x x 7 –3 2x + 7 = –3

2. To eliminate 7 on the LHS, we add –7 on both sides.

–7

x x 7 –3 –7 2x + 7 –7 = –3 –7

3. Group the numbers on the right-hand side.

x x –10
2x = –10

4. Use the ‘Divide Menu’ button to divide both sides by 2.

x –5 2x –10
=
2 2

  x = –5

\  the solution is x = –5.


Equations 2x + 7 = –3 and 2x = –10 are equivalent equations. Both of them have the solution x = –5. More specifically, they
are equivalent to the equation x = –5.

149
Questions
1. Does x = –5 mean the same as –5 = x?

Yes x = –5 mean the same as –5 = x.

2. Work with a partner and use AlgeDiscTM application (Linear Equation, Activity 2) in AlgeToolsTM to help you form the
following equations and solve them.
(a) x – 2 = 5 (b) 3x + 4 = 10
(c) 2x – 5 = –9 (d) 2x = x + 7
(e) 3x + 2 = –2x – 6 (f) x – 3 = –5x + 8

(a) x – 2 = 5
x – 2 + 2 = 5 + 2
x = 7

(b) 3x + 4 = 10
3x + 4 – 4 = 10 – 4
3x = 6
x = 2

(c) 2x – 5 = –9
2x – 5 + 5 = –9 + 5
2x = –4
x = –2

(d) 2x = x + 7
2x – x = x + 7 – x
x = 7

(e) 3x + 2 = –2x – 6
3x + 2 – 2 = –2x – 6 – 2
3x = –2x – 8
3x + 2x = –2x – 8 + 2x
5x = –8
8
x = –
5

(f) x – 3 = –5x + 8
x – 3 + 3 = –5x + 8 + 3
x = –5x + 11
x + 5x = –5x + 11 + 5x
6x = 11
11
x =
16

Chapter 6  Simple Equations in One Variable


150
Class Activity 2
Objective: To formulate linear equations to solve problems.

Questions
1. There are 54 students in a choir. If there are 6 more girls than boys, how many girls are there?

(a) Let x be the number of girls in the choir.


As there are 6 more girls than boys, express the number of boys in terms of x.

Number of boys = x – 6

(b) As the total number of girls and boys is 54, formulate an equation in x.

x + x – 6 = 54

(c) Solve the equation and hence write down the number of girls.

x + x – 6 = 54
2x – 6 = 54
2x = 54 + 6
2x = 60
60
x =
2
x = 30

There are 30 girls.

2. In a bakery, the number of buns made is three times as many as cakes. After selling 200 buns and 40 cakes, the number
of buns left is twice as many as cakes left. Find the number of cakes made.
(a) Let x be the number of cakes made. Express the following quantities in terms of x:

(i) the number of buns made, 3x

(ii) the number of buns left after selling, 3x – 200

(iii) the number of cakes left after selling. x – 40

(b) Using the condition that the number of buns left is twice as many as cakes left, formulate an equation in x.

3x – 200 = 2(x – 40)

(c) Solve the equation and hence write down the number of cakes made.

3x – 200 = 2(x – 40)


3x – 200 = 2x – 80
3x – 2x = 200 – 80
x = 120

120 cakes were made.

151
Try It! 6. Given that c = 4x – 5, d = –x – 2 and c = d, write down
an equation in x and solve it.
Section 6.1
1. Solve the equation x – 2 = 7. Solution

Solution c=d
x – 2 = 7 4x – 5 = –x – 2
x – 2 + 2 = 7 + 2
x = 9 4x + x = 5 – 2
5x = 3
2. Solve the equation x + 5 = 11. 3
x =
5
Solution
x + 5 = 11

x + 5 – 5 = 11 – 5 Section 6.2
x = 6
7. Solve the equation 7(x + 2) = 2(2x + 9).
x
3. Solve the equation = 5. Solution
3
7(x + 2) = 2(2x + 9)
Solution 7x + 14 = 4x + 18

x
= 5 7x – 4x + 14 = 4x – 4x + 18
3 3x + 14 – 14 = 18 – 14
x
× 3 = 5 × 3 3x = 4
3 4
x = 15 x =
3

4. Solve the equation –10x = 35. 2x – 5 3x – 2


8. Solve the equation = .
7 4
Solution

–10x = 35 Solution
2x – 5 3x – 2
–10x 35 =
= 7 4
–10 –10
7
4(2x + 5) = 7(3x – 2)
x = – 8x + 20 = 21x – 14
2
1 8x – 21x = –20 – 14
x = –3 –13x = –34
2
34
x =
13
5. Solve the equation 10x + 21 = –25 – 13x. 8
x = 2
13
Solution
10x + 21 = –25 – 13x z z–6
9. Solve the equation + = 2.
10x + 21 + 13x = –25 – 13x + 13x 3 4
23x + 21 = –25 Solution
23x + 21 – 21 = –25 – 21 z–6
z
23x = –46     + = 2
3 4
46
x = – 4z + 3(z – 6) = 24
23
4z + 3z –18 = 24
x = –2 7z = 42
z = 6

Chapter 6  Simple Equations in One Variable


152
10. Given the formula A =
1
(a + b)h, find the value of a 14. Mr Rashid is 4 times as old as his son. Four years ago,
2 the sum of their ages was 37 years. Find the son’s present
when b = 13, h = 9 and A = 90. age.
Note: A and a stand for different variables.
Solution
Solution Let the son’s present age be x years.
1 (4x – 4) + (x – 4) = 37
Putting b = 13, h = 9 and A = 90 into A = (a + b)h,
2 4x – 4 + x – 4 = 37
we have the equation:
1 5x – 8 = 37
(a + 13) × 9 = 90 5x = 45
2 2
a + 13 = 90 × x = 9
9
a + 13 = 20 The son’s present age is 9 years.
a = 7
15. The price of a book is $1 more than twice the price of a
ballpoint pen. The total price of 5 books and 4 ballpoint
Section 6.3 pens is $47. Find the prices of a ballpoint pen and a book.
8
11. Solve the equation = 4.
x+1
Note: Similarly, x = –1 cannot be a solution of the given Solution
equation. Let the price of a ballpoint pen be $x.
5(2x + 1) + 4x = 47
Solution 10x + 5 + 4x = 47
8 14x = 42
= 4
x+1 x = 3
8 = 4(x + 1) 2x + 1 = 7
8 = 4x + 4 The price of a ballpoint pen is $3 and that of a book is
4x = 4 $7.
x = 1

3 2
12. Solve the equation = .
y+4 y

Solution
3 2
=
y+4 y
3y = 2(y + 4)
3y = 2y + 8
y = 8

Section 6.4
13. The sum of three consecutive integers is 144. Find the
integers.

Solution
Let x be the smallest integer.
x + (x + 1) + (x + 2) = 144
3x + 3 = 144
3x = 141
x = 47
The integers are 47, 48 and 49.

153
Exercise 6.1 ( j) 3x + 8 = –1
Level 1 3x + 8 – 8 = –1 – 8
3x = –9
1. Solve the following equations.
9
(a) x + 8 = 9 (b) x + 36 = –40 x = – 
3
(c) x – 9 = 5 (d) x – 22 = –15 x = –3
x x
(e) = 3 (f ) = –4
2 5
(k) –5x + 2 = –3
(g) 4x = 24 (h) –9x = 21 –5x = –5
(i) 2x – 3 = 7 ( j) 3x + 8 = –1 x = 1
x
(k) –5x + 2 = –3 (l) –2=0 x
3 (l) – 2 = 0
1 1 3
(m) x + 9 = 6 (n) 1 – x = –8 x
4 7 =2
3

Solution x = 6
(a) x + 8 = 9 1
x + 8 – 8 = 9 – 8 (m) x + 9 = 6
4
x = 1 1
x = –3
(b) x + 36 = –   40 4
x + 36 – 36 = – 40 – 36 x = –12
x = –76 1
(n) 1 – x = –8
(c) x – 9 = 5 7
x – 9 + 9 = 5 + 9 1
– x = –9
7
x = 14
x = (–9)(–7)
(d) x – 22 = –15
x – 22 + 22 = –15 + 22 x = 63
x = 7
x
(e) = 3 Level 2
2
x 2. Solve the following equations.
2 × = 2 × 3 (a) 3x = x + 6 (b) 2x – 5 = x + 1
2
(c) 9 – 5x = x – 1 (d) –3x + 4 = –5x + 8
x = 6
(e) 3x + 6 = – 4x – 8 (f) –1 – 7x = –3 + 2x
x (g) 2x + 7x – 3 = 5x + 4
(f ) = – 4
5 (h) 19 – 3x – 6x = 7 – 4x
x
5 × = 5 × –   4
5 Solution
x = –20 (a) 3x = x + 6
3x – x = 6
(g) 4x = 24 2x = 6
4x 24 x = 3
= (b) 2x – 5 = x + 1
4 4
2x – x = 1 + 5
x = 6
x = 6
(h)
–9x = 21 (c) 9 – 5x = x – 1
–9x 21 –5x – x = –1 – 9
= –6x = –10
–9 –9
7 –10
x = – x =
3 –6
5
x = –2
1 x =
3
3
2
x = 1
(i) 2x – 3 = 7 3
2x – 3 + 3 = 7 + 3
(d) –3x + 4 = –5x + 8
2x = 10
–3x + 5x = 8 – 4
2x 10
= 2x = 4
2 2
x = 2
x = 5
Chapter 6  Simple Equations in One Variable
154
(e) 3x + 6 = –4x – 8 5. The distance, d km, of a train from a station after
3x + 4x = –8 – 6 t hours is given by the formula d = 240 – 80t. Find the
7x = –14 time t when
x = –2 (a) d = 40,
(b) the train reaches the station.
(f) –1 – 7x = –3 + 2x
–7x – 2x = –3 + 1
Solution
–9x = –2
(a) d = 240 – 80
2
x = When d = 40,
9
40 = 240 – 80t
(g) 2x + 7x – 3 = 5x + 4 80t = 240 – 40
9x – 3 = 5x + 4 80t = 200
9x – 5x = 4+3 200
t =
4x = 7 80
7 1
x = t = 2
4 2
3 (b) When the train reaches the station, d = 0.
x = 1
4 0 = 240 – 80t
(h) 19 – 3x – 6x = 7 – 4x 80t = 240
19 – 9x = 7 – 4x 240
t =
–5x = –12 80
12 t = 3
x =
5
2 6. In the diagram, a spring is hung from a ceiling
x =
5
with a mass m kg attached at one end. The
length, L cm, of the spring is given by the
formula L = 70 + 5m. Find
Level 3 L
(a) the length of the spring when m = 6,
3. The cost $C of screen printing x T-shirts is given by the (b) the value of m when L = 85.
formula m
C = 100 + 12x. Solution
(a) Find the cost of printing 50 T-shirts. (a) L = 70 + 5m
(b) If the budget is $976, how many T-shirts can be When m = 6,
printed? L = 70 + 5(6)
= 100
Solution i.e. the length of the spring is 100 cm.
(a) C = 100 + 12x
When x = 50, (b) When L = 85,
C = 100 + 12(50) 85 = 70 + 5m
= 700 5m = 85 – 70
The required cost is $700. 5m = 15
m = 3
(b) When C = 976,
976 = 100 + 12x 7. Give an equation of the form ax + b = c, where a, b and
12x = 976 – 100 c are constants, such that the solution of the equation is
12x = 876 x = 4.
x = 73
73 T-shirts can be printed. Solution
The equation below is of the form ax + b = c, where
4. In balancing the chemical reaction a = 4, b = 11 and c = 27.
nNaOH + H2SO4 → Na2SO4 + nH2O, 4x + 11 = 27
if we count the number of hydrogen (H) atoms on both x = 4
sides, the number n has to satisfy the equation n + 2 = The solution of the equation is x = 4.
2n. Find the value of n.
8. Give an example of a linear equation in one variable
Solution which has
n + 2 = 2n (a) more than one solution,
2n – n = 2 (b) no solution.
n = 2

155
Solution Solution
(a) The equation 2x + 3 = 2x + 3 is satisfied by any (a) 3x + 4 = 2(2x + 7)
value of x. 3x + 4 = 4x + 14
It has infinitely many solutions. 3x – 4x = 14 – 4
–x = 10
(b) The equation x + 3 = x has no solution.
x = –10

9. (a) Let x = 0.27. (b) 3(5x + 8) = 3x – 2


(i) Write down the values of 100x and 99x. 15x + 24 = 3x – 2
(ii) Hence, express x as a fraction in its simplest 15x – 3x = –2 – 24
form. 12x = –26
(b) Extending the technique in (a), express each of the –26
x =
12
following as fractions in its simplest form.
(i) 0.4 1
x = –2
6
(ii) 0.216
(c) 2(8x + 5) = 4(3x + 1)
Solution
16x + 10 = 12x + 4
(a) (i) 100x = 100 + 0.27
16x – 12x = 4 – 10
= 27.2727…
4x = –6
99x = 100x – x
3
= 27.2727… – 0.2727… x = –
2
= 27 1
x = –1
(ii) 99x = 27 2
27
x = (d) 3(4x – 1) = 7(2x – 5)
99
3
12x – 3 = 14x – 35
x = 12x – 14x = –35 + 3
11
–2x = –32
x = 16
(b) (i) Let x = 0.4
99x = 100x – x (e) 5(x + 3) – 4(2x – 9) = 0
99x = 100 × 0.4 – 0.4 5x + 15 – 8x + 36 = 0
99x = 44 –3x + 51 = 0
4 3x = 51
x =
9 x = 17
(ii) Let x = 0.216 (f ) 3(3x – 1) – 4(5 – 2x) = –10
999x = 1000x – x 9x – 3 – 20 + 8x = –10
999x = 1000 × 0.216 – 0.216 17x = –10 + 23
999x = 216 17x = 13
8 13
x = x =
37 17

(g) 9x – 2(x + 8) = 5x – 11
Exercise 6.2 9x – 2x – 16 = 5x – 11
7x – 16 = 5x – 11
Level 1 2x = 5
1. Solve the following equations. 5
x =
(a) 3x + 4 = 2(2x + 7) 2
(b) 3(5x + 8) = 3x – 2 1
x = 2
(c) 2(8x + 5) = 4(3x + 1) 2
(d) 3(4x – 1) = 7(2x – 5) (h) 1 – 4(2x + 3) = 5(x – 2) – 3(x – 1)
(e) 5(x + 3) – 4(2x – 9) = 0 1 – 8x – 12 = 5x – 10 – 3x + 3
(f) 3(3x – 1) – 4(5 – 2x) = –10 –8x – 11 = 2x – 7
(g) 9x – 2(x + 8) = 5x – 11 –8x – 2x = 11 – 7
(h) 1 – 4(2x + 3) = 5(x – 2) – 3(x – 1) –10x = 4
2
x = –
5

Chapter 6  Simple Equations in One Variable


156
2. Solve the following equations. ( f)
2(1 – 4x)
– 9 = 3(2 – x)
2 5
(a) 12 + x = 16 2(1 – 4x) – 45 = 15(2 – x)
5
4 2 – 8x – 45 = 30 – 15x
(b) 9 – x = –7 –8x – 43 = 30 – 15x
3
2x + 9 7x = 73
(c) =5 73
5 x =
3x – 11 7
(d) –2=0 3
7 x = 10
7
3(5x – 6)
(e) + 2 = 4x 7x – 2 5x – 3
4 (g) =
2(1 – 4x) 2 3
( f) – 9 = 3(2 – x) 3(7x – 2) = 2(5x – 3)
5
7x – 2 5x – 3 21x – 6 = 10x – 6
(g) = 11x = 0
2 3
2x + 3 3x – 15 x = 0
(h) =
3 11
2x + 3 3x – 15
(h) =
3 11
Solution
2
11(2x + 3) = 3(3x – 15)
(a) 12 + x = 16 22x + 33 = 9x – 45
5
2
13x = –78
x = 4 x = – 6
5
5
x = 4 ×
2
x = 10 Level 2
3. Solve the following equations.
4
(b) 9 – x = –7 (a) x +
x
= 15
3 4
4
– x = –16 (b)
x

x
=7
3 2 3
3 t+2
x = –16 × – (c)
t
– =2
4 7
5
x = 12 2t – 1 3t – 4
(d) + =t
3 5
2x + 9 2y 5y y
(c) = 5 (e) 3
+ 4 = –8
5 6
2x + 9 = 25 y+9 y–3 y
2x = 16 (f ) = –
2 4 3
x = 8 z–7 z–5 2z – 27
(g) – =
3 12 15
3x – 11
(d) – 2 = 0 4z + 3 7z – 1 2 – 19z
7
(h) – =
3x – 11 5 3 10
= 2
7
3x – 11 = 14 Solution
x
3x = 25 (a)
x+ = 15
4
25
x = 4x + x = 60
3
5x = 60
1
x = 8 x = 12
3
x x
(e)
3(5x – 6)
+ 2 = 4x (b) – = 7
2 3
4
3(5x – 6) + 8 = 16x 3x – 2x = 42
15x – 18 + 8 = 16x x = 42
x = –10

157
t t+2 Solution
(c) – = 2 5
5 7
(a) C = (F – 32)
7t – 5(t + 2) = 70 9
7t – 5t – 10 = 70 When C = 100,
2t = 80 100
= (F – 32)
5
t = 40 9
9
100 × = F – 32
2t – 1 3t – 4 5
(d) + = t 180 = F – 32
3 5
5(2t – 1) + 3(3t – 4) = 15t F = 212
10t – 5 + 9t – 12 = 15t The boiling point of water is 212 degree Fahrenheit.
19t – 17 = 15t (b) When C = 37,
4t = 17 5
37
= (F – 32)
17 9
t = 9
4 37 × = F – 32
1 5
t = 4 F = 66.6 + 32
4
2y 5y
F = 98.6
y
(e) 3
+ 4
= –8 The body temperature is 98.6 degree Fahrenheit.
6
4(2y) + 3(5y) = 2y – (12)8
8y + 15y = 2y – 96 5. The average, m, of two numbers a and b is given by
21y = –96 a+b
the formula m = . When a = 7 and m = 10, find the
96 2
y = – value of b.
21
4
y = – 4 Solution
7
a+b
y+9 y–3 y
m =
  2
(f ) = –
2 4 3 When a = 7 and m = 10,
6(y + 9) = 3(y – 3) – 4y 7+b
6y + 54 = 3y – 9 – 4y 10 =
2
6y + 54 = –y – 9 20 = 7 + b
7y = –63 b = 13
y = –9
z–7 z–5 2z – 27 6. The kinetic energy, E joules, of an object of mass m kg
(g) – = 1
3 12 15 moving with velocity v m/s, is given by E = mv2. Find
20(z – 7) – 5(z – 5) = 4(2z – 27) 2
20z – 140 – 5z + 25 = 8z – 108 the value of m when E = 160 and v = 8.
15z – 115 = 8z – 108
Solution
7z = 7
1
z = 1 E = mv2
2
4z + 3 7z – 1 2 – 19z When E = 160 and v = 8,
(h) – = 1
5 3 10 160 = × m × 82
2
6(4z + 3) – 10(7z – 1) = 3(2 – 19z)
160 = 32m
24z + 18 – 70z + 10 = 6 – 57z
160
– 46z + 28 = 6 – 57z m =
32
11z = –22
m = 5
z = –2

7. Two wooden blocks of masses m kg and n kg are moving


Level 3 on the ground in the same direction with speeds of p m/s
4. The conversion formula between F degree Fahrenheit and q m/s respectively. When they collide, they move
5 together with a speed of v m/s given by the formula
and C degree Celsius is C = (F – 32).
9 mp + nq = (m + n)v.
(a) The boiling point of water is 100 °C. Find the 1
If m = 5, p = 6, q = 4 and v = 5 , find the value of n.
boiling point of water in degree Fahrenheit. 4
(b) The body temperature is 37 °C. Find the body
temperature in degree Fahrenheit.

Chapter 6  Simple Equations in One Variable


158
Solution (c)
1
– 4 = 0
mp + nq = (m + n)v x
1 1 – 4x = 0
When m = 5, p = 6, q = 4 and v = 5 ,
4 4x = 1
1 1
5(6) + n(4) = (5 + n) 5 x =
4 4
105 21
30 + 4n = + n (d) 9 –
6
= 0
4 4
x
21 105
n – 4n = 30 – 9x – 6 = 0
4 4
5 15
9x = 6
n = x =
2
4 4
3
15 4
n = × 2 1
4 5
(e) – 3 =
n = 3 x 2
4 – 6x = x
x+a
–7x = – 4
8. Create an equation in variable x of the form = c, x =
4
b
7
where a, b and c are constants, such that the solution of
the equation is x = –1.  2
(f ) = 4
x–3
Solution 2 = 4(x – 3)
x–3 2 = 4x – 12
= –2
2 4x = 14
x – 3 = –4
x =
7
x = –1 2
1
x–3 x = 3
= –2 is an equation whose solution is x = –1. 2
2
7 2
(g) =
2x + 5 3
Exercise 6.3 21 = 2(2x + 5)
Level 1 21 = 4x + 10
1. Solve the following equations. 4x = 11
15 7 11
(a) = 3 (b) = –2 x =
x x 4
1 6 3
(c) – 4 = 0 (d) 9 – =0 x = 2
x x 4

2 1 2 7
(e) –3= (f ) = 4 (h) – 5 = 0
x 2 x–3 x–9
7 2 7 7 – 5(x – 9) = 0
(g) = (h) –5=0 7 – 5x + 45 = 0
2x + 5 3 x–9
5x = 52
52
Solution x =
5
15
(a) =3 x = 10
2
x
5
15 = 3x
x = 5
Level 2
7
(b) = –2 2. Solve the following equations.
x
5 7 3 1
7 = –2x (a) = (b) =
x+1 2x 2x – 5 x+1
7
x = – 4 –3 3 4
2 (c) = (d) =
2x – 1 x+1 5–x 1 – 2x
1
x = –3 t+5 3 7–z 1
2 (e) = (f) =–
t 2 3z 6
3y + 2 3u + 2 5
(g) = 4 (h) =
2y – 7 u–1 u–1

159
Solution 3u + 2 5
(h) =
5 7 u–1 u–1
(a) =
x+1 2x 3u + 2 = 5
5(2x) = 7(x + 1) 3u = 3
10x = 7x + 7 u = 1
3x = 7 But when u = 1, the denominator of the original
7 equation u – 1 is 0. There is no solution.
x =
3
1
x = 2
3
Level 3
3 1 n
(b) = 3. A fraction is given by . If the fraction is equal to
2x – 5 x+1 n+3
3
3(x + 1) = 2x – 5 , find the value of n.
4
3x + 3 = 2x – 5
Solution
x = –8
n 3
=
4 –3 n+3 4
(c) = 4n = 3(n + 3)
2x – 1 x+1
4(x + 1) = –3(2x – 1) 4n = 3n + 9
4x + 4 = –6x + 3 4n – 3n = 9
4x + 6x = 3 – 4 n = 9
10x = –1
1
x = – 4. In the diagram, a block of mass m kg on a smooth table
10
is pulled by another mass n kg hanging over a smooth
3 4 pulley. The acceleration, a m/s2, of the mass is given by
(d) = ng
5–x 1 – 2x
a= .
3(1 – 2x) = 4(5 – x) n–m
3 – 6x = 20 – 4x 1
If a = 3 , g = 10 and m = 8, find the value of n.
3
4x – 6x = 20 – 3
–2x = 17 Solution
17 ng
x = – a =
2 n–m
1
1 When a = 3 , g = 10 and m = 8,
x = –8 3
2 1 n × 10
3 =
3 n+8
t+5 3
(e) = 10 10n
t 2 =
3 n+8
2(t + 5) = 3t
2t + 10 = 3t
n + 8 = 3n
3t – 2t = 10 8 = 2n
t = 10 n = 4

7–z 1 5. The time t seconds for a car to increase its speed from
(f) = –
3z 6 u m/s to v m/s is given by
6(7 – z) = –3z 2d
t= ,
42 – 6z = –3z u+v
6z – 3z = 42 where d m is the distance covered by the car. If d = 40,
3z = 42 t = 5 and u = 4, find the value of v.
z = 14
Solution
3y + 2 2d
(g) =4 t =
2y – 7 u+v
3y + 2 = 4(2y – 7) When d = 40, t = 5 and u = 4,
3y + 2 = 8y – 28 2 × 40
5
=
–5y = –30 4+v
y = 6 4 + v = 16
v = 12

Chapter 6  Simple Equations in One Variable


160
6. The object distance, u cm, and the image distance, v cm, 3. There are 7 more Mathematics books than Science books
of a lens is related by the formula on a shelf. Let x be the number of Science books.
1 1 1 (a) Express the number of Mathematics books in terms
+ = ,
u v f of x.
where f cm is the focal length of the lens. If f = 20 and (b) If there is a total of 39 Mathematics and Science
u = 30, find the value of v. books, find the number of Science books.
Solution Solution
1 1 1 (a) Number of Mathematics books = x + 7
+ =
u v f
(b) x + (x + 7) = 39
When f = 20 and u = 30,
1 1
x + x + 7 = 39
1
+ = 2x = 32
30 v 20
1 1 1 x = 16
= – The number of Science books is 16.
v 20 30
1 3–2
= 4. Every week Rahim works 3 times as many hours as
v 60
1 1 Jinlan. Let the number of hours Jinlan work in a week
= be x hours.
v 60
v = 60 (a) Express the number of hours Rahim works in a
week in terms of x.
(b) If their total number of working hours in a week
7. Is x ÷ x always equal to 1? If not, when is x ÷ x not equal is 56 hours, find the number of hours Jinlan works
to 1? Explain briefly. in a week.
Solution Solution
x
When x ≠ 0, = 1, (a) Number of hours Rahim works in a week
x
x = 3x hours
when x = 0, is undefined.
x
(b) 3x + x = 56
x ÷ x is NOT always equal to 1. 4x = 56
x = 14
Jinlan works 14 hours in a week.
Exercise 6.4
Level 1 5. There are 3 consecutive even integers. Let x be the
smallest one.
1. Tim is 5 kg heavier than Amy. Let Amy’s mass be
(a) Express the other two integers in terms of x.
x kg.
(b) If the sum of these 3 integers is 66, find the integers.
(a) Express Tim’s mass in terms of x.
(b) If Tim’s mass is 63 kg, find Amy’s mass. Solution
(a) the middle integer = x + 2
Solution the largest integer = x + 4
(a) Tim’s mass = (x + 5) kg
x + (x + 2) + (x + 4) = 66
(b)
x + 5 = 63
(b) x + x + 2 + x + 4 = 66
x = 58 3x + 6 = 66
Amy’s mass is 58 kg. 3x = 60
x = 20
2. The price of a watch is $50 more than twice the price The integers are and 20, 22 and 24.
of a gold ring. Let the price of the ring be $x.
(a) Express the price of the watch in terms of x. 6. Mrs Chen buys 3 identical boxes of chocolates from a
(b) If the price of the watch is $208, find the price supermarket. She uses a $50 note to pay for them and
of the ring. gets $8 change. How much does each box of chocolates
cost?
Solution
(a) Price of the watch = $(2x + 50) Solution
Let the cost of each box of chocolates be $x.
(b) 2x + 50 = 208 3x + 8 = 50
2x = 158 3x = 42
x = 79 x = 14
The price of the ring is $79. Each box of chocolates costs $14.

161
7. The total price of a teapot and 4 identical cups is $64. Solution
If the price of the teapot is $28, find the price of a cup. Let $x be the amount won by the runner-up.
x + (2x + 100) = 2350
Solution 3x = 2250
Let the price of a cup be $x. x = 750
28 + 4x = 64 The runner-up won $750.
4x = 36
x = 9
12. A group of boys and girls planted a total of 148 trees.
The price of a cup is $9.
Each boy planted 7 trees and each girl planted 5 trees.
There were 4 more boys than girls in the group. How
8. A computer shop displays a total of 50 type A and type many boys were there in the group?
B mice. The cost of a type A mouse is $6 and that of a
type B mouse is $13. If the total cost of both types of Solution
mice is $433, find the number of type A mice displayed. Let x be the number of boys in the group.
Number of girls = x – 4
Solution 7x + 5(x – 4) = 148
Let the number of type A mice displayed be x. 7x + 5x – 20 = 148
Then number of type B mice = 50 – x. 12x = 168
6x + 13(50 – x) = 433 x = 14
6x + 650 – 13x = 433 There were 14 boys.
–7x = –217
x = 31
13 . Aaron, Ben and Chetan shared 110 marbles. Ben received
The number of type A mice displayed is 31.
twice as many marbles as Aaron. Chetan received 10
more marbles than Aaron. How many marbles did each
boy receive?
Level 2
9. Tom is twice as old as Arul. In 4 years’ time, the sum Solution
of their ages will be 32 years. Find Arul’s present age. Let Aaron’s share be x marbles.
Ben’s share = 2x marbles.
Solution Chetan’s share = (x + 10) marbles.
Let Arul’s present age be x years. x + 2x + (x + 10) = 110
(x + 4) + (2x + 4) = 32 4x = 100
3x + 8 = 32 x = 25
3x = 24 Aaron received 25 marbles, Ben received 50 marbles,
x = 8 and Chetan received 35 marbles.
Arul’s present age is 8 years.

14 . 144 coins are divided equally among some children.
10. Halim has 4 more $50 notes than $10 notes in his wallet. If there were 3 children fewer, each child would have
If the total amount of the notes is $380, how many 16 coins. How many children are there?
$10 notes does Halim have?
Solution
Solution Let the number of children be x.
Let the number of $10 notes Halim have be x. 144
10x + 50(x + 4) = 380 = 16
x–3
10x + 50x + 200 = 380 9
60x = 180 = 1
x–3
x = 3 9 =x–3
Halim has 3 $10 notes.
x = 12
There are 12 children.
11. In a Mathematics competition, the winner won $100
more than twice the amount won by the runner-up. How
much money did the runner-up win if both of them won 15. The denominator of a fraction is 3 more than its
$2350 altogether? numerator. If 2 is added to both the numerator and the
2
denominator, the new fraction is equivalent to . Find
3
the original fraction.
Hint: Let the numerator of the original fraction be x.

Chapter 6  Simple Equations in One Variable


162
Solution Solution
Let the numerator of the original fraction be x. Let x g be the mass of sulphur before the reaction.
Thus the denominator of the original fraction is x + 3. Mass of iron before the reaction = 3x g
x+2 2 Mass of iron after the reaction = (3x – 10)g
= Mass of sulphur after the reaction = (x – 6)g
(x + 3) + 2 3
x+2 2 From the given information,
= 3x – 10 = 4(x – 6)
x+5 3
3(x + 2) = 2(x + 5) 3x – 10 = 4x – 24
3x + 6 = 2x + 10 x = 14
x = 4 Mass of iron before the reaction
4
The original fraction is . = 3 × 14
7 = 42 g

19. Mrs Tan has some money to buy fruits. She can buy n
Level 3 mangoes at $1.60 each and have $0.80 left. Alternatively,
16. A triathlon is made up of swimming, cycling and running. she can buy (n + 10) apples at $0.70 each and have $0.10
The cycling distance is 4 times the running distance. left.
The swimming distance is 8.5 km less than the running (a) Find the value of n.
distance, and is 38.5 km less than the cycling distance. (b) How much money does Mrs Tan have for buying
Find the total distance of the race. fruits?
(c) If Mrs Tan buys 3 mangoes and uses the rest of the
Solution money to buy apples,
Let x km be the running distance. (i) how many apples can she buy?
cycling distance = 4x (ii) how much money will she have left?
swimming distance = x – 8.5
As swimming distance = cycling distance – 38.5, Solution
we have (a) Consider the sum of money for buying fruits.
x – 8.5 = 4x – 38.5 1.6n + 0.80 = 0.7(n + 10) + 0.10
3x = 30 Multiplying the above equation by 10:
x = 10 16n + 8 = 7(n + 10) + 1
Total distance 16n + 8 = 7n + 70 + 1
= x + 4x + (x – 8.5) 9n = 63
= 10 + 4(10) + (10 – 8.5) n = 7
= 51.5 km The value of n is 7.
(b) The required amount = $(1.6 × 7 + 0.80)
17. In the diagram, three currents X, Y and Z are flowing = $12
in a circuit. The current Y is 1 ampere less than twice (c) (i) The amount left after buying 3 mangoes
the current X. The current Z is 2 amperes more than the = $12 – $1.60 × 3
current Y. The total current of X, Y and Z is 15 amperes. = $7.20
How many amperes is current X? Let the number of apples bought be x.
0.7x  7.2
Solution
7.2
Let x amperes be the current X. x 
0.7
Current Y = (2x – 1) amperes
Current Z = (2x – 1) + 2 2
x  10
= (2x + 1) amperes 7

x + (2x – 1) + (2x + 1) = 15 She can buy 10 apples.
x + 2x – 1 + 2x + 1 = 15
(ii) The amount left = $7.20 – $(0.70 × 10)
5x = 15
= $0.20
x = 3
Current X is 3 amperes.
20. Write an application problem such that the equation to
be formed for solving the problem is
18. Before a chemical reaction, the mass of iron is 3 times 5x + 4(x – 10) = 140.
the mass of sulphur. After the reaction, the masses of iron
and sulphur are reduced by 10 g and 6 g respectively,
and the mass of iron is 4 times the mass of sulphur. Find
the mass of iron before the reaction.

163
Solution (b) 2(5x – 8) + 6 = 11
Consider the following problem. 10x – 16 + 6 = 11
In a class, the number of girls is 10 fewer than the number 10x – 10 = 11
of boys. Each boy runs 5 laps and each girl runs 4 laps. 10x = 21
The total number of laps run by the boys and girls is 21
x =
140. Find the number of boys in the class. 10
The equation to be formed is as follows: 1
x = 2
10
5x + 4(x – 10) = 140
x represents the number of boys and x – 10 represents 2x x
the number of girls. (c) + = 13
3 5
10x + 3x = 195
21. The average travel times from HarbourFront MRT station 13x = 195
to Serangoon MRT station on the North East Line and on x = 15
the Circle Line are (2x + 5) minutes and (5x – 4) minutes
respectively. The difference in the average travel times 4
1
(d) 1 – x = 23 + x
is hours. 7
4 7 – 4x = 161 + 7x
(a) Form an equation in x and solve it. –11x = 154
(b) Hence, find the average travel time from x = –14
HarbourFront MRT station to Serangoon MRT
station on the North East Line.
4x – 5 7x – 3
(e) =
2 9
Solution
(a) The required equation is 9(4x – 5) = 2(7x – 3)
(5x – 4) – (2x + 5) = × 60 36x – 45 = 14x – 6
i.e. (5x – 4) – (2x + 5) = 15 22x = 39
39
5x – 4 –2x – 5 = 15 x =
22
3x – 9 = 15 17
3x = 24 x = 1
22
x = 8
(b) The required time x–4 2x + 1 5x – 1
(f ) – =
= 2(8) + 5 3 6 2
= 21 min 2(x – 4) – (2x + 1) = 3(5x – 1)
2x – 8 – 2x – 1 = 15x – 3
–9 = 15x – 3
Revision Exercise 6 15x = –6
2
1. Solve the following equations. x = –
5
(a) 13x – 22 = 30
(b) 2(5x – 8) + 6 = 11 2
(g) = 6
2x x x–7
(c) + = 13 2 = 6(x – 7)
3 5
4 2 = 6x – 42
(d) 1 – x = 23 + x 6x = 44
7
4x – 5 7x – 3 22
(e) = x =
2 9 3
x–4 2x + 1 5x – 1 1
(f ) – = x = 7
3 6 2 3
2
(g) =6
x–7
4x – 1 4x – 1 5
(h) =
5 (h) =
5x + 1 5x + 1 7
7
7(4x – 1) = 5(5x + 1)
Solution 28x – 7 = 25x + 5
(a) 13x – 22 = 30 3x = 12
13x = 52 x = 4
x = 4

Chapter 6  Simple Equations in One Variable


164
2. Given the formula D = b2 – 4ac, find 5. Peter has 96 stamps and Ajit has 63 stamps. How many
(a) the value of D when a = 1, b = –5 and c = 3, stamps should Ajit give Peter so that Peter will have
(b) the value of c when a = 2, b = 3 and D = 49. twice as many stamps as Ajit?

Solution Solution
(a) When a = 1, b = –5 and c = 3, Let x be the number of stamps that Ajit gives to Peter.
D = (–5)2 – 4(1)(3) 96 + x = 2(63 – x)
= 25 – 12 96 + x = 126 – 2x
= 13 3x = 30
x = 10
(b) When a = 2, b = 3 and D = 49,
Ajit should give Peter 10 stamps so that Peter will have
49 = 32 – 4(2)c
twice as many stamps as Ajit.
49 = 9 – 8c
8c = – 40
c = –5 6. A boy is 26 years younger than his father. In 3 years’
1
n(a + b) time, his age will be his father’s age. Find the present
3. Given the formula S = , find 3
2 age of the boy.
(a) the value of S when a = 1, b = 25 and n = 12,
(b) the value of a when b = 41, n = 15 and S = 330. Solution
Let the boy’s present age be x years.
Solution His father’s present age = (x + 26) years
(a) When a = 1, b = 25 and n = 12, 1
x + 3 = [(x + 26) + 3]
12(1 + 25) 3
S =
2 1
x + 3 = (x + 29)
= 156 3

(b) When b = 41, n = 15 and S = 330, 3x + 9 = x + 29


2x = 20
15(a + 41)
330
= x = 10
2
2
The boy’s present age is 10 years.
a + 41 = 330 ×
15
a + 41 = 44 7. The price of a skirt is $25 more than the price of a
a = 3 T-shirt. The total price of 3 skirts and 8 T-shirts is $339.
Find the price of a skirt.
4. The lengths of the sides of a triangle are (2x + 1) cm, Solution
(3x + 2) cm and (4x – 1) cm. Let $x be the price of a skirt.
(a) Find the perimeter of the triangle in terms of x. 3x + 8(x – 25) = 339
(b) If the perimeter of the triangle is 47 cm, find the 3x + 8x – 200 = 339
value of x. 11x = 539
x = 49
The price of a skirt is $49.
+2
3x
+1

8. In a certain week, the amount of time Lisa spent on


2x

watching television was 3 hours more than twice the


time she spent on doing her Mathematics homework.
4x – 2 If the total time she spent on these two activities was
30 hours in that week, how many hours did Lisa spend
on doing her Mathematics homework?
Solution
(a) Perimeter of the triangle Solution
= [(2x + 1) + (3x + 2) + (4x – 1)] Let the amount of time Lisa spent on doing her
= (2x + 1 + 3x + 2 + 4x – 1) Mathematics homework be x hours.
= (9x + 2) cm x + (2x + 3) = 30
3x + 3 = 30
(b) 9x + 2 = 47
3x = 27
9x = 45
x = 9
x = 5
Lisa spent 9 hours on doing her Mathematics homework.

165
9. The number of books in a class library is 17 more than
3 times the number of students in the class. If 5 students
are absent, each student can borrow exactly 4 books from
the library. Find the number of students in the class.

Solution
Let x be the number of students in the class.
3x + 17
=4
x–5
3x + 17 = 4x – 20
x = 37
The number of students in the class is 37.

10. In a flower fertilisation experiment, the number of


pink flowers yielded is twice that of red flowers. The
number of white flowers yielded is 3 more than that of
red flowers. The number of white flowers yielded is 37
less than that of pink flowers. Find the total number of
flowers yielded.

Solution
Let x be the number of red flowers.
Number of pink flowers = 2x
Number of white flowers = x + 3
From the given information,
2x – (x + 3) = 37
2x – x – 3 = 37
x = 40
Total number of flowers
= x + 2x + (x + 3)
= 4x + 3
= 4(40) + 3
= 163

Chapter 6  Simple Equations in One Variable


166
7 Angles and Parallel Lines

Class Activity 1
Objective: To describe the features of points, lines, rays and line segments.

Tasks

Point, Line, Ray and Line Segment

Tool Object Observations from the object

Point tool Point A point has position but no size.

Segment tool Line Segment A line segment has length and two end points.

Ray tool Ray A ray has one fixed end point and can be extended indefinitely in one direction.

Line tool Line A line has no width and can be extended indefinitely in two directions.

Questions
1. Based on your observations, describe the differences between a line, a line segment and a ray.

A line consists of infinitely many points. A ray is a part of a line and it has a fixed end point. A line segment is a part of a line with two fixed end points.

2. From the task above, the points and the lines lie on a flat surface. What is the name of this flat surface? What features
does this flat surface have?

The points and the lines lie on a plane. It has no thickness.

167
Class Activity 2
Objective: To investigate the properties of angles formed by two parallel lines and a transversal.

Angles Formed by Two Parallel Lines and a Transversal

Questions
1. Observe and name the pairs of corresponding angles between the parallel lines. What is the relationship between two
corresponding angles?

The pairs of corresponding angles are ∠q and ∠b ; ∠r and ∠c; ∠p and ∠a; ∠s and ∠d.

The corresponding angles are equal.

2. Observe and identify the pairs of alternate angles between the parallel lines. What is the relationship between two alternate
angles?

The pairs of alternate angles are ∠s and ∠b; ∠r and ∠a.

The alternate angles are equal.

3. Observe and identify the pairs of interior angles between the parallel lines. What can you say about the sum of each pair
of interior angles?

The pairs of interior angles are ∠r and ∠b ; ∠s and ∠a.

The sum of each pair of interior angles is 180˚.

Chapter 7  Angles and Parallel Lines


168
Class Activity 3
Objective: To investigate the property of a perpendicular bisector of a line segment.

Perpendicular Bisector

Questions
1. Based on your observation, what is the relationship between the line segment AB and the line PQ?

Line segment AB is perpendicular to line segment PQ.

2. Describe the relationship between a point on the perpendicular bisector of a line segment and its end points.

The point on the perpendicular bisector of a line segment is equidistant to its end points.

169
Class Activity 4
Objective: To investigate the property of an angle bisector of an angle.

Angle Bisector

Questions
1. Based on your observation, what is the relationship between ∠BAP and AZ? Explain your answer.

AZ bisects ∠BAC into exactly two halves, ∠BAP and ∠CAP.

2. Based on your observation, what do you think the relationship between the point P, and the sides AB and AC is?

Point P is the shortest distance to sides AB and AC.

3. Describe the relationship between a point on the angle bisector of an angle and its sides.

A point on the angle bisector is equidistant from its sides.

Chapter 7  Angles and Parallel Lines


170
Try It! 4. In the figure, PS, QT and RU are straight lines, intersecting
at V. Find the value of z.
Section 7.1
1. The figure shows four points A, B, C and D on a plane.
(a) Draw the lines formed by these points. How many
lines are there?
(b) Name the line segments that can be formed by the
points A, B and C.
(c) Name three rays that can be formed by the points
B, C and D.

Solution
(a) There are 6 lines.
Solution
∠UVT = 70°  (vert. opp. ∠s)
z° + ∠UVT + z° = 180° (adj. ∠s on a st. line)
z° + 70° + z° = 180°
2z = 110
z = 55
(b) Line segments AB, BC, CD, AD, AC and BD can
be formed by points A, B and C. Section 7.3
(c) Rays BC, CB, CD, DC, DB and BD can be formed 5. In the figure, AB // CD.
by points B, C and D. Find the angles p, q and r.

Section 7.2
2. In the figure, XYZ is a straight line. Find the value of w.

Solution
Solution ∠p = 115° (alt. ∠s, AB // CD)
w° + 140° + w° = 180°  (adj. ∠s on a st. line) ∠r = 115°  (corr. ∠s, AB // CD)
2w = 40
∠q + 115° = 180°  (int. ∠s, AB  // CD)
w = 20 ∠q = 65°

3. Find the value of x in the figure. 6. In the figure, AB // DE, BC // EF, BHC and DHE are
straight lines. Find the values of x and y.

Solution
5x° + x° + 3x° + 54° = 360°  (∠s at a point)
9x + 54 = 360
9x = 306 Solution
x = 34 ∠ABH + ∠DHB = 180°  (int. ∠s, AB // DE)
72° + y° + 23° = 180°
y = 85
∠DHC = ∠ABH (corr. ∠s, AB // DE)
= 72°
x° = ∠DHC (corr. ∠, BC // EF)
= 72°
x = 72

171
7. In the figure, BA // DE. Find the angle x.

Solution

Construct CF // BA // DE.
∠x1 = 20°  (alt. ∠s, CF // DE)
55° + ∠x2 = 180° (int. ∠s, CF // BA)
∠x2 = 125°
  ∠x = ∠x1 + ∠x2
= 20° + 125°
= 145°

Chapter 7  Angles and Parallel Lines


172
Exercise 7.1 Level 3
Level 1 5. In the figure, M is a point on the line segment
1. In the diagram, A and B are two points on 1
PQ such that PM = PQ. Show that PM = MQ.
2
a plane. How many lines can be drawn
passing through
(a) A,
(b) both A and B?

Solution Solution
(a) infinite Line segment PQ is made up of line segments PM and
(b) 1 MQ.
PM + MQ = PQ
1 1
2. In the diagram, C and D are two PQ + MQ = PQ Replace PM with PQ
2 2
points on a plane. Draw the ray
1 1
CD, where C is the end point. MQ = PQ – PQ Replace PQ with PM
2 2
You may trace the points on your
1
exercise book. MQ = PQ
2

Solution MQ = PM (shown)

6.


The above figure shows a line segment ABCD in which
AB = CD.
Level 2 (a) State the relation between AC and BD.
(b) Give the reason for the result in (a).
3. In the figure, A, B and C are three points not on a straight
line.
Solution
(a) Name the straight lines that can be formed.
(a) AC = BD
(b) Name the rays with the end point A.
(b) AB = CD (given)
Solution AB + BC = BC + CD
(a) AB, BC, AC ∴  AC = BD
(b) BA, CA
7. In the diagram, AB and CD are two line segments, and
AB is shorter than CD.
4. In the diagram, A, B and C are three points on a straight
line.
(a) Find the number of different line segments that can
be formed by these points.
(b) Find the number of different rays that can be formed
by these points.
(a) How many points are on the line segment
Note: When three points lie on a straight line, they
(i) AB?
are said to be collinear.
(ii) CD?
(b) Is it true that there are more points on CD than on
AB?

Solution
(a) (i) infinite

Solution (ii) infinite


(a) 3 line segments can be formed. They are AB, BC (b) No
and AC.
(b) 6 rays can be formed. They are AB, BC, AC, CA,
CB and BA.

173
Exercise 7.2 Solution
Level 1 (a) ∠x = 33°  (vert. opp. ∠s)
33° + ∠y = 180°  (adj. ∠s on a st. line)
1. In each figure, AOB is a straight line. Calculate each ∠y = 147°
unknown marked angle.
(a) (b) (b) ∠BFC = ∠AFD  (vert. opp. ∠s)
∠p + 76° = 128°
∠p = 52°
∠q + 128° = 180°  (adj. ∠s on a st. line)
∠q = 52°

Solution Level 2
(a) 40° + 90° + ∠a = 180°  (adj. ∠s on a st. line) 4. In the figure, ABC is a straight line.
∠a = 50°
(b) 61° + ∠c + 31° + 18° = 180° (adj. ∠s on a st. line)
∠c = 70°

2. Find the unknown angle x in each figure.


(a) (b) (a) Find the value of x.
(b) What type of angle is ∠ABD?

Solution
(a) x° + x° = 180°  (adj. ∠s on a st. line)
2x = 180
x = 90
(b) ∠ABD is a right angle.

Solution 5. In the figure, LMN is a straight line.
(a) ∠x + 125° + 90° + 64° = 360°  (∠s at a point)
∠x = 81°
(b) ∠x + 60° + 78° + 89° + 54° = 360°(∠s at a point)
∠x = 79°
(a) Find the value of y.
3. In each of the figures, the straight lines AB and CD (b) What type of angle is
intersect at a point. Find the unknown marked angles. (i) ∠PMN?
(ii) ∠LMN?
(a)
Solution
(a) 134° + y° + y° = 180°  (adj. ∠s on a st. line)
2y = 46
y = 23
(b) (i) ∠PMN = 2y°
= 46°
(b) ∠PMN is an acute angle.
(ii) ∠LMN = 180°
∠LMN is a straight angle.

6. In the figure, PQR is a straight line.


Chapter 7  Angles and Parallel Lines


174
(a) Find the value of x. 9. In the figure, the lines AD, BE and CF intersect at the
(b) What type of angle is ∠SQT? point G. Find the values of a, b and c.

Solution
(a) 2x° + 5x° + x° = 180°  (adj. ∠s on a st. line)
8x = 180
x = 22.5
(b) ∠SQT = 5 × 22.5°
= 112.5°
∠SQT is an obtuse angle. Solution
38° + a° + 55° = 180° (adj. ∠s on a st. line)
a = 87
7. Find the value of x in the figure. b° = 55°  (vert. opp. ∠s)
b = 55
c° = a°  (vert. opp. ∠s)
c = 87

Level 3
10. In the figure, what type of angle is
(a) ∠a,
Solution a
(b) ∠b,
x° + 50° + 3x° + 70° + 4x° = 360°  (∠s at a point)
(c) ∠c, b
8x = 240
(d) ∠d ? d
x = 30 c

Solution
8. In each figure, three lines intersect at a point. Find the (a) Right angle
value of x. (b) Reflex angle
(a) (b) (c) Acute angle
(d) Obtuse angle

11. The clock shows 4 o’clock. Find ∠x and ∠y.


x
Solution
y
(a)

Solution
4
∠x = × 360°
12
t° = 84° (vert. opp. ∠s) = 120°

x° + t° + x° = 180 (adj. ∠s on a st. line) 8
2x = 96 ∠y = × 360°
12
x = 48 = 240°
(b)
12. In the figure, an end of each wooden frame ABCD and
EFGH is cut to form an angle x such that ∠ABC + ∠FGH
is aright angle. Find the value of x.

y° = x°  (vert. opp. ∠s)


2x° + y° + 2x° = 180° (adj. ∠s on a st. line)
5x = 180
x = 36

175
Solution Solution

x° + x° = 90° 2x° + 7x° = 180°  (adj. ∠s on a st. line)
2x = 90 9x = 180
x = 45 x = 20
2x° = y° (vert. opp. ∠s)
y = 40
13. The figure shows a logo. Find the value of y.

17. In the figure, ABC is a


straight line. Write down
(a) 2 acute angles,
(b) 2 obtuse angles,
found in the
figure.

Solution Solution
y° + y° + y° = 360°  (∠s at a point) (a) ∠ABF, ∠FBE, ∠ABE and ∠CBD are acute angles.
3y = 360 (b) ∠ABD, ∠FBD, ∠FBC and ∠EBC are obtuse angles.
y = 120
18. In the figure, AD, BE and CF are straight lines which
14. A circular pizza is divided into 10 equal pieces by intersect at G.
5 cuts through its centre as shown. Find ∠x. (a) Find two pairs of possible values of x and y.
(b) If y is 3 times of x, find the values of x and y.

Solution
360° Solution
∠x = (∠s at a point)
10 (a) ∠AGB = ∠DGE (vert. opp. ∠s)
∠x = 36° = 50°
x° + ∠AGB + y° = 180° (adj. ∠s on a st. line)
15. In the figure, AB is a plane mirror. A light ray PQ hits the x + 50 + y = 180
mirror at Q and is reflected along QR such that ∠AQP x + y = 130
= ∠BQR = x°. If ∠PQR = 110°, find the value of x. The two pairs of possible values of x and y are:
x = 30 and y = 100,
P R x = 40 and y = 90.
(b) If y = 3x, we have
110° x + 3x = 130
x° x° 4x = 130
A Q B   x = 32.5
and y = 3 × 32.5 = 97.5
Solution

x° + 110° + x° = 180 (adj. ∠s on a st. line)
2x = 70 Exercise 7.3
x = 35 Level 1
1. Find the unknown marked angles in each of the following
16. The figure shows a pair of tongs. Find the values of figures.
x and y. (a) (b)

7x°
2x° y°


Chapter 7  Angles and Parallel Lines
176
(c) (d) ∠EBC = 180° – 90°
(g) (adj. ∠s on a st. line)
= 90°
∠s = ∠EBC (alt. ∠s, BE//FC)
= 90°
∠t = 180° – ∠s (adj. ∠s on a st. line)
= 180° – 90°
= 90°
(h) ∠x + 63° = 180° (int. ∠s, AB//DC)
(e) (f ) ∠x = 117°
∠y + 63° = 180° (int. ∠s, AD//BC)
∠y = 117°

2. Find the unknown angle x in each of the following figures.


(a)

(g) (h )

Solution (b)
(a) ∠a = 50° (vert. opp. ∠s)
∠b = ∠a  (alt. ∠s, AB // CD)
  ∠b = 50°
∠c + ∠a = 180°  (int. ∠s, AB // CD)
∠c + 50° = 180°
∠c = 130°
(b) ∠AHF = ∠AGD  (corr. ∠s, CD // EF)
= 120°
∠d + ∠AHF = 180°  (adj. ∠s on a st. line) (c)
∠d + 120° = 180°
∠d = 60°
∠e = ∠AHF  (vert. opp. ∠s)
= 120°
(c) ∠ f = 113°  (alt. ∠s, DE // CF)
∠g + ∠ f = 180°  (adj. ∠s on a st. line)
∠g + 113° = 180° C D
∠g = 67° (d)

(d) ∠h = 110°  (alt. ∠s, AC // DF) x


A B 113°
∠h + 24° + ∠k = 180°  (adj. ∠s on a st. line)
110° + 24° + ∠k = 180° E F
∠k = 46° Solution
(e) ∠m = 38°  (alt. ∠s, AB // DC) (a) ∠GEF = ∠ABE (corr. ∠s, AB // GE)
∠ABC + ∠BCD = 180°  (int. ∠s, AB // DC) = 75°
67° + 38° + ∠n = 180° ∠x = ∠GEF  (corr. ∠s, EF // CD)
∠n = 75°   ∠x = 75°

(f ) ∠p + 62° + 47° = 180°  (adj. ∠s on a st. line) (b)
∠p = 71°
∠q = 47°  (alt. ∠s, AC // ED)
∠r = ∠p  (alt. ∠s, AC // ED)
  ∠r = 71°


177
∠y = 64°  (corr. ∠s, PQ // RS) (b)
∠z = ∠y  (corr. ∠s, AB // CD)
∠z = 64°
∠x + ∠z = 180°  (adj. ∠s on a st. line)
∠x + 64° = 180°
∠x = 116°
(c) Produce BE to a point G. Construct CF // AB // ED.
F ∠x = 53° (alt. ∠s, ED // CF)
G
260° + ∠x + ∠z = 360° (∠s at a point)
C
D 260° + 53° + ∠z = 360°
x
∠z = 47°
A E 35° ∠y = ∠z (alt. ∠s, AB // CF)
68°
  ∠y = 47°
B

∠DEG = 68°  (corr. ∠s, AB // DE) (c)
∠FEG = 35°  (corr. ∠s, BC // EF)
∠x = ∠DEG + ∠FEG
= 68° + 35°

= 103°
(d) ∠CDE = 113°  (alt. ∠s, CD // EF)
Construct CF // BA // DE.
∠BCD + ∠CDE = 180°  (int. ∠s, BC // ED)
∠x + 153° = 180°  (int. ∠s, CF // DE)
∠BCD + 113° = 180°
∠x = 27°
∠BCD = 67°
∠x + ∠y + 115° = 180°  (int. ∠s, CF // BA)
∠x = ∠BCD (alt. ∠s, AB // CD)
27° + ∠y + 115° = 180°
∴  ∠x = 67°
  ∠y = 38°

Level 2 (d)
3. Find the unknown angle y in each of the following figures.
(a) (b)

Construct GC and HD which are parallel to AB and


EF.
(c) (d) ∠a = 20°  (alt. ∠s, HD // EF)
∠b = 65° – ∠a
= 65° – 20°
= 45°
∠x + ∠b = 180°  (int. ∠s, HD // GC)
∠x + 45° = 180°
∠x = 135°
∠z + 30° = 180°  (int. ∠s, AB // GC)
Solution ∠z = 150°
  ∠y = ∠x + ∠z
(a)
= 135° + 150°
= 285°

4. In the figure, AB // CD and SEFT is a straight line. Find


the values of x and y.
Construct FC // AB // ED.
∠c1 + 150° = 180°  (int. ∠s, ED // FC)
∠c1 = 30°
120° + ∠c2 = 180°  (int. ∠s, AB // FC)
∠c2 = 60°
  ∠y = ∠c1 + ∠c2
= 30° + 60°
= 90°

Chapter 7  Angles and Parallel Lines


178
Solution Solution
2x° + 3x° = 180° (int. ∠s, AB // CD) x = 180° – 105°
5x = 180 = 75° (int ∠s, CD//BA)
  x = 36 ∠CDE = x
  y° = 3x°  (vert. opp. ∠s) = 75° (alt ∠s, BC//DE)
  y = 108 y = 360° – ∠CDE
= 360° – 75°
= 285° (∠s at a point)
5. In the figure, AC//DE and BCD is a straight line. Find
the angles x and y.
8. In the figure, ABCDE is the roof of a house, GB//FD
and ∠ACE = 106°. Find the angle x.

Solution
Solution
∠ACD = ∠CDE (alt. ∠s, AC // DE)
∠BDC = ∠CDB
35° + ∠x = 110°
(180 – 106)°
∠x = 75° =
2
∠y = 180° – (35 + x)°
= 37° (base ∠s of isos )
∠y = 180° – 110°
∠GBD = ∠FDB
∠y = 70°
= 90°
∠x = 180° – 37° – 90°
= 53° (adj ∠s on st. line)
Level 3
6. In a mechanical system, rods are joined together as shown
in the diagram. If AB // DC and AD // BC, 9. In the figure, a line PQ is drawn through the vertex A
and parallel to the side BC of ABC.

(a) find ∠ADY, (a) Which angle is equal to ∠b?


(b) name the geometrical shape ABCD. (b) Which angle is equal to ∠c?
(c) Find ∠a + ∠p + ∠q.
Solution (d) Find ∠a + ∠b + ∠c.
(a)
∠BCD = 72°  (alt. ∠s, AX // DC) (e) What can you conclude from the result in (d)?
∠ADY = ∠BCD (corr. ∠s, AD // BC)
∠ADY = 72° Solution
(a) ∠b = ∠p (alt. ∠s, PA // BC)
(b) Since both pairs of opposite sides are parallel, (b) ∠c = ∠q (alt. ∠s, AQ // BC)
ABCD is a parallelogram. (c) ∠a + ∠p + ∠q = 180°  (adj. ∠s on a st. line)
(d) Substituting ∠b and ∠c into the result in (c),
7. The figure shows 2 steps in which BA//CD//EF, ∠a + ∠b + ∠c = 180°
BC // DE and ∠ABC = 105°. Find the angles x and y. (e) The angle sum of a triangle is 180°.

10. Perform the following tasks using The Geometer’s


Sketchpad.

179
(a) Draw two parallel lines L1 and L2. Steps
(b) From a point A on the line L 1 , draw a ray 1. Draw a circle with centre O and radius 2 cm.
perpendicular to L1 to cut the line L2 at D. 2. Construct a diameter AOB of the circle.
(c) From a point C on the line L 2 , draw a ray 3. Draw the semicircle ACO of radius 1 cm.
perpendicular to L2 to cut the line L1 at B. 4. Draw the semicircle BDO of radius 1 cm.
(d) What can you say about the line segments AD and Then we get the required diagram.
BC?
(e) What type of quadrilateral is ABCD?
2. Draw a line segment AB 4 cm long and construct the
perpendicular bisector of AB using a ruler and compasses.
Solution
(d) AD = BC and AD // BC
Solution
(e) Since L1 // L2 and AD // BC, ABCD is a parallelogram.
(a)
Since ∠BAD = 90°, ABCD is a rectangle.

Exercise 7.4
Level 1
1. Draw the following diagram using a ruler and compasses.
(a) radius of each circle = 1.5 cm and the centre of
each circle lies on the other two sides.

Steps for constructing the perpendicular bisector of AB:


1
1. With A as centre and a radius  AB, draw an arc
2
on each side of AB.
(b) 2. With B as centre and same radius as in step 1, draw
two arcs to cut the previous arcs at C and D. Then
CD is the ⊥ bisector of AB.

  3. Draw a line segment 6 cm long and divide it into 4 equal
parts using a ruler and compasses.

Solution Solution
(a)
A

1.5 cm
B C

Steps
1. Draw a circle with centre at A and radius 1.5 cm.
2. Draw a circle with a point B on the first circle as
the centre and radius 1.5 cm.
3. Let C be the point of intersection of the first two
circles. Draw a circle with centre at C and radius
1.5 cm.
Then we get the required diagram.

(b) A In the figure, AB = 6 cm.


Steps for dividing AB into 4 equal parts are:
C
1. Draw the ⊥ bisector CD of AB to cut AB at its
O midpoint M.
D
2. Draw the ⊥ bisector EF of AM to cut AM at its
midpoint N.
B
3. Draw the ⊥ bisector GH of MB to cut MB at its
midpoint P.
Then AN = NM = MP = PB.

Chapter 7  Angles and Parallel Lines


180
4. Draw each angle using a protractor and construct its Level 2
angle bisector using a ruler and compasses. 6. Draw the following figure using a protractor, a ruler and
(a) (b) compasses.
(a)

Solution

(b)

(a) Steps for constructing the angle bisector of


∠BAC: Solution
1. With A as centre, draw an arc to cut AB and (a) Y
AC at D and E respectively.
2. With D and E as centres and equal radii, draw
B
arcs to cut at point Z.
3. Join AZ.
Then AZ is the angle bisector of ∠BAC.

X
(b) O A
Steps
1. Draw a right angle, ∠XOY, using a protractor.
2. With centre at O and radius 2.5 cm, draw an
arc to cut OX at A and OY at B.
3. Draw the line segments AB, OA and OB.
Then we get the required diagram.

Steps are similar to those in (a). (b)


XT is the angle bisector of ∠YXZ.
B Q
5. Draw an angle of 200° using a protractor and divide it C O

into 4 equal angles using a ruler and compasses.


A
P
Solution
Steps
1. Draw a reflex angle ∠POQ of 300° using a
protractor.
2. With O as centre and radius 1.5 cm, draw the
arc ACB which cuts OP at A and OQ at B.
3. Draw the line segments OA and OB.
Then we get the required diagram.

7. Draw a line segment ABC with AB = 1.5 cm and


In the figure, reflex ∠AOB = 200°. BC = 2.5 cm as shown in the figure.
Steps for dividing reflex ∠AOB into 4 equal angles are:
1. Construct OD to divide reflex ∠AOB into 2 equal
angles, ∠AOD and ∠BOD.
2. Construct the angle bisectors OC and OE of ∠AOD (a) Construct the perpendiculars PM and QN of AB
and ∠BOD respectively. and BC, where M and N are the midpoints of AB
Then ∠AOC = ∠COD = ∠DOE and ∠EOB. and BC respectively.
(b) How are the lines PM and QN related geometrically?
(c) What is the relation between the lengths of MN
and AC?

181
Solution (b) Diagram is not drawn to scale.
(b)
P

R S
T
3 cm

A B
M 4 cm

(c) PM and QN are parallel. Q


1
(d) MB = AB PQ is the perpendicular bisector of AB.
2
RS is the perpendicular bisector of BC.
1
= × 1.5 cm PQ and RS intersect at T.
2
= 0.75 cm (c) Let M and N be the midpoints of AB and BC
1 respectively.
BN = BC ∠PMB + ∠MBN = 90° + 90°
2
1 = 180°
= × 2.5 cm ∴  PM//CB (sum of int. ∠s = 180°)
2
= 1.25 cm ∠MTN + ∠TNB = 180° (int. ∠s, PM//CB)
MN = MB + BN ∠MTN + 90° = 180°
= (0.75 + 1.25) cm ∠MTN = 90°
= 2 cm
Since AC = 4 cm, 9. Copy the following figure, where ABC is a straight line
MN =
1
AC. and ∠ABD = 110°.
2 (a) Construct the bisectors BS and BT of ∠ABD and
∠CBD respectively using a ruler and compasses
8. (a) Draw the given figure. only.
(b) Construct the perpendicular bisectors of AB and (b) Find the size of ∠SBT, giving reasons for your
BC using a ruler and compasses. steps.
(c) Giving reasons for your answers, find the angle
where the perpendicular bisectors in (b) intersect.

110°

(a) Given that ∠ABD = 110°, construct the bisectors


BS and BT of ∠ABD and ∠CBD respectively using
a ruler and compasses.
Solution (b) What is the size of ∠SBT ? Give a reason for this.
(a)
Solution
(a)

1
(b) ∠SBD = ∠ABD
2
1
= × 110°
2
= 55°

Chapter 7  Angles and Parallel Lines


182
∠CBD = 180° – 110°  (adj. ∠s on a st. line) Level 3
= 70° 11.
1
∠DBT = ∠CBD
2
1
= × 70°
2
= 35°
∴  ∠SBT = ∠SBD + ∠DBT
= 55° + 35°
= 90° (a) Using a ruler and compasses,
(i) draw a triangle ABC with AB = 4 cm,
10. Draw the given figure using set squares. BC = 3 cm and AC = 3.5 cm,
(ii) construct the perpendicular bisectors of the
3 sides of the triangle.
(b) What do you observe about the perpendicular
bisectors?

Solution
(a) (i) and (ii)
(a) Find ∠BCD.
(b) Bisect ∠ABC and ∠BCD using a ruler and
compasses.
(c) Suppose the angle bisectors in (b) meet at T.
Find ∠BTC, giving reasons for your answer.

Solution
(a)

(b) The three perpendicular bisectors meet at a point


O. If we draw a circle with centre O and radius
OA, the circle will touch the vertices A, B and C.
(b) ∠BCD + 60° = 180°  (int. ∠s, BA // CD)

∠BCD = 120°
12.
(c) As shown in the diagram in (a),
BT is the angle bisector of ∠ABC.
CT is the angle bisector of ∠BCD.
(d) Construct ST // BA.
1
∠p = ∠BCD
2
1
= × 120°
2
= 60° (a) Using a ruler and compasses,
∠x = ∠p (alt. ∠s, ST // CD) (i) construct an angle of 60°,
= 60° (ii) draw a triangle ABC with AB = 4 cm,
1
AC = 3 cm and ∠BAC = 60°,
∠q = ∠ABC
(iii) construct the angle bisectors of the three
2
1 angles of the triangle.
= × 60° (b) What do you observe about the angle bisectors?
2
= 30° (c) Draw a circle which touches the three sides of the
∠y = ∠q (alt. ∠s, ST // BA) triangle as shown. Where is the centre of this circle?
= 30°
∴  ∠BTC = ∠x + ∠y
= 60° + 30°
= 90°

183
Solution (b)
(a) (i), (ii) and (iii)

Solution
(b) The angle bisectors of the angles of ABC meet (a) 41° + x° + 63° = 180°  (adj. ∠s on a st. line)
at a point I. x = 76
(c) If we draw a circle with centre I and the
(b) 3x° + 4x° + 2x° + 45° = 360°  (∠s at a point)
perpendicular distance from I to AB as its radius,
9x + 45 = 360
the circle will touch the three sides of ABC.
9x = 315
x = 35
13. Using compasses, design some patterns involving circles
and arcs.
2. Find the values of x and y in each of the following figures.
Solution (a) AOB and COD are straight lines.
Students may come up with various designs using circles
and arcs. Those designs can be used as classroom display.
14. The figure shows three towns A, B and C. Where should
the hospital be built if it is to be equidistant from the
three towns? (b) ABE is a straight line.

Town A

Town B Hospital Solution



(a) = x  (vert. opp. ∠s)
4
y = 4x
Town C

x° + y° = 180° (adj. ∠s on a st. line)
x + 4x = 180
Solution 5x = 180
The hospital should be located at the point of intersection x = 36
of the perpendicular bisectors of ABC, where A, B and y = 144
C are the locations of the towns A, B and C.
(b) x° = y°  (corr. ∠s, AD // BC)
2x + x° = 180 (int. ∠s, AB // DC)
3x = 180
x = 60
Revision Exercise 7 y = 60
1. Find the value of x in each of the following figures.
(a) ABC is a straight line. H
3. In the figure, a squash ball hits a C
vertical wall HK along the path AB.
It is rebounded along the path BC. y°
It is given that ∠ABC = 56°. Find 56° B
the value of y. y°

A
K

Chapter 7  Angles and Parallel Lines


184
Solution 6. Find the angles p and q in the figure.
56° + 2y° = 180° (adj. ∠s on a st. line)
2y = 124
y = 72

4. In the figure, a piece of wire is bent into the shape as


shown. If ∠ABC = 110°, find ∠x and ∠y.
Solution
B A
110°

y
F
E
x
C D

Solution
∠x = 180° – 110° (int. ∠s, AB//DC) Construct CF // AB // ED.
∠x = 70°
∠p = 360° – 270°  (∠s at a point)
∠FED = ∠x (int. ∠s, FE// CD) = 90°
∠y = 360° – ∠x (∠s at a point)
∠y = 42°  (alt. ∠s, AB // CF)
∠y = 360° – 70°
∠x = ∠p – ∠y
∠y = 290° = 90° – 42°
= 48°

∠q = ∠x (alt. ∠s, CF // ED)
5. Find the angle x in each of the following figures.
∠q = 48°
(a)
7. In the figure, ACE, BCF and DCG are straight lines and
AB // HC. Find the angles p, q, r and s.

(b)

Solution
Solution ∠FCH = ∠CBA  (corr. ∠s, AB // HC)
(a) ∠BCD = 75°  (alt. ∠s, AB // CD)   ∠p + 20° = 65°
∠x = ∠BCD (alt. ∠s, BC // DE) ∠p = 45°
∴  ∠x = 75° ∠q = 50°  (alt. ∠s, AB // HC)
∠p + 20° + ∠q + ∠r = 180°  (adj. ∠s on a st. line)
(b) 45° + 20° + 50° + ∠r = 180°
∠r = 65°
∠s = ∠ACG  (vert. opp. ∠s)
= 20° + 50°
= 70°

8. In the figure, ABC is a straight line, ∠ABD = 150°,


Construct CH // BG // DF. ∠CBE = 70°, AE // BD and BE // CD. Find ∠x, ∠y and
∠x1 = 45° (corr. ∠s, CH // DF) ∠z.
∠x2 + 110° = 180° (int. ∠s, CH // BG)
∠x2 = 70°
  ∠x = ∠x1 + ∠x2
= 45° + 70°
= 115°

185
Solution The perpendicular bisectors meet at a point O
∠ABE + ∠CBE = 180°  (adj. ∠s on a st. line) outside ABC.
(150° – ∠x) + 70° = 180° A circle with centre O and radius OA will pass
∠x = 40° through the points B and C.
∠y = ∠x  (alt. ∠s, AE // BD)
  ∠y = 40°
10. In the figure, AOB and COD are straight lines such that
∠z + 70° = 180°  (int. ∠s, BE // CD)
∠BOD = 50°.
∠z = 110°
(a) Using a ruler and compasses, construct
(i) the line XOY such that the ray OY is the angle
9. (a) Using a ruler and compasses, construct bisector of ∠BOD,
(i) ABC with AB = 2 cm, BC = 2.5 cm and (ii) the line TOZ such that the ray OT is the angle
AC = 4 cm, bisector of ∠BOC.
(ii) the perpendicular bisectors of the three sides (b) Is OX the angle bisector of ∠AOC? Why?
of ABC. (c) Find ∠XOT, giving reasons for your answer.
(b) What do you observe about the perpendicular
bisectors?

50°

Solution
Solution (a) (i) and (ii)
(a)


OY is the bisector of ∠BOD.
OT is the bisector of ∠BOC.

(i) Steps for constructing ABC: (b) ∠AOX = ∠BOY  (vert. opp. ∠s)
1. Draw the line segment AB with length ∠COX = ∠DOY  (vert. opp. ∠s)
   2 cm. By construction, ∠BOY = ∠DOY.
2. With A as centre, draw an arc of radius Hence ∠AOX = ∠COX.
4 cm. OX is the angle bisector of ∠AOC.
3. With B as centre, draw an arc of radius
1
2.5 cm to cut the previous arc at C. (c) ∠COX = ∠AOC
2
4. Join A and C. Join B and C. Then ABC 1
is the required triangle. ∠COT = ∠BOC
2
(ii) The perpendicular bisectors of the sides of 1 1
∠COX + ∠COT = ∠AOC + ∠BOC
2 2
ABC are drawn as shown in the diagram.
(b) C
1
∠XOT = (∠AOC + ∠BOC)
2
1
4 = × 180°  (adj. ∠s on a st. line)
2
2.5
∠XOT = 90°

A 2 B

Chapter 7  Angles and Parallel Lines


186
8 Triangles and Polygons

Class Activity 1
Objective: To investigate the properties relating the sides and angles of a triangle.

Tasks
1. With the aid of a ruler and a pair of compasses, construct the following triangles where possible.
(a) ABC with AB = 6 cm, BC = 5 cm and CA = 3 cm
Step 1: Draw and label a line segment AB of length 6 cm.
Step 2: With A as centre and a radius of 3 cm, draw an arc above AB.
Step 3: With B as centre and a radius of 5 cm, draw the second arc to cut the first arc at C.
Step 4: Draw and label the line segments AC and BC to obtain ABC.
C

3 cm 5 cm

A 6 cm B

(b) DEF with DE = 5 cm, EF = 2 cm and FD = 3 cm


Since EF + FD = DE, DEF cannot be constructed.

(c) GHK with GH = HK = 4 cm and KG = 7 cm


Step 1: Draw and label a line segment GH of length 4 cm.
Step 2: With G as centre and a radius of 7 cm, draw an arc above GH.
Step 3: With H as centre and a radius of 4 cm, draw the second arc to cut the first arc at K.
Step 4: Draw and label the line segments GK and HK to obtain GHK.
K

7 cm
4 cm

G 4 cm H

(d) PQR with PQ = 7 cm, QR = 2 cm and RP = 4 cm


Since QR + RP < PQ, PQR cannot be constructed.

(e) XYZ with XY = YZ = ZX = 5 cm


Step 1: Draw and label a line segment XY of length 5 cm.
Step 2: With X as centre and a radius of 5 cm, draw an arc above XY.
Step 3: With Y as centre and a radius of 5 cm, draw the second arc to cut the first arc at Z.
Step 4: Draw and label the line segments XZ and YZ to obtain XYZ.
Z

5 cm 5 cm

X 5 cm Y
Note: only triangles (a), (c) and (e) can be constructed.
187
2. Let us observe some properties of the triangles above in Task 1. Complete the table below. Part of (a) has been done for
you.

(a) AB = 6 cm AB + BC = 11 cm AB + BC > CA
BC = 5 cm BC + CA = 8 cm BC + CA > AB
CA = 3 cm CA + AB = 9 cm CA + AB > BC
(b) DE = 5 cm DE + EF = 7 cm DE + EF > FD
EF = 2 cm EF + FD = 5 cm EF + FD = DE
FD = 3cm FD + DE = 8 cm FD + DE > EF
(c) GH = 4 cm GH + HK = 8 cm GH + HK > KG
HK = 4 cm HK + KG = 8 cm HK + KG > GH
KG = 7 cm KG + GH = 11 cm KG + GH > HK
(d) PQ = 7 cm PQ + QR = 9 cm PQ + QR > RP
QR = 2 cm QR + RP = 6 cm QR + RP < PQ
RP = 4 cm RP + PQ = 11 cm RP + PQ > QR
(e) XY = 5cm XY + YZ = 10 cm XY + YZ > ZX
YZ = 5 cm YZ + ZX = 10 cm YZ + ZX > XY
ZX = 5 cm ZX + XY = 10 cm ZX + XY > YZ

Questions
1. Identify those triangles that cannot be constructed in Task 1. Use your own words, explain why you have difficulty in
consructing them.

DEF and PQR cannot be constructed. The sum of the lengths of two sides of the triangle is smaller than or equal to the length of the third side.

2. Identify those triangles that can be constructed in Task 1. Describe a relationship between the sides of a triangle.

ABC, GHK and XYZ can be constructed.

The sum of the lengths of any two sides of a triangle is greater than the length of the third side.

3. For each triangle that can be constructed in Task 1, locate the longest side and the largest angle. What pattern do you
notice?

Triangle Sides in descending order Angles in descending order


ABC AB, BC, CA ∠C, ∠A, ∠B
GHK HK, KG, GH ∠G, ∠H, ∠K
XYZ XY, YZ, ZX ∠X, ∠Y, ∠Z

In any triangle, the longest side is opposite the biggest angle.

4. What can you say about the angles in an isosceles triangle?

In an isosceles triangles, the angles facing the equal sides are equal.

5. What can you say about the angles in an equilateral triangle?

Each angle in an equilateral triangle is 60°.

Chapter 8  Triangles and Polygons


188
Class Activity 2
Objective: To make a connection between an exterior angle of a triangle and its interior angles.

y A
a

c x
b
B C D

Questions
In the diagram above, ∠x and ∠y are the exterior angles of ABC.

1. What is the sum of ∠a, ∠b and ∠c? Give your reason.

∠a + ∠b + ∠c = 180° (∠ sum of )

2. What is the sum of ∠c and ∠x? Give your reason.

∠c + ∠x = 180° (adj. ∠s on a st. line)

3. Derive the relationship between ∠a, ∠b and ∠x from the two results obtained above. Explain your answer.

∠c + ∠x = 180°
\  ∠c = 180° – ∠x
∠a + ∠b + ∠c = 180°
∠a + ∠b + 180° – ∠x = 180° (Substitute ∠c = 180° – ∠x)
∠a + ∠b – ∠x = 0
\  ∠x = ∠a + ∠b


4. Similarly, what is the sum of ∠a and ∠y? Derive the relationship between ∠b, ∠c and ∠y.

∠a + ∠y = 180°
\  ∠a = 180° – ∠y
∠a + ∠b + ∠c = 180°
180° – ∠y + ∠b + ∠c = 180° (Substitute ∠a = 180° – ∠y)
– ∠y + ∠b + ∠x = 0
\  ∠y = ∠b + ∠c

189
Class Activity 3
Objective: To classify different types of quadrilaterals.

A K
G 2
15 Q P
3 3 30 L
H 13
B D 5 5
20 25
F
3 13
3 30
I J M N
C E 2
25
Fig. 1 Fig. 2 Fig. 3 Fig. 4
T
W
C
3
10 3 V
4
5 2.5
10 Z Y
F
U 5 4
5
2 D O
2
R S W X 1 3
6 5 E
U
Fig. 5 Fig. 6 Fig. 7 Fig. 8

Task
Identify each of the quadrilaterals above, e.g. parallelogram, rectangle, rhombus, square and trapezium.

Name of Quadrilateral Figure


Parallelogram 1, 2, 4, 6, 8
Rectangle 2
Square 1
Rhombus 6
Trapezium 5, 7

Questions
1. Which of the figures above are not parallelograms?

Figures 3, 5 and 7.

2. Excluding those figures which are not parallelograms, what feature do the remaining figures have in common?

They have two pairs of parallel sides.

Chapter 8  Triangles and Polygons


190
Class Activity 4
Objective: To discover the property of a parallelogram.

Tasks

Parallelogram

D C

A B

1. Let us construct a parallelogram ABCD in The Geometer’s Sketchpad as shown above.

2. Measure the sides and angles of the parallelogram.

Note that the opposite sides and opposite angles of a parallelogram are equal.

3. Measure the lengths AE, BE, CE and DE on the diagonals.

Note that AE = CE and BE = DE.

4. Drag a vertex of the parallelogram and observe the variation of the above measurements.

The measurements vary accordingly such that AB = CD, BC = AD, ∠BAD = ∠BCD, ∠ABC = ∠CDA, AE = CE and
BE = DE.

Questions
1. Base on your observation, what can you say about

(a) the sides of a parallelogram,

The opposite sides of a parallelogram are equal.

(b) the angles of a parallelogram,

The opposite angles of a parallelogram are equal.

(c) the diagonals of a parallelogram?

The diagonals of a parallelogram bisect each other.

191
Class Activity 5
Objective: To discover the properties of some special quadrilaterals (rectangle, rhombus, square and trapezium).

Quadrilaterals
D N M
C

A B K L
Rectangle Rhombus
S R
Z Y

P Q W X
Square Trapezium

Tasks
1. Let us draw a rectangle ABCD, a rhombus KLMN, a square PQRS and a trapezium WXYZ in The Geometer’s Sketchpad
as shown above.
2. Construct the diagonals of each quadrilateral.
3. Drag a vertex of each quadrilateral to change its size. What do you observe about their sides, angles and diagonals? In
the following checklist, indicate with a ✓ against the properties that each quadrilateral possesses.

Parallelogram Rectangle Rhombus Square Trapezium
It has at least 1 pair of parallel
✓ ✓ ✓ ✓ ✓
sides

Sides It has 2 pairs of parallel sides ✓ ✓ ✓ ✓


Its opposite sides are equal ✓ ✓ ✓ ✓
Its 4 sides are equal ✓ ✓
It has 4 right angles ✓
Angles
Its opposite angles are equal ✓ ✓ ✓ ✓
Its diagonals are equal ✓ ✓
Its diagonals are perpendicular
✓ ✓
to each other
Diagonals
Its diagonals bisect each other ✓ ✓ ✓ ✓
Its diagonals bisect its interior
✓ ✓
angles

Questions
1. Based on your observation, describe relationship(s) between
(a) a parallelogram and a trapezium,

A trapezium has only 1 pair of parallel sides whereas a parallelogram has 2 pairs of parallel sides.

(b) a rectangle, a rhombus and a square.

A rhombus with equal adjacent sides is a square.

A rhombus whose angles are right angles is a square.

Chapter 8  Triangles and Polygons


192
Class Activity 6
Objective: To derive the formula for the sum of interiors angles of a polygon.

Tasks
Copy and complete the following table.

Triangle Quadrilateral Pentagon Hexagon Heptagon


Polygon … n-gon

Number of
3 4 5 6 7 n
sides
Number of
dissected 1 2 3 4 5 n–2
triangles
Sum of
180° 2 × 180° 3 × 180° 4 × 180° 5 × 180°
interior (n – 2) ×180°
= (3 – 2) × 180° = (4 – 2) × 180° = (5 – 2) × 180° = (6 – 2) × 180° = (7 – 2) × 180°
angles

Questions
1. What patterns can you observe from the task?

The number of dissected triangles is always 2 less than the number of sides. The sum of interior angles is always the number of dissected triangles

multiplied by 180°.

2. If a polygon has 10 sides, i.e. a decagon,

(a) how many triangles does it make up with?

A decagon would have (10 – 2) or 8 triangles.

(b) what is the sum of its interior angles?

The sum of interior angles of a decagon would be (10 – 2) × 180° or 1440°.

3. If a polygon has n sides, what is the sum of its interior angles? Express your answer in terms of n.

For a polygon with n sides, the sum of its interior angles would be (n – 2) × 180°.

193
Class Activity 7
Objective: To derive the formula for the sum of exterior angles of a polygon.

Tasks

Sum of Exterior Angles of a Polygon


Y
S
G
X
C T
D
Z R
H
P
A Q W
B F V

1. Let us construct a quadrilateral ABCD and a pentagon PQRST in The Geometer’s Sketchpad as shown above.

2. Measure each exterior angle of the quadrilateral ABCD and calculate their sum. What do you observe?

The sum of the exterior angles of a quadrilateral is 360°.

3. Measure each exterior angle of the pentagon PQRST and calculate their sum. What do you observe?

The sum of the exterior angles of a pentagon is 360°.

4. Drag the vertices of ABCD and PQRST around. Does the sum of the exterior angles of each diagram change?

No, the sum of the exterior angles of each polygon remains the same.

5. Draw some other polygons and find the sum of exterior angles of these polygons.

The sum of the exterior angles of a polygon is 360°.

Questions
1. What do you notice about the sum of exterior angles of a polygon?

The sum of exterior angles of a polygon is always 360°.

2. Derive a formula to obtain the exterior angle of a regular n-sided polygon.


360°
1 exterior angle of a regular n-sided polygon = .
n

3. Does a regular polygon with an exterior angle of 75° exist? Explain your answer.
360°
Using the formula derived in (2), we have 75° = .
n
This gives us n = 4.8. Such a polygon does not exist as 4.8 is not a whole number.

Chapter 8  Triangles and Polygons


194
Class Activity 8
Objective: To recognise the line symmetry in some special quadrilaterals and regular polygons.

Tasks
1. Take a look at your surroundings. Take photos of some special quadrilaterals and regular polygons
from objects, such as floor tiles, company logos, buildings and bridges.
2. Highlight the shapes and discuss whether they have any lines of symmetry.
3. Draw the lines of symmetry on the shapes if they exist.
4. The diagram on the right is a regular octagon. Write down as many mathematical statements or
observations as you can. Hint: You may use a cut-out to help you from your ideas, e.g. by folding,
by drawing lines, etc.

A regular octagon has 8 lines of symmetry as shown below.

Class Activity 9
Objective: To recognise the rotational symmetry in some special quadrilaterals and regular polygons.

Tasks
1. Let us look at the regular hexagon shown on the right.
(a) If the hexagon is rotated about its centre O, could you find an angle of rotation less
than 360° such that the hexagon has the same orientation after being rotated once?
If so, write down the angle of rotation. O

Angle of rotation = 60°.

(b) Would you think the regular hexagon has rotational symmetry? Explain your answer.

Yes, the regular hexagon has rotational symmetry of order 6. The hexagon coincides with itself six times as it rotates
through 360°.

195
2. (a) Let us number the vertices of the star shown on the right. 1

5 2

4 3

(b) Identify the centre of the star.


1

5 2
O

4 3


(c) Rotate the star about its centre. Determine whether the star has rotational symmetry. Explain your answer.

Yes, the star has rotational symmetry of order 5 with angle of rotation of 72°. The star coincides with itself five
times as it rotates through 360°.

3. (a) Take a look at your surroundings. Take photos of some special quadrilaterals and regular polygons from objects,
such as floor tiles, company logos, buildings and bridges.

Answer varies.

(b) Highlight the shapes and discuss whether they have rotational symmetry.

Answer varies.

Chapter 8  Triangles and Polygons


196
Try It! Solution
∠EBD = ∠BAE + ∠AEB (ext. ∠ of )
Section 8.1
= 36° + 31°
1. In the figure, AB = AC and DA // BC. = 67°
Find the angles x, y and z. ∠DCF = ∠EBC (corr. ∠s, BE // CF)
= 67°

4. In the diagram, ABC, CDE, AFD and BFE are straight


lines. Find ∠CAD.

Solution
∠z = ∠EAD  (corr. ∠s, DA // BC)
= 55°
∠y = ∠z  (base ∠s of isos. )
= 55°
∠x = 180° – ∠y – ∠z (∠ sum of )
= 180° – 55° – 55°
= 70°
Solution
In EFD,
2. In the diagram, BCD is a straight line, ∠FDC = ∠DFE + ∠DEF (ext. ∠ of )
ABC is equilateral, CE = DE and = 61° + 39°
∠CED = 90°. Find the angles x and y. = 100°
In ACD,
∠CAD = 180° – ∠ACD – ∠FDC (∠ sum of )
= 180° – 52° – 100°
= 28°

5. Find angle x in the figure.


Solution
∠CDE = ∠x (base ∠s of isos. )
∠x + ∠CDE + ∠CED = 180°  (∠ sum of )
∠x + ∠x + 90° = 180°
2∠x = 90°
∠x = 45° x
∠ACB = 60°  (∠ of equilateral )
∠y = 180° – ∠x – ∠ACB (adj. ∠s on a st. line)
= 180° – 45° – 60°
= 75°
Solution

3. In the figure, ABCD is a straight line, BE // CF. Find
∠DCF.

Produce CD to meet AB at E.
∠y = 55° + 36°  (ext. ∠ of )
= 91°
∠ADC = ∠y + ∠DAE (ext. ∠ of )
= 91° + 30°
= 121°

197
Section 8.2 9. In the figure, PQRS is a rhombus.
6. In the figure, PQRS is a parallelogram, PS = 3x cm, Find the angles x and y.
QR = 21 cm, QT = (5y – 10) cm, ST = (3y + 2) cm. Find
the values of x and y.

Solution
∠SQR = ∠SQP  (diagonals of rhombus)
= 65°
Solution ∠y = ∠SQR  (base ∠s of isos. )
PS = QR  (opp. sides of //gram) = 65°
3x = 21 ∠x = 180° – ∠y – ∠SQR (∠ sum of )
x = 7 = 180° – 65° – 65°
ST = QT  (diagonals of //gram) = 50°
3y + 2 = 5y – 10
2y = 12 10. In the figure, ABCD is a square and ABE is equilateral.
y = 6 Find ∠AED.

7. In the figure, PQRS is a parallelogram.


Find the angles x and y.

Solution
∠EAB = 60° (∠ of equilateral )
∠DAB = 90°  (∠ of square)
∴  ∠DAE = ∠DAB – ∠EAB
Solution
= 90° – 60°
∠x = ∠PSR (opp. ∠s of //gram) = 30°
= 110° ∠ADE = ∠x (base ∠of isos. )
40° + ∠x + ∠y = 180°  (∠ sum of ) ∠ADE + ∠x + ∠DAE = 180°  (∠ sum of )
40° + 110° + ∠y = 180° ∠x + ∠x + 30° = 180°
∠y = 30° 2∠x = 150°
∠x = 75°
8. In the figure, PQRS is a rectangle. ∠AED = 75°
Find the angles x and y.

Section 8.3
11. Find the angle y in the figure.

Solution
∠x + 61° = 90°  (∠ of rectangle)
∠x = 29°
PT = QT  (diagonals of rectangle)
∠TPQ = ∠x  (base ∠s of isos. ) Solution
= 29° ∠y + 68° + 75° + 90° = 360°  (∠ sum of polygon)
∠y = ∠TPQ + ∠x (ext. ∠ of ) ∠y = 127°
= 29° + 29°
= 58°

Chapter 8  Triangles and Polygons


198
12. In the figure, ABCDEF is a regular hexagon. Find the 15. Find the size of each interior angle of a regular octagon.
angles x, y and z.
Solution
Each exterior angle
= 360° ÷ 8  (ext. ∠ sum of polygon)
= 45°
Each interior angle
= 180° – 45° (adj. ∠s on a st. line)
= 135°
Solution
Angle sum of hexagon 16. If each interior angle of an n-sided polygon is 156°, find
= (6 – 2) × 180°  (∠ sum of polygon) the value of n.
= 720°
∠x = 720° ÷ 6 Solution
= 120° Each exterior angle
∠ABF = ∠AFB  (base ∠s of isos. ) = 180° – 156°  (adj. ∠s on a st. line)
∠ABF + ∠AFB + ∠x = 180°  (∠ sum of ) = 24°

∠ABF + ∠ABF + 120° = 180° 24 × n = 360   (ext. ∠ sum of polygon)
∠ABF = 30° n = 15
1
∠y = ∠BCD
2

= × 120°
1 Section 8.4
2
17. (a)
Construct PQR in which PQ = 5 cm, QR = 4 cm
= 60° and ∠PRQ = 90° cm.
∠CBF = ∠ABC – ∠ABF (a) Is PQR an isosceles triangle.
= 120° – 30°
= 90° Solution
∠y + ∠z + ∠CBF = 180°  (∠ sum of ) Y
60° + ∠z + 90° = 180°
∠z = 30°

13. The angle sum of the interior angles of an n-sided polygon P


is 1620°. Find the value of n.
5 cm
Solution
(n – 2) × 180° = 1620°  (∠ sum of polygon)
1620 Q 4 cm R
n – 2 =
180
n – 2 = 9 Construction Steps:
n = 11 1. Draw a line segment PQ 5 cm long.
2. With P as centre and 3 cm as radius, draw an arc
above PQ.
14. Find the angles x and y in the figure. 3. With Q as centre and 4 cm as radius, draw an arc to
cut the first arc at R.
4. Draw line segments PR and QR. PQR is the required
triangle.

18. Construct LMN with LM = 4 cm, MN = 4 cm and


∠LMN = 30°.

Solution
Solution
∠x = 180° – 44°  (adj. ∠s on a st. line)
= 136°
∠x + ∠y + 111° = 360°  (ext. ∠ sum of polygon)
136° + ∠y + 111° = 360°
∠y = 113°

199
Construction Steps: 20. Construct a trapezium PQRS in which PQ// SR,
1. Construct a line segment MN 4 cm long. PQ = 7 cm, SR = 3 cm and ∠SPQ = ∠PQR = 50°.
2. Draw a ray with the end point M and making an angle
of 30° with MN using a protractor. Solution
3. With M as centre and 4 cm as radius, draw an arc to
cut the ray at L. A B
4. Join L and N. LMN is the required triangle.
R
S
19. Construct a quadrilateral XYZT in which XY = 4 cm,
YZ = 5 cm, TX = 4 cm, ∠TXY = 130° and ∠XYZ = 90°.
50° 50°
Solution
P 4 cm E 3 cm Q

Construction Steps:
1. Draw a line segment PQ 7 cm long and mark a point
E on it where PE = 4 cm.
2. Draw lines PB and EA such that ∠EPB = 50° and
∠PEA = 50° .
3. Let S be the point of intersection of PB and EA. Draw
the line segment SR parallel to PQ and of length 3
cm.
Construction Steps:
4. Draw the side RQ. Then PQRS is the required
1. Draw a line segment XY 4 cm long.
trapezium.
2. Using a protractor, draw a ray through X and making
an angle of 130° with XY.
3. With X as centre and 4 cm as radius, cut the ray at
T.
4. Using a protractor, draw a ray through Y and making
an angle of 90° with XY.
5. With Y as centre and 5 cm as radius, cut the ray at
Z.
6. Draw side TZ. XYZT is the required quadrilateral.

Chapter 8  Triangles and Polygons


200
Exercise 8.1 to obtain GHK.
Level 1 (d) N

1. Draw each of the following triangles if possible and state


the type of the triangle according to its sides. If it is not
4 cm 4 cm
possible to draw the triangle, explain briefly why it is
not possible to do so.
(a) ABC with AB = 7 cm, BC = 4 cm and
L 4 cm M
CA = 5 cm
(b) DEF with DE = 6 cm, EF = 3 cm and
Construction Steps:
FD = 2 cm
1. Draw and label a line segment LM of length
(c) GHK with GH = 4 cm, HK = 3 cm and
4 cm.
KG = 4 cm
2. With L as centre and a radius of 4 cm, draw an
(d) LMN with LM = 4 cm, MN = 4 cm and
arc above LM.
NL = 4 cm
3. With M as centre and a radius of 4 cm, draw
the second arc to cut the
Solution
first arc at N.
(a)
4. Draw and label the line segments NL and NM
C
to obtain LMN.

5 cm 4 cm 2. State the biggest angle in each of the following triangles.


(a) ABC with AB = 10 cm, BC = 13 cm and
CA = 9 cm
A 7 cm B (b) PQR with PQ = 15 cm, QR = 15 cm and
RP = 21 cm
Construction Steps:
1. Draw and label a line segment AB of length Solution
7 cm. (a) ∠BAC = 86.2°
2. With A as centre and a radius of 5 cm, draw an (b) ∠PQR = 88.9°
arc above AB.
3. With B as centre and a radius of 4 cm, draw 3. (a) Draw an isosceles right-angled triangle ABC with
the second arc to cut the first arc at C. AC = BC = 3 cm and ∠ACB = 90°.
4. Draw and label the line segments AC and BC (b) Find ∠ABC.
to obtain ABC.
Solution
(b) Since EF + FD < DE, it is not possible to draw (a)
DEF. B

(c)
K

3 cm
4 cm 3 cm

G 4 cm H A 3 cm C

Construction Steps: (b) ∠ABC = 45°


1. Draw and label a line segment GH of length
4 cm. 4. Find the unknown angle x in each of the following
2. With G as centre and a radius of 4 cm, draw triangles.
an arc above GH. (a) (b)
3. With H as centre and a radius of 3 cm, draw
the second arc to cut the first arc at K.
4. Draw and label the line segments GK and HK

201
(c) (d) (c) ∠y + 50° = 80°  (ext. ∠ of )
∠y = 30°
(d) ∠y + 90° = 150° (ext. ∠ of )
∠y = 60°

6. Find the unknown angles x and y in each of the following



figures.
Solution (a) (b)
(a) ∠x + 65° + 38° = 180°  (∠ sum of )
∠x = 77°
(b) ∠x + 35° + 32° = 180°  (∠ sum of )
∠x = 113°
(c) ∠LNM = 63°  (base ∠ of isos. )
∠x + 63° + ∠LNM = 180°  (∠ sum of )
∠x + 63° + 63° = 180° (c) (d)
∠x = 54°
(d) ∠PRQ = ∠x (base ∠ of isos. )
54° + ∠x + ∠PRQ = 180°  (∠ sum of )
54° + ∠x + ∠x = 180°
2∠x = 126°
∠x = 63°

5. Find the unknown angle y in each of the following figures.
Solution
(a) (a) ∠x = 180° – 136°  (adj. ∠s on a st. line)
= 44°
∠x + ∠y = 112°  (ext. ∠ of )
44° + ∠y = 112°
∠y = 68°
(b) ∠x = 180° – 60°  (adj. ∠s on a st. line)
= 120°
(b) ∠x + ∠y = 150°  (ext. ∠ of )
120° + ∠y = 150°
∠y = 30°
(c) ∠x = 38°  (base ∠s of isos. )
∠y = 38° + ∠x (ext. ∠ of )
= 38° + 38°
= 76°
(d) ∠RTS = ∠x   (base ∠s of isos. )
∠RTS + ∠x + 90° = 180°  (∠ sum of )
(c) ∠x + ∠x + 90° = 180°
2∠x = 90°
∠x = 45°
∠y = ∠x + 90°  (ext. ∠ of )
= 45° + 90°
= 135°
(d)

Level 2
7. Find the unknown angle x in each of the following figures.
(a) (b)

Solution
(a) ∠y = 52° + 80°  (ext. ∠ of )
= 132°
(b) ∠y = 114° + 22° (ext. ∠ of )
= 136°

Chapter 8  Triangles and Polygons


202
(c) (d) (c) (d)

Solution Solution
(a) In CDE, (a) ∠x = 45°  (corr. ∠s, BD // CE)
∠BCD = 35° + 39°  (ext. ∠ of ) ∠x + ∠y + 108° = 180°  (∠ sum of )
= 74° 45° + ∠y + 108° = 180°
In ABC, ∠y = 27°
∠x + 48° = ∠BCD (ext. ∠ of )
∠x + 48° = 74° (b) ∠x + 123° = 180°  (int. ∠s, BA // FD)
∠x = 26° ∠x = 57°
∠x + ∠y = 110°  (ext. ∠ of )
(b) In HLG, 57° + ∠y = 110°
∠FGM = 40° + 35°  (ext. ∠ of ) ∠y = 53°
= 75°
In FGM, (c) ∠y = 96°  (alt. ∠s, QP // SR)
∠x + ∠FGM = 105°  (ext. ∠ of ) ∠x + ∠y = 145°  (ext. ∠ of )
∠x + 75° = 105° ∠x + 96° = 145°
∠x = 30° ∠x = 49°
(c) In PQS, (d) (40° + ∠ZXY) + (28° + 43°)
56° + 90° + ∠QSP = 180°  (∠ sum of ) = 180°  (int. ∠s, WX // ZY)
∠QSP = 34° ∠ZXY = 69°
In PRS, ∠x = 43° + ∠ZXY (ext. ∠ of )
∠x + 90° + ∠PSR = 180°  (∠ sum of ) = 43° + 69°
∠PSR = ∠QSP = 112°
∠x + 90° + 34° = 180° ∠y = ∠WXY (corr. ∠s, WZ // XY)
∠x = 56° = 40° + 69°
= 109°
(d)

Level 3
9. The figure shows a section of a roof in which AB = AC
and ∠ABC = 25°. Find ∠BAC.
Produce TW to meet UV at Y.
In UTY,
∠WYV = 30° + 100°  (ext. ∠ of )
= 130°
In WYV, Solution
∠WYV + ∠x = 145°  (ext. ∠ of ) ∠BAC = 180° – 25° – 25° (base ∠s of isos. )
130° + ∠x = 145° ∠BAC = 130°
∠x = 15°

10. The figure shows a rectangular flag. AC and BD intersect


8. Find the unknown angles x and y in each of the following at E such that AE = BE = CE = DE. If ∠AED = 64°, find
figures. the angles x, y and z.
(a) (b)

203
Solution (ii)
x = 180° – 64° (sum of adj. ∠s on a st. line)
x = 116°
64°
y = (ext. ∠ of , base ∠s of isos. )
2
y = 32° When ∠AMC = 36°,
z =
116°
(ext. ∠ of , base ∠s of isos. ) ∠p = ∠q (base ∠s of isos. )
2 ∠p + ∠q + 36° = 180°  (∠ sum of )
z = 58° ∠q + ∠q + 36° = 180°
∠q = 72°
11. In the figure, AC and BC are two equal legs ∠r = ∠s (base ∠s of isos. )
of a pair of compasses. If ∠ACB = x° and ∠r + ∠s = 36°  (ext. ∠ of )
∠CAB = 2x°, find the value of x. ∠r + ∠r = 36°
∠r = 18°
  ∠ACB = ∠q + ∠r
= 72° + 18°
= 90°
(iii)
Solution
∠ABC = 2x° (base ∠s of isos. )
∴  x° + 2x° + 2x° = 180°  (∠ sum of )
5x = 180
x = 36

When ∠BMC = 100°, along the same line of


12. The figure shows a simple device in which AMB reasoning in (i) and (ii),
is a straight rod and CM is a rod pivoted at M with 1
AM = BM = CM. ∠d = × 100°
2
(a) Find ∠ACB in each of the following cases. = 50°
(i) ∠CAM = 60°
1
(ii) ∠AMC = 36° ∠c = × (180° – 100°)
2
(iii) ∠BMC = 100°
(b) What can you generalise from the results in (a)? = 40°
∴  ∠ACB = ∠c + ∠d
= 40° + 50°
= 90°
(b) ∠ACB is always 90°.

Solution 13. In the figure, BA // CD, BE is the angle bisector of ∠ABC,


(a) (i) CE is the angle bisector of ∠BCD.

When ∠CAM = 60°,


∠x = 60°  (base ∠s of isos. )
∠y + ∠x + 60° = 180°  (∠ sum of )
∠y + 60° + 60° = 180°
∠y = 60° (a) Construct the figure using Sketchpad.
∠t = ∠z (base ∠s of isos. ) (b) Measure ∠BEC and write down the result.
∠t + ∠z = ∠y (ext. ∠ of ) (c) Drag the points B and C around. What do you
∠t + ∠t = 60° observe?
∠t = 30° (d) Derive the size of ∠BEC by reasoning.
  ∴   ∠ACB = ∠x + ∠t
= 60° + 30°
= 90°

Chapter 8  Triangles and Polygons


204
Solution (d)
(a) Construction Steps:
1. Draw a horizontal line AB.
2. Mark a point C above the line AB and construct
a line CD parallel to AB.
3. Join B and C.
4. Use the angle bisector command to construct
the angle bisectors BE and CE of ∠ABC and
∠BCD respectively. Let the angles x and y be as shown in the above
diagram.
(b) ∠BEC = 90°
2∠x + 2∠y + 65° = 180°  (∠ sum of )
(c) ∠BEC is always 90°. 2(∠x + ∠y) = 115°
(d) ∠x + ∠y = 57.5°
∠BIC + ∠x + ∠y = 180°  (∠ sum of )
∠BIC + 57.5° = 180°
∴  ∠BIC = 122.5°

Exercise 8.2
Level 1
Since BE and CE are the angle bisectors of ∠ABC
1. Find the unknown values or angles x and y in each of
and ∠BCD respectively, we can let the angles be
the following figures where ABCD is a parallelogram.
x and y as shown in the above diagram.
∠x + ∠x + ∠y + ∠y = 180°  (int. ∠s, BA // CD) (a) (b)
2∠x + 2∠y = 180°
∴  ∠x + ∠y = 90°
∠x + ∠y + ∠BEC = 180°  (∠ sum of )
∴  90° + ∠BEC = 180°
∠BEC = 90°

14. In ABC, ∠BAC = 65°, BI and CI are the angle bisectors (c) (d)
of ∠ABC and ∠ACB respectively.
(a) Construct ABC using Sketchpad.
(b) Measure ∠BIC and write down your answer.
(c) Drag the point B or the point C to vary its position
while keeping ∠BAC = 65°. What do you observe?
Solution
(d) Derive the size of ∠BIC by reasoning.
(a) DC = AB (opp. sides of //gram)
2x = 10
x = 5
∠y = 72° (opp. ∠s of //gram)
(b) 3x = 15 (diagonals of //gram)
x = 5
4y = 12 (diagonals of //gram)
y = 3
Solution
(a) Construction Steps: (c) ∠x = 54°  (opp. ∠s of //gram)
1. Draw a line segment AB. ∠x + ∠y + 59° = 180° (∠ sum of )
2. Rotate AB by 65° to another line segment and 54° + ∠y + 59° = 180°
mark a point C on the line segment. ∠y = 67°
3. Join B and C to form ABC. (d) ∠x = 126°  (opp. ∠s of //gram)
4. Use the angle bisector command to draw the ∠y + 90° = ∠x  (ext. ∠ of )
angle bisectors BI and CI of the angles ∠ABC ∠y + 90° = 126°
and ∠ACB respectively. ∠y = 36°
(b) ∠BIC = 122.5°
(c) ∠BIC is always 122.5°

205
2. Find the unknown values or angles x and y in each of (b) ∠x = 45° (diagonals of square)
the following rectangle EFGH. ∠UVP = ∠y  (vert. opp. ∠s)
(a) (b) ∠x + ∠UVP = 125° (ext. ∠ of )
45° + ∠y = 125°
∠y = 80°

Level 2
5. In the figure, ABCD is a
Solution parallelogram and
(a) ∠x + 36° = 90°  (∠ of rectangle) ∠DAB = 73°.
∠x = 54° Find the angles x and y.
∠y = 36°  (alt. ∠s, EF // HG)
(b) x = 18  (diagonals of rectangle)
3y = 18  (diagonals of rectangle) Solution
y = 6 ∠x + 38° = 73°  (opp. ∠s of //gram)
∠x = 35°
3. Find the unknown values or angles x and y in each ∠y + 73° = 180°  (int. ∠s, AD // BC)
rhombus KLMN. ∠y = 180° – 73°
(a) (b) ∠x = ∠KNM = 107°

6. In the figure, PQRS is a parallelogram and PQ = PR.


Find the angle x.

Solution
(a) 3x = 24  (adj. sides of rhombus)
x = 8
Similarly, Solution
2y – 6 = 24 ∠PRQ = ∠PQR (base ∠s of isos. )
2y = 30 ∠PRQ + ∠PQR + 40° = 180°  (∠ sum of )
y = 15 ∠PQR + ∠PQR = 140°
∠PQR = 70°
(b) ∠MLN = 23°  (diagonals of rhombus)
∠x = ∠PQR (opp. ∠s of //gram)
∠x = ∠KLM  (opp. ∠s of // gram)
= 70°
∴  ∠x = 23° + 23°
= 46°
∠x + ∠y = 180°  (int. ∠s, LM // KN) 7. In the figure, ABCD is a rhombus. AC = 14 cm and ABE
46° + ∠y = 180° is a straight line.
∠y = 134°

4. Find the unknown values or angles x and y in each square


PQRS.
(a) (b)
(a) Find the length of AF.
(b) Find the angles x, y and z.

Solution
1
(a) AF = AC  (diagonals of rhombus)
2
Solution 1
= × 14 cm
(a) ∠x = 90°  (diagonals of square) 2
3y + 2 = 17  (adj. sides of square) = 7 cm
y = 5

Chapter 8  Triangles and Polygons


206
(b) ∠x = 90°  (diagonals of rhombus) (c) ∠EBF = 45°  (diagonals of square)
∠BDC = 180° – 90° – 20°  (∠ sum of ) ∠BFE = ∠x (base ∠s of isos. )
= 70° ∠x + ∠BFE + ∠EBF = 180°  (∠ sum of )
∠y = ∠BDC (alt. ∠s, AB // DC) ∠x + ∠x + 45° = 180°
= 70° 2∠x = 135°
∠DBC = ∠y  (diagonals of rhombus) ∠x = 67.5°
= 70° ∠x + ∠y = 90° (diagonals of square)
∠z + ∠y + ∠DBC = 180°  (adj. ∠s on a st. line) 67.5° + ∠y = 90°
∠z + 70° + 70° = 180° ∠y = 22.5°
∠z = 40°
10. In the diagram, ABCD is a parallelogram, PAQ // RCS,
8. In the diagram, PQRS is a rectangle. Given that PT = 8 cm and AB = (3t + 1) cm and CD = (5t – 7) cm.
∠QTR = 62°, find

(a) the length of QS,
(b) the angles x and y.

Solution
(a) PR = 2PT  (diagonals of rectangle)
= 2 × 8 cm
Find
= 16 cm
(a) the value of t,
QS = PR  (diagonals of rectangle)
(b) the length of CD,
= 16 cm
(c) the angles x and y.
(b) ∠TRQ = ∠x (base ∠s of isos. )
62° + ∠TRQ + ∠x = 180°  (∠ sum of )
62° + ∠x + ∠x = 180° Solution
2∠x = 118° (a) AB = CD  (opp. sides of //gram)
∠x = 59° 3t + 1 = 5t – 7
∠x + ∠y = 90°  (∠ of rectangle) 2t = 8
59° + ∠y = 90° t = 4
∠y = 31°
(b) CD = (5 × 4 – 7) cm
9. In the figure, ABCD is a square with BE = BF, = 13 cm
AB = (7t – 6) cm and AD = (2t + 9) cm.
(c)

Construct EB // PAQ // RCS.
Find
∠b1 = 38°  (alt. ∠s, EB // CS)
(a) the value of t,
∠b2 = 41°  (alt. ∠s, EB // AQ)
(b) the length of AB,
∠x = ∠b1 + ∠b2 (opp. ∠s of //gram)
(c) the angles x and y.
= 38° + 41°
= 79°
Solution
∠DAB + ∠x = 180° (int. ∠s, AB //  DC)
(a) AB = AD  (adj. sides of square)
∠DAB + 79° = 180°
7t – 6 = 2t + 9
∠DAB = 101°
5t = 15
∠y + ∠DAB + 41° = 180°  (adj. ∠s on a st. line)
t = 3
∠y + 101° + 41° = 180°
(b) AB = (7 × 3 – 6) cm ∠y = 38°
= 15 cm

207
Level 3 (d) ∠SQR = 45°  (diagonals of square)
11. In the figure, 4 pieces of identical 30°– 60°– 90° set ∠CQR = 60°  (given)
squares are placed together to form a quadrilateral ABCD ∴  ∠CQS = 45° + 60°
with AC = 18 cm. = 105°

13. The figure shows a logo in which ABCD and BCEF
are parallelograms, BF and CD intersect at G.
AD = (2x + 1) cm, FE = (3x – 5) cm, ∠BAD = 80° and
∠CEF = 32°. Find
(a) the value of x,
(b) the angle y.
(a) What type of quadrilateral is ABCD?
(b) Find the length of AE.
(c) Find ∠BAD.
(d) What type of triangle is ABD?
Solution
Solution (a) Since AD = BC and BC = FE,
(a) Since AB = BC = CD = DA, AD = BC
ABCD is a rhombus. 2x + 1 = 3x – 5
3x – 2x = 1 + 5
1
(b) AE = AC  (diagonals of rhombus) x=6
2
1 (b) ∠BCD = 180° (opp. ∠s of //gram)
= × 18 cm
2 ∠FBC = 32° (opp. ∠s of //gram)
= 9 cm y = ∠BCD + ∠FBC (ext. ∠ of )
y = 80° + 32°
(c) ∠BAD = 30° + 30° y = 112°
= 60°
(d) ∠ABD = ∠BAD = ∠ADB = 60° 14. In the figure, a square tile ABCF is joined with a rhombus
∴  ABD is an equilateral triangle. tile CDEF. AB = 6 cm and ∠DEF = 140°. Find
(a) the lengths of DE and CD,
12. In the figure, 4 pieces of identical 30°– 60°– 90° set (b) ∠BFD and ∠FBD.
squares are arranged to form a quadrilateral ABCD.

Solution
(a) AB = FC (sides of square)
(a) What type of quadrilateral is ABCD? FC = DE = CD (sides of rhombus)
(b) Find ∠SPQ. DE = CD = 6 cm
(c) What type of quadrilateral is PQRS? (b) ∠BFD = ∠BFD + ∠CFD
(d) Find ∠CQS. 180° – 90° 180° – 140°
∠BFD = +
2 2
Solution (base ∠s of isos. )
(a) Since AB = BC = CD = DA ∠BFD = 45° + 20°
and ∠BAD = 90°, ABCD is a square. ∠BFD = 65°
(b) ∠SPQ
= 180° – ∠APS – ∠QPB (adj. ∠s on a st. line) 15. (a) Construct a quadrilateral ABCD using Sketchpad.
= 180° – 30° – 60° Let P, Q, R and S be the midpoints of the sides
= 90° AB, BC, CD and DA respectively.
(c) Since PQ = QR = RS = SP and ∠SPQ = 90°, (b) Join the points P, Q, R and S to form a quadrilateral.
PQRS is a square. What type of quadrilateral is PQRS?

Chapter 8  Triangles and Polygons


208
(c) If ABCD is a rhombus, what type of quadrilateral (c) ∠ sum of the heptagon
is PQRS? = (7 – 2) × 180°  (∠ sum of polygon)
(d) Compare the areas of ABCD and PQRS by dragging = 900°
any point. How are the areas related? ∴  ∠x = 900° – 130° – 120° – 162°
– 140° – 96° – 115°
Solution = 137°
(a)
(d) 50° + ∠x + 100° + ∠x
= 360°  (∠ sum of polygon)
2∠x = 210°
 ∠x = 105°

2. Find the sum of the interior angles of the following


polygons.
(a) Hexagon (b) Octagon
Use the line segment command to create the (c) Dodecagon (12-gon) (d) 20-gon
quadrilateral ABCD. Then use the midpoint
command to create the points P, Q, R and S. Solution
(b) PQRS is a parallelogram. (a) Sum of interior angles of a hexagon
(c) = (6 – 2) × 180°
= 720°
(b) Sum of the interior angles of an octagon
= (8 – 2) × 180°
= 1080°
(c) Sum of the interior angles of a 12-gon
= (12 – 2) × 180°
If ABCD is a rhombus, PQRS is a rectangle. = 1800°
1 (d) Sum of the interior angles of a 20-gon
(d) Area of PQRS = × Area of ABCD.
2 = (20 – 2) × 180°
= 3240°

Exercise 8.3
3. Find the unknown angle x in each of the following figures.
Level 1
(a) (b)
1. Find the unknown angle x in each of the following figures.
(a) (b)


(c) (d)
(c) (d)

Solution Solution
(a) ∠x + 130° + 61° + 65° (a) ∠x + 101° + 112°
= 360°  (∠ sum of polygon) = 360°  (ext. ∠ sum of polygon)
∠x = 104° ∠x = 147°
(b) ∠ sum of the pentagon (b) ∠x + 90° + 113° + 72°
= (5 – 2) × 180°  (∠ sum of polygon) = 360°  (ext. ∠ sum of polygon)
= 540° ∠x = 85°
∴  ∠x = 540° – 125° – 79° – 112° – 116°
= 108°

209
(c) Exterior angle at vertex S 6. Find the unknown angles x and y in each of the following
= 180° – 131°  (adj. ∠s on a st. line) figures.
= 49° (a) (b)
∠x + 49° + 75° + 77° + 60°
= 360°  (ext. ∠ sum of polygon)
∠x = 99°
(d) ∠x + ∠x + ∠x + 75° + 50° + 67°
= 360°  (ext. ∠ sum of polygon)
3∠x = 168°
 ∠x = 56°

4. Find the size of each exterior angle of a regular polygon Solution


given its number of sides. (a) ∠x + 68° = 180°  (int. ∠s, AE // BC)  

(a) 5 sides (b) 8 sides ∠x = 112°


(c) 18 sides (d) 24 sides ∠x + 115° + ∠y + 140° + 68°
= (5 – 2) × 180°  (∠ sum of polygon)
Solution 112° + 115° + ∠y + 140° + 68° = 540°
(a) Each exterior angle of a 5-gon ∠y = 105°
360°
=  (ext. ∠ sum of polygon) (b) ∠BCD = 102°  (corr. ∠s, AB // DC)
5  
∠BCD + ∠x = 180°  (adj. ∠s on a st. line)
= 72°
102° + ∠x = 180°
360° ∠x = 78°
(b) Each exterior angle of an 8-gon = ∠x + ∠y + ∠y + ∠y + y + 102°
8
= 45° = 360°  (ext. ∠ sum of polygon)
360° 78° + 4∠y + 102° = 360°
(c) Each exterior angle of an 18-gon = 4∠y = 180°
18
∠y = 45°
= 20°
360°
(d) Each exterior angle of a 24-gon =
24 7. If each exterior angle of a regular polygon is 12°, find
= 15° the number of sides of the polygon.

Solution
Level 2 Let n be the number of sides of the polygon.
Then 12° × n = 360°  (ext. ∠ sum of polygon)
5. Find the value of x in each of the following figures.
n = 30
(a) (b) The number of sides of the polygon is 30.

8. If each interior angle of a regular polygon is 140°, find


the number of sides of the polygon.

Solution
Let n be the number of sides of the polygon.
Solution Each exterior angle
(a) 2x° + x° + 71° + 85° = 180° – 140°  (adj. ∠s on a st. line)
= 360°  (∠ sum of polygon) = 40°
3x + 156 = 360 ∴ 40° × n = 360°  (ext. ∠ sum of polygon)
3x = 204 n = 9
x = 68 The number of sides of the polygon is 9.

(b) x° + x° + x° + 90° + 90°


= (5 – 2) × 180°  (∠ sum of polygon)
3x + 180 = 540
3x = 360
x = 120

Chapter 8  Triangles and Polygons


210
9. The sum of the interior angles of a polygon is 1980°. (b) ∠ACB = ∠CAB (base ∠s of isos. )
Find the number of sides of the polygon. By ∠ sum of , we have
1
Solution ∠ACB = ∠CAB = (180° – ∠ABC)
2
Let n be the number of sides of the polygon. 1
(n – 2) × 180° = 1980° = (180° – 120°)
2
n – 2 = 11 = 30°
n = 13 1
The number of sides of the polygon is 13. Similarly, ∠CBG = ∠BGC = (180° – 150°)
2
= 15°
10. In the figure, ABCDE is a regular pentagon. The sides
1
∠BAG = ∠BGA = (180° – 90°)
AB and DC are produced to meet at F. Find the angles 2
x, y and z. = 45°
  ∠CAG = ∠CAB + ∠BAG = 30° + 45°
= 75°
Similarly, ∠ACG = 30° + 15°
= 45°
and ∠AGC = 45° + 15°
= 60°

Solution
Sum of the interior angles of a pentagon Level 3
= (5 – 2) × 180°  (∠ sum of polygon)
12. The figure represents a coin that is
= 540°
in the shape of a regular decagon.
  ∠x = 540° ÷ 5
O is the centre of the coin.
= 108°
Find the size of
∠x + ∠CBF = 180°  (adj. ∠s on a st. line)
(a) an interior angle of the coin.
108° + ∠CBF = 180°
(b) each angles of OAB.
∠CBF = 72°
Similarly, ∠y = 72° Solution
∠y + ∠z = ∠x (ext. ∠ of ) (a) Sum of the interior angles of a decagon
72° + ∠z = 108° = (10 – 2) × 180°  (∠ sum of polygon)
∠z = 36° = 1440°
Each interior angle of the coin = 1440° ÷ 10
11. In the figure, ABCDEF is a regular hexagon and ABGH = 144°
is a square. 1
(b) By symmetry, ∠OAB = ∠BAJ
2
1
= × 144°
2
= 72°
Similarly, ∠OBA = 72°.
Hence
∠AOB = 180° – 72° – 72°  (∠ sum of )
= 36°
(a) Find ∠ABC and ∠CBG.
(b) Find the angles of ACG.
13. (a) Find the angle x in Fig. 1.
(b) Some lines are drawn on Fig. 1 to form a company
Solution
logo as shown in Fig. 2. Find
(a) Sum of the interior angles of a hexagon
(i) ∠GEC, (ii) ∠GCD.
= (6 – 2) × 180°  (∠ sum of polygon)
= 720°
  ∠ABC = 720° ÷ 6
= 120°
∠ABG = 90°  (∠ of square)
∠ABC + ∠ABG + ∠CBG = 360°  (∠s at a point)
120° + 90° + ∠CBG = 360° Fig. 1 Fig. 2
∠CBG = 150°

211
Solution 15. (a) Draw polygons with 5, 6, 7 and 8 sides.
(a) 90° + 4∠x + 90° (b) Draw all the diagonals in each polygon. Copy and
= (6 – 2) × 180°  (∠ sum of polygon) complete the following table.
180° + 4∠x = 720°
∠x = 135° Number of
∠GEC = ∠GCE (base ∠s of isos. )
(b) (i) sides of a 3 4 5 6 7 8
∠GEC + ∠GCE = 108°  (ext. ∠ of ) polygon (n)
2∠GEC = 108° Number of
0 2
∠GEC = 54° diagonals (D)

(ii) ∠GBC = ∠GCB (base ∠s of isos. )
∠GBC + ∠GCB + 108° (c) Suggest a formula connecting n and D.
= 180°  (∠ sum of )
∠GCB = 36° Solution
Hence ∠GCD = ∠x – ∠GCB (a) Pentagon (5 sides)
= 135° – 36°
= 99°

14. (a) In the figure, two identical regular hexagons are


placed side by side as shown.

Hexagon (6 sides)

(i) Calculate the angles x, y and z.


(ii) Can another identical hexagon be placed to
fill the angle z without leaving any gap?
(b) Suppose you have six of these hexagons. Arrange
them to form patterns such that at least one side
of a hexagon is in common with a side of another Heptagon (7 sides)
hexagon.

Solution
(a) (i) Sum of interior angles of a hexagon
= (6 – 2) × 180°
= 720°
∴  ∠x = 720° ÷ 6 Octagon (8 sides)
= 120°
Similarly, ∠y = 120°
∠x + ∠y + ∠z = 360°  (∠s at a point)
120° + 120° + ∠z = 360°
∠z = 120°
(ii) Since ∠z = 120° is the size of an angle of a
regular hexagon, an identical hexagon can be
placed to fill the angle without leaving any
gap.
(b) Some patterns are as follows:
(b) n 3 4 5 6 7 8
D 0 2 5 9 14 20

1
(c) The required formula is D = n(n – 3).
2

Chapter 8  Triangles and Polygons


212
Exercise 8.4 Solution
Level 1 (a) Q
P
1. (a) Construct ABC in which AB = BC = 3 cm and
∠ABC = 90°. F
(b) What type of triangle is ABC?
(c) Find ∠BAC.

Solution
(a) Y 60° 45°
D 7 E
Construction Steps:
1. Draw a line segment DE of length 7 cm.
C 2. Draw an angle PDE of 60°.
3. Draw an angle QED of 45°.
4. Mark the intersection of DP and EQ as F.
5. Draw the line segments DF and BF.
3
Then DEF is the required triangle.
(b) ∠DFE = 180° – 60° – 45° (∠ sum of ∆)
= 75°
DEF is an acute-angled triangle.
X A 3 B
(c) DFE is the largest angle. Hence ∠DFE is opposite
the longest side.
Construction Steps:
1. Draw an angle XBY of 90° using a protractor.
2. With centre B and radius 3 cm, draw two arcs 3. (a) Construct GHK in which
to cut BX at A and BY at C. GH = 5 cm, GK = 4 cm and
3. Draw the line segments AB, AC and BC. ∠HGK = 110°.
Then ABC is the required triangle. (b) Measure the length of HK and
(b) ABC is an isosceles right-angled triangle. give your answer correct to
the nearest 0.1 cm.
(c) ∠BAC = ∠ACB (base ∠s of isos. ∆) (c) Classify GHK by its sides.
∠BAC + ∠ACB + ∠ABC = 180° (∠ sum of ∆)
∴ 2∠BAC + 90° = 180° Solution
∠BAC = 45° (a)

2. (a) Construct DEF in which DE = 7 cm,


∠FDE = 60° and ∠DEF = 45°.
(b) Classify DEF by its angles.
(c) Which angle is opposite the longest side?

Construction Steps:
1. Draw a line segment GH 5 cm long.
2. Using a protractor, draw a ray through G making
an angle of 110° with GH.
3. With G as centre and 4 cm as radius, draw an
arc to cut the ray at K.

213
4. Draw the line segment HK. Then GHK is the 3. Mark a point D on the ray such that AD = 2 cm.
required triangle. 4. With B as centre and 2 cm as radius, draw an arc.
5. With D as centre and 3 cm as radius, draw an arc to
(b) HK = 7.4 cm
cut the previous arc at C.
(c) GHK is a scalene triangle. 6. Draw the line segments BC and CD. Then ABCD is
the required parallelogram.
4.
6. Construct a rhombus PQRS in which PR = 4 cm and
QS = 3 cm.

Solution
(a) Construct LMN in which LM = 4 cm,
∠NLM = ∠LMN = 45°.
(b) Measure ∠LNM and give your answer correct to
the nearest degree.
(c) Measure the lengths of LN and MN and give your
answers correct to the nearest 0.1 cm.
(d) What type of triangle is LMN?

Solution
(a)
Construction Steps:
1. Draw a line segment PR 4 cm long and mark its
midpoint M.
2. Draw a line segment SMQ perpendicular to PR such
that SM = MQ = 1.5 cm.
3. Draw the line segments PQ, QR, RS and SP. Then
PQRS is the required rhombus.
Construction Steps:
1. Draw a line segment LM 4 cm long.
7. Construct a trapezium KLMN in which KL // NM,
2. Using a protractor, draw a ray with end point
KL = 4 cm, LM = 3 cm, NK = 2.5 cm and ∠K = 90°.
L and making an angle of 45° with LM.
3. Using a protractor, draw a ray with end point
M and making an angle of 45° with LM such
that it cuts the previous ray at N. Then LMN
is the required triangle.
(b) ∠LNM = 90° (correct to the nearest degree)
(c) LN = 2.8 cm  (correct to the nearest 0.1 cm)
MN = 2.8 cm  (correct to the nearest 0.1 cm)
(d) LMN is a right-angled isosceles triangle.

5. Construct a parallelogram ABCD in which AB = 3 cm,


AD = 2 cm and ∠A = 70°. Solution
Construction Steps:
Solution 1. Draw a line segment KL 4 cm long.
2. Draw a ray with end point K and perpendicular to
KL.
3. Mark a point N on the ray such that KN = 2.5 cm.
4. Draw a ray with end point N, perpendicular to KN
and on the same side of KN as the line segment KL.
5. With L as centre and 3 cm as radius, draw an arc to
cut the previous ray at M and M′.
6. Draw the line segments LM and LM′. Then KLMN
and KLM′N are two possible solutions of the required
Construction Steps:
trapezium.
1. Construct a line segment AB 3 cm long.
2. Draw a ray with end point A and making an angle
of 70° with AB.

Chapter 8  Triangles and Polygons


214
8. Construct a quadrilateral XYZT in which XY = 3.5 cm, Solution
YZ = 3.5 cm, ZT = 2 cm, TX = 3 cm and TY = 4 cm. (a)

Solution


Construction Steps:
Construction Steps: 1. Draw a line segment AB 4 cm long.
1. Draw a line segment XY 3.5 cm long. 2. Draw a ray with end point B and making an
2. With X as centre and radius 3 cm, draw an arc. angle of 30° with AB.
3. With Y as centre and radius 4 cm, draw an arc to cut 3. With A as centre and 4 cm as radius, draw an
the previous arc at T. arc to cut the ray at C.
4. With T as centre and radius 2 cm, draw an arc. 4. Join A and C. Then ABC is the required
5. With Y as centre and radius 3.5 cm, draw an arc to triangle.
cut the previous arc at Z.
6. Draw the line segments YZ, ZT and TX. Then XYZT (b) ∠BAC = 120° (correct to the nearest degree)
is the required quadrilateral. ∠ACB = 30° (correct to the nearest degree)
(c) ABC is an isosceles triangle.

Level 2
11. Construct a quadrilateral ABCD in which AD = 4 cm,
9. (a) Construct XYZ in which YZ = 2.5 cm,
BC = 2 cm, CD = 3 cm, ∠C = 120° and ∠D = 100°.
XZ = 6.5 cm and ∠XYZ = 90°.
(b) Measure the length of XY and give your answer
Solution
correct to the nearest 0.1 cm.
(c) Classify XYZ by its sides.

Solution
(a)

Construction Steps: Construction Steps:


1. Draw a line segment YZ 2.5 cm long. 1. Draw a line segment CD 3 cm long.
2. Using a protractor, draw a ray with end point 2. Draw a ray with end point C and making an angle
Y and making an angle of 90° with YZ. of 120° with CD.
3. With Z as centre and 6.5 cm as radius, draw an 3. Mark a point B on the ray such that BC = 2 cm.
arc to cut the ray at X. 4. Draw a ray with end point D on the same side of CD
4. Join X and Z. Then XYZ is the required as BC such that it makes an angle of 100° with CD.
triangle. 5. Mark a point A on the previous ray such that
(b) XY = 6.0 cm  (correct to the nearest 0.1 cm) AD = 4 cm.
(c) XYZ is a scalene triangle. 6. Join A and B. Then ABCD is the required quadrilateral.

10. (a)
Construct ABC in which AB = 4 cm, AC = 4 cm
and ∠ABC = 30°.
(b) Measure ∠BAC and ∠ACB and give your answers
correct to the nearest degree.
(c) Classify ABC by its sides.

215
Level 3 Construction Steps:
12. (a) Construct ABC in which AB = 4.5 cm, 1. Draw a line segment AB.
AC = 4.5 cm and ∠BAC = 130° using Sketchpad. 2. Draw two circles with centres at A and B and
(b) Measure ∠ABC and ∠ACB correct to the nearest equal radii AB.
degree. 3. Mark C as one of the intersecting points of the
(c) Draw a perpendicular line from A to meet the line circles.
BC at D. 4. Draw the line segments AC and BC. Then ABC
(d) Measure the lengths of BD and CD and give your is an equilateral triangle.
answers correct to the nearest 0.1 cm. (b) Use the midpoint command to create the midpoints
(e) What do you observe from the result in (d)? D, E, F of the sides AB, BC and CA.
(c) Draw the line segments DE, EF and FD to form
Solution DEF.
(a)
(d) DEF is an equilateral triangle.
DE
(e) = 0.5
AB

14. In ABC, AB = 4 cm, BC = 3 cm and ∠BAC = 30°.


(a) Construct ABC.
(b) How many different triangles that satisfy the given
measurements can you construct?
(c) What can you conclude about constructing a triangle
given two sides and an angle that is not included
between the two sides?
Construction Steps:
1. Draw a line segment AB 4.5 cm long. Solution
2. Rotate AB about A for 130° to AC.
3. Join B and C. Then ABC is the required
triangle.
(b) ∠ABC = 25° (correct to the nearest degree)
∠ACB = 25° (correct to the nearest degree)
(d) BD = 4.1 cm (correct to the nearest 0.1 cm)
CD = 4.1 cm (correct to the nearest 0.1 cm.)
(e) When AB = AC, the perpendicular AD from A to
BC bisects BC.
Construction Steps:
1. Draw a line segment AB 4 cm long.
13. (a) Draw an equilateral triangle ABC using Sketchpad.
2. Draw a ray with end point A and making an
(b) Plot the midpoints D, E and F of the sides AB, BC
angle of 30° with AB.
and CA.
3. With B as centre and 3 cm as radius, draw an
(c) Draw DEF.
arc to cut the ray at C and C′.
(d) What type of triangle is DEF?
4. Draw the segments BC and BC′. Then ABC
DE
(e) Find the value of . and ABC′ are two solutions of the required
AB
triangle.
(b) Two triangles.
Solution
(a) (c) When two sides and a non-included angle of a
triangle are given, sometimes we can construct two
triangles.

1 5. (a) Draw two different quadrilaterals ABCD in which


AB = CD = 3 cm and AD = BC = 2 cm.
(b) What types of quadrilateral would ABCD be?

Chapter 8  Triangles and Polygons


216
Solution 2. Find the angles x and y in the following figures.
(a) (a) (b)

Solution
(a) ∠x + 33° = 115°  (ext. ∠ of )
∠x = 82°
∠y + 36° = ∠x    (ext. ∠ of )
∠y + 36° = 82°
(b) ABCD can be a rectangle or a parallelogram. ∠y = 46°
(b) ∠x = 35° + 31°  (ext. ∠ of )
= 66°
Revision Exercise 8 ∠AST = 30° + 36°  (ext. ∠ of )
1. It is given that AB = 7 cm, BC = 2 cm, CA = 4 cm, = 66°
PQ = 5 cm, QR = 6 cm and RP = 4 cm. ∠y + ∠x + ∠AST = 180°  (∠ sum of )
(a) Determine which triangle, ABC or PQR, can ∠y + 66° + 66° = 180°
be constructed. ∠y = 48°
(b) Draw the triangle.
(c) State the biggest angle of the triangle. 3. Find the sum of all the marked angles in each figure.
(a) AOD, BOE and FOC are straight lines.
Solution
(a) BC + CA = 2 + 4
= 6 cm
< AB
∴ ABC cannot be constructed.
PQ + QR > RP
PQ + RP > QR
QR + RP > PQ
∴ PQR can be constructed.
(b) R (b) APQD, BPSG, CQRF and HSRE are straight lines.

6
4

P 5 Q Solution
(a) ∠a + ∠b = ∠BOD  (ext. ∠ of )
Construction Steps: ∠c + ∠d = ∠DOF  (ext. ∠ of )
1. Draw a line segment QR of length 6 cm. ∠e + ∠ f = ∠FOB  (ext. ∠ of )
2. With centre Q and radius 5 cm, draw an arc   ∠a + ∠b + ∠c + ∠d + ∠e + ∠ f
above QR. = ∠BOD + ∠DOF + ∠FOB
3. With centre R and radius 4 cm, draw an arc to = 360°  (∠s at a point)
cut the first arc at P.
4. Draw the line segments PQ and PR.
Then ∆PQR is the required triangle.
(c) Since QR is the largest side, ∠QPR is the biggest
angle of the triangle.

217
(b) By ext. ∠ of , Solution
∠a + ∠b = ∠APS, (a) ∠PSQ = ∠x (base ∠s of isos. )
∠c + ∠d = ∠CQP, ∠PSQ + ∠x + 112° = 180°  (∠ sum of )
∠e + ∠ f = ∠ERQ, ∠x + ∠x + 112° = 180°
∠g + ∠h = ∠GSR. 2∠x = 68°
∠a + ∠b + ∠c + … + ∠h ∠x = 34°
= ∠APS + ∠CQP + ∠ERQ + ∠GSR ∠y + 112° + ∠x + 80° + 57°
= 360°  (ext. ∠ sum of polygon) = 360°  (∠ sum of polygon)
∠y + 112° + 34° + 80° + 57° = 360°
∠y = 77°
4. Find the value of x in the following figures.
(a) (b) ∠WRQ = 26°  (vert. opp. ∠s)
∠x + ∠WRQ = 73°  (opp. ∠s of //gram)
∠x + 26° = 73°
∠x = 47°
∠x + ∠y = 180° (int. ∠s, SR // PQ)
47° + ∠y = 180°
∠y = 133°
(b)
6. In the figure, PQRS is a
rhombus and PE = 4 cm.
(a) Find the length of PR.
(b) Find the angles x, y
and z.

Solution
Solution (a) PR = 2PE  (diagonals of rhombus)
3x°
(a)
x° + 2x° + = 180°  (∠ sum of ) = 2 × 4 cm
2
= 8 cm
2x° + 4x° + 3x° = 360°
9x° = 360° (b) ∠z = 90°  (diagonals of rhombus)
x = 40 ∠QPR = 35°  (diagonals of rhombus)
∠x = ∠QPS (opp. ∠s of rhombus)
(b) 3x° + 5x° + 4x° = 360°  (ext. ∠ sum of polygon) = 35° + 35°
12x° = 360° = 70°
x = 30 ∠x + ∠y = 180°  (int. ∠s, PQ // SR)
70° + ∠y = 180°
5. Find the angles x and y in the following figures. ∠y = 110°
(a) ∠y = ∠PSR.
7. In the figure, ABCD is
a rectangle, CEF is
equilateral and BE = 3 cm.
Find
(a) the length of AC,
(b) the angles x, y and
z.
(b) QRV and WRU are straight lines.
Solution
(a) BD = 2BE  (diagonals of rectangle)
= 2 × 3 cm
= 6 cm
AC = BD  (diagonals of rectangle)
= 6 cm
(b) ∠x + 65° = 90°  (∠ of rectangle)
∠x = 25°
∠ADE = 65°  (base ∠s of isos. )

Chapter 8  Triangles and Polygons


218
∠y + 65° + ∠ADE =
180°  (∠ sum of ) Solution
∠y + 65° + 65° = 180° (a) Each exterior angle
∠y =50° = 360° ÷ 5  (ext. ∠ sum of polygon)
∠FEC =60°  (∠ of equilateral ) = 72°
∠DEF =180° – ∠y – ∠FEC 
(b) ∠BAE = 180° – 72°  (adj. ∠s on a st. line)
(adj. ∠s on a st. line)
= 108°
= 180° – 50° – 60°
= 70° (c) ∠CDF = 60°  (∠ of equilateral )
∠EDF = ∠z (base ∠s of isos. ) ∠EDF = 108° – 60°
∠EDF + ∠z + ∠DEF = 180°  (∠ sum of ) = 48°
∠z + ∠z + 70° = 180° ∠DEF = ∠DFE  (base ∠s of isos. )
∠z = 55° ∠DEF + ∠DFE + ∠EDF = 180°  (∠ sum of )
∠DFE + ∠DFE + 48° = 180°
∴  ∠DFE = 66°
8. Find the unknown angle x in the following figures.
(a) (b)

10. In the diagram, ABCDEF is a regular hexagon and


ABPQRSTU is a regular octagon.

Solution
(a) ∠AED = 180° – 78°  (adj. ∠s on a st. line) Find
= 102° (a) ∠ABC,
90° + ∠x + 104° + ∠x + 102° (b) ∠ABP,
= (5 – 2) × 180°  (∠ sum of polygon) (c) ∠CBP,
2∠x + 296° = 540° (d) ∠CAP.
2∠x = 244°
∠x = 122° Solution
(b) (a) Sum of interior angles of a hexagon
M = (6 – 2) × 180°  (∠ sum of polygon)
A
= 720°
x
B ∠ ABC = 720° ÷ 6
F = 120°
80° G
32° H
K 65° (b) Sum of interior angles of an octagon
E C = (8 – 2) × 180°  (∠ sum of polygon)
40° D = 1080°
75°
J I ∠ABP = 1080° ÷ 8
= 135°
Produce BA to M.
∠FAM = 180° – ∠x (adj. ∠s on a st. line) (c) ∠CBP = 360° – ∠ABC – ∠ABP (∠s at a point)
(180° – ∠x) + 65° + 40° + 75° + 32° + 80° = 360° – 120° – 135°
= 360°  (ext. ∠ sum of polygon) = 105°
∠x = 112° (d) ∠ACB = ∠CAB (base ∠s of isos. )
∠ACB + ∠CAB + ∠ABC = 180°  (∠ sum of )
8. In the diagram, ABCDE is a ∴  ∠CAB + ∠CAB + 120° = 180°
regular pentagon and CDF ∠CAB = 30°
is equilateral. Similarly,
1
(a) Find the size of an exterior ∠PAB = × (180° – 135°)
2
angle of the pentagon.
(b) Find ∠BAE. = 22.5°
(c) Find ∠DFE. ∴  ∠CAP = ∠CAB + ∠PAB
= 30° + 22.5°
= 52.5°

219
11. (a) Find the sum of the interior angles of a 20-sided Construction Steps:
polygon. 1. Draw a line segment AB 3 cm long.
(b) If the sum of the interior angles of an n-sided 2. With A as centre and radius 3 cm, draw an arc.
polygon is 2520°, find the value of n. 3. With B as centre and radius 4 cm, draw an arc to cut
the previous arc at D.
Solution 4. With D as centre and radius 2.5 cm, draw an arc.
(a) Sum of interior angles of a 20-gon 5. Withe B as centre and radius 3.5 cm, draw an arc to
= (20 – 2) × 180°  (∠ sum of polygon) cut the previous arc at C.
= 3240° 6. Draw the line segments BC, CD and DA. Then ABCD
(b) (n – 2) × 180° = 2520°  (∠ sum of polygon) is the required quadrilateral.
n – 2 = 14
n = 16 14. In the figure, ABCDEF is a regular hexagon, ABGH is
a square and AB = 3 cm.
12. In the figure, AB, BC and CD are three sides of a regular
polygon, ∠ABC = 144°, AB and DC are produced to
meet at E. Find
(a) the number of sides
of the polygon,
(b) ∠BEC,
(c) ∠BAC.
(a) Draw the figure.
(b) Find ∠FAH.
Solution (c) Find ∠FAG.
(a) Let n be the number of sides of the polygon, (d) Find ∠AFH.
∠EBC = 180° – 144°   (adj. ∠s on a st. line)
= 36° Solution
36° × n = 360°  (ext. ∠ sum of polygon) (a)
n = 10
The polygon has 10 sides.
(b) ∠ECB = ∠EBC (ext. ∠ of regular polygon)
= 36°
∠BEC = 180° – ∠ECB – ∠EBC (∠ sum of )
= 180° – 36° – 36°
= 108°
(c) ∠BCA = ∠BAC (base ∠s of isos. )
∠BCA + ∠BAC + ∠ABC = 180°  (∠ sum
of )
∠BAC + ∠BAC + 144° = 180°
∠BAC = 18°

13. Draw a quadrilateral ABCD in which AB = 3 cm,


BC = 3.5 cm, CD = 2.5 cm, DA = 3 cm and Construction Steps:
BD = 4 cm. 1. Draw a line segment AB 3 cm long.
2. With A and B as centres and 3 cm as the radius,
Solution draw two arcs that intersect at O.
3. Draw a circle with centre O and radius 3 cm.
4. Mark the vertices of the hexagon on the circle
using arcs of radius 3 cm.
5. Draw the sides of the hexagon ABCDEF.
6. Construct the square ABGH by drawing the
sides AH and BG perpendicular to AB using a
protractor.

Chapter 8  Triangles and Polygons


220
(b) Sum of interior angles of a hexagon
= (6 – 2) × 180°  (∠ sum of polygon)
= 720°
∠FAB = 720° ÷ 6
= 120°
∠HAB = 90°  (∠ of square)
∠FAH = ∠FAB – ∠HAB
= 120° – 90°
= 30°
(c) ∠HAG = 45°  (diagonals of square)
∠FAG = ∠FAH + ∠HAG
= 30° + 45°
= 75°
(d) ∠AHF = ∠AFH (base ∠s of isos. )
∠AHF + ∠AFH + ∠FAH = 180°  (∠ sum of )
∠AFH + ∠AFH + 30° = 180°
  ∠AFH = 75°

15. In the figure, ABCDEFGHI is a regular 9-sided polygon.


(a) Find ∠ABC.
(b) If AC and BI intersect at P, find ∠CPI.

F
Solution
G E
(a) Angle sum of the
polygon
= (9 – 2) × 180° H D
(∠ sum of polygon)
= 1260° I C
∴ ∠ABC = 1260° ÷ 9
= 140° A B

(b) ∠BAC = ∠ACB (base ∠s of isos. )


∠BAC + ∠ACB + ∠ABC = 180° (∠ sum of )
∴ 2∠BAC + 140° = 180°
∠BAC = 20°
Similarly, ∠AIB = 20°
∠IAP = ∠IAB – ∠BAC
= 140° – 20°
= 120°
In ∆API,
∠CPI = ∠AIB + ∠IAP (ext. ∠ of )
= 20° + 120°
= 140°

221
Review Exercise 2 Solution
1. Simplify the following. Let $y be the price of the printer.
(a) –2(3m + 4n – 7) + 5(2m – 6n + 3) Then the price of the computer is $5y.
5(2x + 1) 4x – 7
(b) – y + 5y = 1320
3 2

Solution 6y = 1320
(a) –2(3m + 4n – 7) + 5(2m – 6n + 3) y = 220
= –6m – 8n + 14 + 10m – 30n + 15
= 4m – 38n + 29 The price of the printer is $220.
(b) 5(2x + 1) 4x – 7

3 2
5. Tom usually works x hours in a week at $14 per hour.
10(2x + 1) – 3(4x – 7) 1
= The hourly wages for overtime is 1 times the
6 2
20x + 10 – 12x + 21
normal hourly wages. In a certain week, he worked 54
= hours and his total salary was $819. Find the value of x.
6

8x + 31
= Solution
6
14x + (14)(1.5)(54 – x) = 819
14x + 21(54 – x) = 819
2. Factorise the following. 14x + 1134 – 21x = 819
(a) 3ax – 6ay + 21a 7x = 315
(b) 6(kt – 3d ) – 2(2ky – 9d) x = 45

Solution 6. In the diagram, AB//DE, ∠BCD = 59° and ∠CDE = 105°.


(a) 3ax – 6ay + 21a Find ∠ABC.
= 3a (x – 2y + 7)
(b) 6(kt – 3d) – 2(2ky – 9d)
= 6kt – 18d – 4ky + 18d
= 6kt – 4ky
= 2k(3t – 2y)

3. Solve the following equations.


(a) 5(2x – 3) = 3(x + 2)
2x 7(x – 4) Solution
(b) + = 25
5 2 D E
105°
Solution
A B X Y
(a) 5(2x – 3) = 3(x + 2)
10x – 15 = 3x + 6 59°

C
7x = 21
x = 3 ∠CXY = Find ∠XDE (corr. ∠s)
2x 7(x – 4) = 105°
(b) + = 25 ∠BXC = 180° – ∠CXY (adj. ∠s on st. line)
5 2
2(2x) + 35(x – 4) = 250 = 180° – 105°
= 75° ∠ABC = ∠BXC + ∠BCX (ext. ∠ of )
4x + 35x – 140 = 250
= 75° + 59°
39x = 390 = 134°
= 10
x
7. (a) Draw a triangle ABC in which AB = 6 cm,
BC = 8 cm and ∠ABC = 73°.
4. The price of a computer is 5 times as much as that of a
(b) Using ruler and compasses, construct
printer. If the total price of the computer and the printer
(i) the angle bisector of ∠ABC,
is $1320, find the price of the printer.
(ii) the perpendicular bisector of AC.

Review Exercise 2
222
Solution ∠CBA = 180° – ∠CBF (adj. ∠s on st. line)
(a) = 180° – 115°
X
= 65°
C ∠DCB = 360° – ∠ADC – ∠DAB – ∠CBA (∠ sum
of polygon)
= 360° - 113° - 80° - 65°
= 102°

9. Three of the angles of a pentagon are 118° each. If the


8 other two angles are x° and (x + 16)°, find the value of x.

Solution
3(118°) + x° + (x + 16)°
= (5 – 2) × 180° (∠ sum of polygon)
2x + 370 = 540
2x = 170
x = 85

73° 10. Carol measured an exterior angle of a regular polygon


and said that it is 50°.
A 6 B
(a) Mario immediately said that her answer was
wrong. Do you know why Mario was able to know
(b) immediately that the answer was wrong?
X
(b) If Carol’s measurement is very close to the size of
C the angle, find the number of sides of the polygon.

Solution
(a) Number of sides
= 360
50
= 7.2
8
(ii) It is impossible to have an exterior angle of 50° as
the number of sides is not a positive integer.
(b) 7 sides.

11. The diagram shows a pattern of n regular hexagons


formed by toothpicks.
(i)

73°
(a) Copy and complete the following table.
A 6 B
n 1 2 3 4
8. In the diagram, EABF is a straight line, Total number
∠EAD = 100°, ∠FBC = 115° and ∠ADC = 113°. Find of toothpicks
∠BCD.
(b) Express the total number of toothpicks in terms of
n if there are n regular hexagons in the pattern.
(c) Find the value of n if the total number of toothpicks
needed to form the pattern of hexagons is 76.

Solution
Solution (a) n 1 2 3 4
∠DAB = 180° – ∠EAD (adj. ∠s on st. line)
Total number
= 180° – 100° 6 11 16 21
of toothpicks
= 80°

223
(b) Total number of toothpicks = 5n + 1 (i) the angle bisector of ∠BAC,
(c) 5n + 1 = 76 (ii) the perpendicular bisector of AC.
5n = 75 (c) Label the point T on the diagram in (b) which
n = 15 is equidistant from the points A and C, and also
equidistant from the lines AB and AC.

12. Paul’s monthly salary is $3x. Queenie’s monthly salary Solution


is $600 more than Paul’s. Robert’s monthly salary is (i) Obtuse triangle
two-third of Queenie’s one.
(a) Express Queenie’s monthly salary in terms of x. C
(b) Express Robert’s monthly salary in terms of x.
(c) If the sum of monthly salaries of Paul, Queenie 7
and Robert is $10 600, find the monthly salary of
Paul. 45°
A 9 B
Solution
(a) $(3x + 600) (ii) Acute triangle
2
(b) $(3x + 600) × = $(2x + 400) C
3

(c) (3x + 600) + (2x + 400) + x = 10 600


6x + 1000 = 10600
6x = 9600
x = 1600
3x = 4800
Paul’s monthly salary is $4800.
7
13. In the diagram, BCD is a straight line, BA//DE,
AC = AD, ∠ABC = 40° and ∠CAD = 66°. Find
(a) ∠CDA,
(b) ∠BAC,
(c) ∠ADE.
Give reasons to support your workings.
45°
A 9 B

(b)

Solution
180° – 66°
(a) ∠CDA = 75° (base ∠ of isos. )
2
(ii)
= 570°
(b) ∠BAC = 180° – 40° – 57° – 66° (∠ sum of )
= 170 7
(i)
(c) ∠ADE = ∠BAD (aH. ∠s)
∠ADE = ∠BAC + ∠CAD T
= 170° + 66°
= 83°

14. In ABC, AB = 9 cm, BC = 7 cm and ∠BAC = 40°. 45°


(a) Using ruler, protractor and compasses, draw two A 9 B
possible shapes of the triangle.
(b) For the acute-angled ABC drawn in (a), using
ruler and compasses, construct

Review Exercise 2
224
15. An interior angle of a regular 3n-sided polygon is 48°
more than that of an interior angle of a regular n-sided
polygon. Find
(a) the value of n,
(b) the size of an exterior angle of the n-sided
polygon.

Solution
(a) Exterior angle of 3n-sided polygon
360°
= 3n

Interior angle of 3n-sided polygon
360°
= 180° –
3n
Exterior angle of n-sided polygon
360°
=
n

Interior angle of n-sided polygon
360°
= 180° –
n
360° 360
180° – – 48 = 180 –
3n n
360 360°
– = 48
n 3n

1080 - 360 = 144n


144n = 720
n=5

(b) Size of an exterior angle of n-sided polygon
360°
=
5

= 72°

225

S-ar putea să vă placă și